Categories
Economics Programs Harvard Undergraduate

Harvard. Economics Department Reports to the Dean, 1946-47 to 1949-50

 

This post adds the Chair’s annual reports on the Harvard Economics Department for the early post-WW II years to previously posted reports for 1932-33 through 1945-46. 

Reports to the Dean of Harvard
from the Department of Economics
.
1932-1941
1941-1946

___________________________

1946-1947

September 29, 1947

Dear Dean Buck:

You have requested a brief report on the work of the Department of Economies for the academic year 1946-47.

This report necessarily follows much the same pattern as the report for last year. Again our work has been dominated by the number of students, undergraduate and graduate, and the lack of a trained junior staff.

The number of undergraduates of course is entirely so beyond our control. In Economies A and in most of our “middle group” courses, the elections taxed our capacity for effective instruction. Under the most propitious conditions the crowded classrooms would have presented many problems but with a dearth of trained teaching fellows and annual instructors the load carried by the senior staff was unduly heavy. Foreseeing this range of problems, the Department voted on February 19, 1946 [sic, 1947 probably correct. In December 1946 departments wereallowed to withdraw from offering tutorials] to suspend tutorial instruction for a period of two years. It may be stated here that this was probably a wise decision. Concentration in Economics appears to have resumed the trend apparent before the war. In the current year the number of concentrators will approach, or perhaps exceed 800. Even should no consideration be given to the expenditure involved, the possibility of finding and training effective tutors even for honors candidates seems somewhat remote.

On the graduate level the problems of instruction were even more difficult. During the year the number of graduate students receiving instruction was approximately 286. Our course offering on this level is large. Nevertheless, the principal graduate courses were crowded to a point where the maintenance of standards was difficult. After the graduate student has completed his preliminary program and has been accepted as a candidate for the Ph.D, degree, the instruction is largely individual. In the last year we were just coming into the situation where a considerable proportion of the students were receiving such instruction. The full impact of this situation will be felt in the current year. Most members of the senior staff will be directing the theses of some 10 to 15 students. Some officers will be responsible for even larger numbers. With the numbers we are attempting to handle on the graduate level the single task of examining candidates in the general and special examinations becomes a major consideration. During the last academic year the staff conducted general and special examinations. Such an amount of examining and of individual instruction on the graduate level has its bearing on tutorial instruction for undergraduates.

The Department voted to accept the large number of graduate students now on our rolls only after considerable investigation and discussion. It is my own personal opinion that we have set our limit altogether too high. However, the pressure upon us for admission has been very strong and our obligations to the Littauer School, where the pressure is hardly less, just be observed.

This matter of the size of the Graduate School in the immediate future is one of our most difficult problems. It will receive our attention in the current year.

In the last two or three years these reports have noted certain experiments in instruction, especially in connection with Economics A. Such experiments are dependent upon the presence of a considerable number of able and mature young men with adequate teaching experience, as well as upon a margin of free time. Both of these factors are lacking to such a degree that substantial and outstanding progress could not be expected but the plans were active and some progress was made.

If full tutorial instruction is not resumed by the Department, experimentation in undergraduate courses is imperative and this we have planned. It is our expectation that a good deal in the way of individual guidance can be accomplished in connection with Economics A and some of our middle group courses. We believe that we can make our instruction more efficient with a much smaller personnel and at much less expense than the tutorial system would involve. However, a definitive decision has not been reached on all of these matters.

It is hardly necessary to emphasize that the heavy instructional demands discussed above affected our research projects. Furthermore, the officers of this Department are severely handicapped by the lack of research funds. This dearth of research funds is a question which has been placed before our Visiting Committee.

In spite of the difficulties involved, the contributions of the members of the Department were substantial. The following books were published:

Teoria de la Competencie Monopolica, by E. H. Chamberlin, Mexico, 1946. (Spanish translation of The Theory of Monopolistic Competition)

Economic Policy and Full Employment, by A. H. Hansen. McGraw-Hill. 1947.

The New Economics, S. B. Harris, editor and contributor Knopf. 1947.

The National Debt and the New Economics, by S. E. Harris. 1947.

Income and Employment, by T. Morgan. Prentice-Hall. 1947.

New enlarged edition of Capitalism, Socialism, and Democracy, by J. A. Schumpeter.

The Challenge of Industrial Relations, by S. H. Slichter, Cornell University Press, 1947.

Postwar Monetary Plans and other Essays, by J. Williams. Knopf, 3rd edition. 1947.

articles were published.

Although we are able to record only one new volume and one republication of an older volume in the Harvard Economic Series for the past year, four other volumes are in the hands of the printer and will appear in the current year.

In the area of distinctions or honors, I believe the only items to be noted concern Dean Edward S. Mason. Last spring he was appointed Economic Advisor to Secretary of State Marshall at the Moscow Conference. In July he was appointed a member of President Truman’s Committee on Foreign Aid.

Sincerely yours,
H. H. Burbank

Dean Paul H. Buck

Source: Harvard University Archives. Department of Economics, Correspondence and Papers (UAV 349.11), Box 2, Folder “Provost Buck—Annual Report of Dept.”

___________________________

1947-1948

September 30, 1948

Dear Provost Buck:

You have requested a brief report on the work of the Department of Economics for the academic year 1947-48.

The report on the work of the Department for the last year can be given in part in the same terms that have been employed in the last three reports. Our major problems have been quantitative and have presented the same difficulties that were emphasized in the other post-war reports. However, we believe that the last year did reach the peak of the load and that the pressure of numbers will abate steadily. The problem of building and maintaining an effective junior staff was hardly less than in the preceding years. Crowded classrooms and insufficiently trained assistants imposed unduly severe burdens upon the senior teachers responsible for course instruction. Some improvement, especially in the middle group courses, is in prospect for the coming year but it is probable that two to three years more will be necessary before these courses will be adequately staffed. In the introductory course which relies heavily upon a large number of young instructors and teaching fellows, the situation is still serious but latterly we have been able to utilize young men with more satisfactory preparation and training. Because of the heavy demands for the services of these young men by other institutions, the turnover is large leaving us each year with a relatively inexperienced staff.

Graduate instruction continues to make unusual demands upon the time and energy of the senior staff. During the past year we conducted 109 general examinations and 26 special examinations. Examining and the related task of directing the research of candidates for the higher degrees undoubtedly have an incidence upon undergraduate instruction which raises questions of fundamental importance. It is encouraging that the number of graduate students is, through the action of the Department, declining.

In spite of the difficulties presented by the numbers of undergraduates and graduates, the Department, perhaps belatedly, has given particular consideration to its commitments in the Areas and in General Education. A report on General Education is enclosed.

Also, the Department has considered at length and in detail various problems of instruction, particularly undergraduate instruction. These considerations will be continued in the current year. By completely revising the content of our basic courses it may be possible to increase the effectiveness of our instruction and reduce somewhat the number of courses offered. A preliminary report on this aspect of our work is included.

A year ago I noted that many of our senior officers were handicapped severely by the lack of research funds. As you know, it can now be recorded with sincere satisfaction that a grant from the Rockefeller Foundation and that several projects under the auspices of the Research Marketing Act, U.S. Department of Agriculture, the Charles H. Hood Dairy Foundation, the Ferguson Foundation Fund, and the Carnegie Corporation Fund, meet the situation effectively for some of our officers. The set-up of these projects promises not only to be of great value to the professors in charge of the research but it contributes heavily to the training of our most promising graduate students and younger officers.

The following books were published by members of the Department:

How Shall We Pay for Education? by Seymour Harris. Harpers.

Stabilization Subsidies by Seymour Harris. Historical Report Series, U.S. Gov’t.

Price Control of International Commodities by Seymour Harris. Archives Volume, Historical Records Office.

International Monetary Policies, by Gottfried Haberler (with Lloyd Metzler and Robert Triffin). Postwar Economic Series, Federal Reserve System Board of Governors.

Problemas de Conjuntura e de Politica Economica, by Gottfried Haberler. Fundacao Getulio Vargas, Rio de Janiero.

Production in the United States, 1866-1914, by Edwin Frickey. Harvard University Press.

Seventy-eight articles have been published. Three books were published in the Harvard Economic Series during the past year. Five volumes are in the hands of the Press to be published later this year.

Professor Edward H. Chamberlin has been appointed to succeed Dr. Arthur B. Monroe as Managing Editor of the Quarterly Journal of Economics. Both the Quarterly Journal of Economies and the Review of Economic Statistics are well established intellectually and financially. With the demands of instruction and research, the editing of the Quarterly Journal of Economics and the Review of Economics and Statistics, as well as the direction of the Harvard Economic Series, raises questions regarding the adequacy of the manpower within the Department.

 In the area of distinctions or honors, Professor Joseph A. Schumpeter was chosen to be President of the American Economic Association for 1948. Dean Edward S. Mason was awarded an honorary degree, D. Litt, from Williams College, June, 1948.

Very sincerely,
H. H. Burbank

Provost Paul H. Buck
5 University Hall

Source: Harvard University Archives. Department of Economics, Correspondence and Papers (UAV 349.11), Box 2, Folder “Provost Buck—Annual Report of Dept.”

___________________________

1948-1949

September 28, 1949

Dear Provost Buck:

The pattern of the report of the Department of Economics on the work of the last year is essentially the same as the other reports for the post-war years. Indeed, not a little of the introduction to the report of a year ago could be utilized in the current report. The quantitative side of our work has been among our major problems. I think I was correct in predicting that the peak of the load would be passed in 1948-49. For the year 1949-50, numbers, particularly on the graduate level, will be approximately less although the total is still beyond the capacities of our senior staff.

Again I can repeat that the problem of building and maintaining a junior staff presents great difficulties. We have strengthened our position on the level of the assistant professor but we are unable to hold our most promising young Ph.D’s for appointment at the instructor level. All of our undergraduate instruction suffers because of this factor, but Economics 1 (the introductory course) is affected particularly. The demand for these young men by other institutions continues at a high level resulting in a high rate of turnover and leaving us sech year with a relatively inexperienced staff. [end of p. 1]

[Note: need to replace unfocussed image of page 2]

[p. 3 begins ] …expectation that we will be able to revise our general examination effectively.

In the post-war years the Department has been striving to meet its obligations to General Education and to the areas. We believe that we have made an excellent beginning in both General Education and in the Russian Area. We are still actively engaged in the attempt to strengthen our position in the Chinese Area. This is exceedingly difficult but I believe that some progress is being made.

Last year we were able to record with great satisfaction that some research projects were being established satisfactorily. These projects under the auspices of the Rockefeller Foundation and under the auspices of various groups interested in agriculture and marketing are now going forward successfully and up proving to be important for us not only as research projects but also because of their general effect upon a relatively large group of our graduate students. We can now give a type of training to our most promising men which would have been impossible without such projects. It should be emphasized at this point that other areas of interest need research funds.

The following books were published:

Collective Bargaining: Principles and Cases, Richard D. Irwin, Inc., 1949, by John I. Dunlop.

Labor in Norway by Walter Galenson. Harvard University Press, 1949.

Monetary Theory and Fiscal Policy, by Alvin Hansen McGraw-Hill, 1949.

The European Recovery Program, by Seymour E. Harris. Harvard University Press.

Foreign Economic Policy for the U.S., edited by Seymour E. Harris, Harvard University Press.

Price Control of International Commodities, by Seymour E. Harris. Archives Volume for Historical Records Office.

Saving American Capitalism, edited by Seymour E. Harris. Knopf.

Economic Planning, by Seymour E. Harris. Knopf.

Post-war Monetary Plans and Other Essays, by John H. Williams. Oxford, Basil Blackwell.

The American Economy, Its Problems and Prospects, by Sumner H. Slichter. Knopf.

There were 62 articles published by members of the Department during the past year. Five books were published in the Harvard Economic Studies and two volumes are in the hands of the Press to be published later this year. There has been a total of 86 books published in the Harvard Economic Studies to this date.

It should be recorded that both the Quarterly Journal of Economics under the editorship of Professor Chamberlin and the Review of Economics and Statistics have prospered during the year. Again I do feel it necessary to refer to the fact that editing the Quarterly Journal of Economics and the Review of Economics and Statistics and the carrying forward of the Harvard Economic Studies continues to raise questions regarding the adequacy of the manpower within the Department.

In the area of distinctions and honors, Professor Slichter was awarded honorary degrees (LL.D.) from the following universities: Lehigh University, Harvard University, University of Rochester, University of Wisconsin and Northwestern University. Professor

Haberler was awarded an honorary degree of Doctor of Economics (“Doktor der Wirtschaftswissenschaft honoris causa”) from Handelshochschule, St. Gallen, Switzerland. Dr. Galbraith was awarded the President’s Certificate of Merit, Medal of Merit Board, for services in Price Control and Economic Stabilization during the war.

Sincerely
[Harold H. Burbank]

Source: Harvard University Archives. Department of Economics, Correspondence and Papers (UAV 349.11), Box 2, Folder “Departmental Annual Reports to the Dean 1948-54”.

___________________________

1949-1950

[Draft] Report to Dean, October 2, 1950
Professor Burbank

In each of the reports for the last three years, emphasis has been placed upon two matters; our efforts to handle the increased numbers incident to the war, particularly on the graduate level, and our attempts to revise and improve our instruction, particularly on the undergraduate level.

With a good deal of satisfaction we are able to report that for the last year substantial progress has been made in each of these areas. Immediately after the war the number of our graduate students increased from approximately 100 to nearly 300. By raising the standards of admission and giving the most careful scrutiny to applications, the numbers on the graduate level are now well under 200, and will be reduced somewhat more for 1950-51.

The work of supervising and directing graduate students falls very unevenly upon the various members of the senior staff. Even with not over 150 graduate students some members of the staff will carry an inordinate part of individual instruction and of examining for the higher degrees. Further, large graduate classes tend to dilute the instruction.

On the undergraduate level the Department has revised its requirements for concentration, including the content of many of our key courses. This plan has been accepted by the Faculty and is now in operation. It is an ambitious scheme that involves not only a change in the content and coverage of our key courses but it also involves the strengthening the staff in these courses and an integration of course work with tutorial work. Undoubtedly it will take some years to complete this plan. Much depends upon our ability to build a strong junior staff, especially on the annual instructor level. When this reorganized instruction is in full operation it is expected that a number of courses now offered for undergraduates may be deleted.

Also it is with a good deal of satisfaction that after a period of suspension tutorial instruction has been reestablished and is developing steadily. The period of suspension was unfortunate but probably inevitable. We are now approaching a position with respect to both graduate and undergraduate instruction that at least approximates a normal situation, with a possibility of a carefully planned and well integrated system of undergraduate instruction. As a part of this plan increased attention has been given to reestablishing the General Examinations on something approximating the level of earlier years. Since we are lacking experienced tutors the establishment of tutorial instruction is a very real task but it is believed it can be done successfully.

We have been fortunate to have been able to attract to the Graduate School a group of unusually able young men. The very top of this group represents ability of the very highest order. Unfortunately only rarely can we retain the services of these young men even on the assistant professor level. However, the Department is keenly aware of the difficulties it faces in recruitment and every effort is being made to follow the progress of the product of other schools as well as the progress of our own young scholars.

Source: Harvard University Archives. Department of Economics, Correspondence and Papers (UAV 349.11), Box 2, Folder “Provost Buck—Annual Report of Dept.”

___________________________

1949-1950

January 5, 1951

Provost Paul H. Buck
5 University Hall
Cambridge, Massachusetts

Dear Provost Buck:

I am now somewhat belatedly submitting the report of the Department of Economics for 1949-50.

I. Undergraduate Instruction

Four hundred eighty-two Harvard and Radcliffe students concentrated in economics in 1949-50 as compared with 608 in the previous year. The enrolment in Economics 1 was 402 as compared with 546 in the previous year. Seventy-seven students graduated with honors; 20 obtaining magna cum laude and 57 cum laude.

The entire senior staff gave courses at the undergraduate level— a practice that distinguishes Harvard sharply from institutions such as Columbia and Chicago which restrict the activities of some of the most talented members of the staff to graduate instruction. Nevertheless, the strength of our undergraduate teaching has depended very largely on the unusually fine group of assistant professors we now have on our staff.

During the past couple of years the Department has been gradually moving toward restoration of the tutorial system and last spring it decided finally to give tutorial instruction to all honors students in their junior and senior years,

II. Graduate Instruction

Two hundred graduate students in economics were in residence last year as compared with 234 the previous year. The Department gave 58 general examinations for the Ph.D. and 47 special examinations.

The number of graduate students is still too large to handle effectively with the present staff. The students themselves justifiably complain that they cannot see enough of the members of the faculty. However if they did see as much of the faculty as they wanted to, the faculty would have little time for reading and research and the quality of instruction would decline. We are planning to deal with this problem as far as possible by making sure that more graduate students attend reasonably small seminars and do have an opportunity to get to know at least one faculty member reasonably well.

I believe that the quality of our graduate work has suffered through overemphasis on course work and preoccupation with grades. We tend to make graduate instruction too much of a prolongation of undergraduate instruction. We also tend too much in the direction of specialization and provide too little encouragement for students to become coordinated in the whole economic field. The remedy for this state of affairs depends more upon the general attitude of the Department rather than any specific measures of reorganization. We shall do whatever is possible to encourage students in the feeling that their main function here is to acquire the maturity that is essential for scholarship rather than to accumulate a collection of pieces of isolated information.

III. Research

Professors Mason, Leontief, Black, Galbraith and Dunlop are all conducting organized research projects within the Department. Apart from their substantive value, these projects give a considerable number of graduate students an opportunity to take part in organized research activity. I believe these projects have an important part to play in the future of the Department as a whole rather than as special interests of individual members. However, I do not share the view that most of our intellectual activities should be directed towards organized research. There is danger that we may become a research bureaucracy and that the merits of individual scholarship may achieve less recognition than they deserve. While the research project is invaluable in training the students in specialized activity, it does little to cultivate the maturity that should be one of the most important products of our graduate training.

IV. The Staff of the Department

Professor Schumpeter’s death has meant a loss to the Department that cannot be covered by any individual that we now have on the staff or could get from the outside. The only way to make up for his absence is for the present members of the faculty to direct part of their attention to the aspects of economic thought in which Schumpeter was particularly interested. This has in part been done. I think it is true to say that since Schumpeter’s death his own work has received more attention in Harvard classrooms than it received while he was alive.

The only new additions to the to the staff at the professorial level in 1949-50 were assistant professors Orcutt and Sawyer. Orcutt is giving a course at the graduate level and the undergraduate level on empirical economies in which he stresses the quantitative aspects of economic theory. He is also a first-class statistician. Since the resignation of Professor Crum we have had only one professional statistician in the Department, and it seems highly desirable to have at least two. Sawyer will add considerable strength to the Department’s work in economic history although he will spend half of his time in the General Education program.

VI. [sic] Distinctions

Members of the Department received the following distinctions:

Professor Edward Chamberlin — An honorary degree (Dr.) awarded by the Universita Catholica del Sacro Cuore, Milan, Italy. December 1949.

Professor Sumner Slichter — President, Industrial Relations Research Association.

Professor Gottfried Haberler — President, International Economic Association for 1950 (held by Professor Schumpeter at the time of his death).

I am attaching a bibliography of the writings of the members of the Department. [not included in this folder]

Sincerely yours,
Arthur Smithies

Source: Harvard University Archives. Department of Economics, Correspondence and Papers (UAV 349.11), Box 2, Folder “Departmental Annual Reports to the Dean 1948-54”.

Images Source: Burbank (left) from the Harvard Class Album 1946, Smithies (right) from the Harvard Class Album 1952.

Categories
Economics Programs Economists Harvard Teaching Undergraduate

Harvard. Promotion for Harold H. Burbank, Job Offer for Allyn Young 1919

This provides some back-story to the rise of Harold Hitchings Burbank in the Harvard economics department. Coincidentally, some light is cast on the salary negotiations involved in the hire of Allyn Young, as well as the hopes the department of economics held in the prospect of Young joining the economics department.

Chairman Bullock’s characterization of Burbank “He does everything willingly, but we are already in danger of driving the willing horse to death” is not exactly the language a chairman today would use today to justify a promotion for an assistant professorship…I hope.

___________________________

Harvard University
Department of Economics

F.W. Taussig
T.N. Carver
W.Z. Ripley
C.J. Bullock
E.F. Gay
W.M. Cole
O.M.W. Sprague
E.E. Day
B.M. Anderson, Jr.
J.S. Davis
H.H. Burbank
E.E. Lincoln

Cambridge, Massachusetts
12 o’clock. January 28, 1919.

Dear Mr. Lovell:

I have failed thus far to get in touch with Dr. Burbank, but will leave word at his house, and he will doubtless come to see you tomorrow.

I wish to express the hope that you will not propose any arrangement to him by which he will have to do any more work or make any more labor-consuming adjustment in connection with his work this year. He does everything willingly, but we are already in danger of driving the willing horse to death. Your suggestion that recent graduates now studying in the Law School be put in to do section work in Economics A. involves, even tho these new men are placed in charge of sections which began work in September, an amount of labor, responsibility, and worry on Burbank’s part which I feel strongly It would be unfair to ask of him.

I have not myself been one of the real sufferers from the war, so far as University work is concerned. Such extra work as I have had to do for the men in Washington has been comparatively limited in amount, and some of my ordinary work has been decreased so that I have not suffered greatly. But the younger men who have stood by us have had a bad time, and I feel so keenly that it is unjust not to give them relief as soon as we can do it that I hate to think of Burbank’s being asked to make any further readjustments in Economics A.

You will recall, if you will review the last two years, that I have not found difficulties in the way of doing the things which it was necessary to ask the Department to do, and have been ready to disorganize, or readjust and adapt, to any necessary extent. I have further found the ways of doing this; and only last fall, in spite of the fact that I felt it was hardly right for Day to be taken from us, I went to a deal of trouble to fix up an arrangement under which he might be released. If I saw any arrangement now, I would surely make it, as I have done in the past. If Burbank can think of any arrangement that I have not been able to think of, I shall be glad to have it put into effect; but I wish to represent to you that it will not only be bad for the course, but very unfair for Burbank to ask him to take young and inexperienced instructors whose heart is in the Law School work anyway, and fit them into section work in Economics A at this time. Moreover, this arrangement involves delay of at least ten days or a fortnight, and our men need relief at the earliest moment. There are certainly no suitable men in the Law School now; and if any register next week, it will take time to find them out, to make arrangement, and to have them get up their work so that they are fit to take charge of a section. should think that under this plan it would be more rather than less than a fortnight before our men would get any relief. If you could know from actual contact with conditions what I have been compelled to know about the work of our young men during the war, I believe you would feel as strongly as I do that what they need now is immediate relief and not a plan by which they will have to spend the next month breaking in green, and possibly inefficient, substitutes. By the time that Burbank gets Economics A running smoothly again, if, indeed, that can be done at all, the term will be most over and the acute need of relief will be almost at an end.

Sincerely yours,
[Signed] Charles J. Bullock

President A. Lawrence Lowell

___________________________

Harvard University
Department of Economics

F.W. Taussig
T.N. Carver
C.J. Bullock
E.F. Gay
W.M. Cole
O.M.W. Sprague
E.E. Day
J.S. Davis
H.H. Burbank
E.E. Lincoln

Cambridge, Massachusetts
March 8, 1919.

My dear Mr. Lovell:

Dr Burbank informs me that he has received from Dartmouth College the offer of a full professorship, and this makes it necessary for the Corporation to consider whether it desires to retain him at Harvard. You will recall that two years ago the Department of Economics recommended that Burbank be advanced to an assistant professorship. This was at the time when he received from Chicago University the offer of an assistant professorship with full charge of their instruction in Public Finance. A year ago I brought the matter to your attention, but you desired to postpone action until Burbank’s book had been published. Last June I asked whether you would be willing to waive the question of publication of Burbank’s book, which was nearly, but not quite, completed. in order that he might accept employment from a committee of the American Economic Association, which would both be remunerative and give him an unusual opportunity to investigate a subject in which he is greatly interested, namely, the practical operation of the Federal income and excess profits taxes. You sent me word through Mr. Pierce that you would waive the requirement, and that you would be glad to have Mr. Burbank accept this employment.

Mr. Burbank made a distinct success of his work for the Economic Association, and such success as the Committee achieved was largely due to him. This year he has been conducting Economics A, and has demonstrated his ability to handle that course in a satisfactory manner. It seems to me that he is an invaluable man for the Department, and I hope that the Corporation will be able nor to advance him to an assistant professorship.

You also asked me this morning to write you concerning Allyn A. Young, whom we have had under consideration for a number of years.

In the winter of 1916-17 the full professors of the Department of Economics, after carefully looking over the field, recommended to you that Mr. Young be called to a full professorship at Harvard University.

You authorized me to write to Young and inquire whether he could be secured, and if so, at what salary; and I was able to report to you that Young would come to Harvard if he were offered a full professorship at a salary of $4500. At this juncture the United States entered the war, and the matter was necessarily dropped.

Last December Professors Gay and Haskins called my attention to the fact that Young was likely to receive an offer from Columbia University, and I held a hurried conference with them, and they later conferred with you. Action was postponed, inasmuch as Mr. Young was going to the Peace Conference as exert on economic resources; and it appeared probable that, if we could offer him a professorship at $5000, we could secure him for Harvard, even tho another offer developed elsewhere.

I hope that the Corporation will feel able to extend a call to Professor Young at this time. Since I talked with you this morning, I have met Professors Carver and Ripley, and they both concur in the recommendation which I make. Professor Gay gave you his opinion in December; and since that time I have heard from Taussig, who still is of the opinion that we ought to call Young.

I have no further knowledge as to the amount of salary that it would be necessary to offer. I assume that we should have to offer at least $4500, which was the figure that would have been necessary in 1916; and in view of Young’s increased experience and enhanced reputation, I should think that a salary of $5000 would be justified.

It is, I believe, important for the Department to secure Young at this time. We had in 1917 a Department of Economics which was recognized as one of the strongest in the country; but we needed Young at that time, and shall need him still more now in order to develop our work during the next decade. With him, I believe we should have a department that would be recognized as very clearly the strongest department in the country.

There is one further consideration to be taken into account in connection with extending a call to Young. If our economic research enterprise proves permanent, Young would be absolutely the best man in the country to coöperate with Professor Persons in carrying through the work we have undertaken. With Young and Persons in the economic research undertaking, we should have almost a monopoly of high class statistical brains. Young’s appointment was recommended by the Department in the winter of 1916-17, before the Committee on Economic Research was established, and without any reference to the development of that Committee’s work. The Department recommended him because they thought he was the one man whom the Department needed. The point I am now making is that Young is the one man whom our economic research undertaking needs, so that it seems upon every account desirable to add him to our staff next fall. Under the arrangement that I have in mind, if our economic research enterprise proves permanent, Professor Persons could give two-thirds of his time to the Committee on Economic Research and one-third to teaching, and Professor Young could give two-thirds of his time to teaching and one-third to the Committee on Economic Research. By this arrangement the Department of Economics would gain two teachers of the very highest reputation at an expense amounting only to the salary of one full professor, while the Committee on Economic Research would secure the services of the two minds in the country which are best adapted for the immediate work it has in hand.

Sincerely yours,
[signed] Charles J. Bullock

President A. Lawrence Lowell

___________________________

Carbon Copy of Letter from President Lowell to Professor Bullock

March 8, 1919

Dear Mr Bullock:

I understand that Mr Burbank is feeling uneasy about his promotion, and has been made valuable offers from elsewhere. Mr Pierce, at my request, wrote you last May that the completion of his book was not essential to his promotion to an assistant professorship. He is as near as possible the soul of the body of tutors; and I think it is important that we should make it clear that good work as a tutor will receive as much recognition as an equally good conduct of lecture courses. Would it not be well, therefore, if Mr Burbank were appointed an assistant professor now? There is a Corporation meeting on Monday, and I should be very glad if you could communicate with me before it takes place, if you come home in time.

Very truly yours,
[stamp] A. Lawrence Lowell

Professor Charles J. Bullock
6 Channing Street
Cambridge, Mass.

Source: Harvard University Archives. President Lowell’s Papers 1917-1919. Box 124. Folder 1689.

Categories
Economics Programs Harvard Undergraduate

Harvard. Economics Department Committee Assignments, 1972-73

 

“The Division of Administration is limited by the Extent of the Department” might have been a chapter title in a history of economics written by Adam Smith were he to have lived two centuries after the publication of the Wealth of Nations. I transcribed and now post the following list of administrative committees/tasks and the names of members of the Harvard economics department belonging to each during the 1972-73 academic year.  The list comes from the economic historian Alexander Gerschenkron’s papers at the Harvard Archives. I have paired this artifact with the printed list of economics faculty teaching economics courses in 1972-73 from the annual Harvard course catalogue.

_____________________________

Committee Assignments for 1972-73

  1. Undergraduate Instruction

Otto Eckstein, Chairman
Elisabeth Allison
Abram Bergson
James Duesenberry
Jerry Green
William Raduchel
Martin Spechler
Francois Wilkinson

  1. Examing Committee

Benjamin Friedman, Chairman
Richard Caves
Abram Bergson
Robert Dorfman
Jerry Greene
Martin Spechler

Graduate Instruction

Robert Dorfman, Chairman
Truman Bewley
Hendrik Houthakker
Gregory Ingram
Richard Musgrave
Thomas Schelling
Gail Pierson

Mathematics Examination

Truman Bewley
David Starrett

Theory Examination

Arthur Smithies, Chairman
Zvi Griliches
Glenn Jenkins
John Lintner
Stephen Marglin
Janet Yellen
Tsuneo Ishikawa (Spring)

Quantitative Methods Examination

Lance Taylor, Chairman
Edward Leamer
William Raduchel
Howard Raiffa
T. N. Tideman
Thomas Horst (Spring)

Economic History Examination

Alexander Gerschenkron
Paul David

Ph.D. in Business Economics

John Lintner

Fellowships and Admissions

Zvi Griliches, Chairman
Elisabeth Allison
Hendrik Houthakker
T. N. Tidema
Marcelo Selowsky

Less Developed Countries Recruitment

Richard Mallon

Recruitment of Black Students

T. N. Tideman

  1. Non-Tenure Personnel

John Kain, Chairman
Richard Caves
James Duesenberry
Harvey Leibenstein
Marc Roberts
David Starrett
Janet Yellen

Placement Officer

Dwight Perkins

  1. Publications

Harvard Economic Studies

Richard Caves, Chairman
Edward Leamer

Quarterly Journal of Economics

Richard Musgrave, Editor

Review of Economics & Statistics

Hendrik Houthakker, Editor

  1. Wells Prize

Harvey Leibenstein, Chair
Alexander Geschenkron
Jerry Green

  1. Political Economy Lectures

Wassily Leontief, Chairman
Stephen Marglin

  1. Schumpeter Prize

Arthur Smithies, Chairman
Wassily Leontief

  1. Goldsmith Prize

Gregory Ingram, Chairman
Richard Musgrave

  1. Department Minutes

Martin Spechler

  1. Harvard Computing Center

William Raduchel

  1. Universities-National Bureau Committee

John Lintner

  1. Administrative Committee, Harvard Institute for Economic Research

William Raduchel, Acting Chairman
James Duesenberry
John T. Dunlop
Zvi Griliches
Dale Jorgenson (fall)
Dwight Perkins

Source: Harvard University Archives. Papers of Alexander Gerschenkron, Box 3, Folder “Economics (General) 1973/74, 1 of 2”.

_____________________________

1972-73
Faculty of the Department of Economics

James S. Duesenberry, William Joseph Maier Professor of Money and Banking (Chairman)

Elisabeth S. Allison, Assistant Professor of Economics

Kenneth J. Arrow, Professor of Economics (on leave 1972-73)

Abram Bergson, George F. Baker Professor of Economics

Truman F. Bewley, Assistant Professor of Economics and of Mathematics

Samuel S. Bowles, Associate Professor of Economics (on leave 1972-73)

Richard E. Caves, Professor of Economics

Hollis B. Chenery, Lecturer on Economics

David C. Cole, Lecturer on Economics

Paul A. David, Visiting Professor of Economics (Stanford University)

Kenneth M. Deitch, Assistant Professor of Economics (on leave 1972-73)

Robert Dorfman, David A. Wells Professor of Economics

John T. Dunlop, Lamont University Professor, Dean of the Faculty of Arts and Sciences

Otto Eckstein, Professor of Economics

Martin S. Feldstein, Professor of Economics (on leave 1972-73)

Benjamin M. Friedman, Assistant Professor of Economics

John Kenneth Galbraith, Paul M. Warburg Professor of Economics (on leave spring term)

James D. Gavan, Lecturer on Population Sciences (Public Health) Lecturer on Economics (spring term only)

Alexander Gerschenkron, Walter S. Barker Professor of Economics

Richard T. Gill, Lecturer on Economics

Carl H. Gotsch, Lecturer on Economics

Jerry Green, Assistant Professor of Economics

Zvi Griliches, Professor of Economics

Albert O. Hirschman, Lucius N. Littauer Professor of Political Economy (on leave 1972-73)

Thomas Horst, Assistant Professor of Economics (on leave fall term)

Hendrik S. Houthakker, Professor of Economics

Gregory Ingram, Assistant Professor of Economics

Dale W. Jorgenson, Professor of Economics (on leave spring term)

John F. Kain, Professor of Economics

Leonard Kopelman, Lecturer on Economics

Edward Leamer, Assistant Professor of Economics

Harvey Leibenstein, Andelot Professor of Economics and Population

Wassily W. Leontief, Henry Lee Professor of Economics

John Lintner, George Gund Professor of Economics and Business Administration (Business)

Arthur MacEwan, Assistant Professor of Economics (on leave 1972-73)

George F. Mair, Visiting Professor of Population Economics (Smith College)

Richard D. Mallon, Lecturer on Economics

Stephen A. Marglin, Professor of Economics

Albert J. Meyer, Professor of Middle Eastern Studies, Lecturer on Economics

Richard A. Musgrave, Harold Hitchings Burbank Professor of Political Economy; Professor of Economics (Law)

Gustav Papanek, Lecturer on Economics (on leave 1972-73)

Dwight H. Perkins, Professor of Modern China Studies and of Economics

Gail Pierson, Assistant Professor of Economics

William J. Raduchel, Assistant Professor of Economics

Howard Raiffa, Frank Plumpton Ramsey Professor of Managerial Economics (Business)

Marc J. Roberts, Associate Professor of Economics

Sherwin Rosen, Visiting Professor of Economics (University of Rochester)

Henry Rosovsky, Frank W. Taussig Research Professor of Economics

Michael Rothschild, Assistant Professor of Economics (on leave 1972-73)

Thomas C. Schelling, Professor of Economics

Marcelo Selowsky, Assistant Professor of Economics

Arthur Smithies, Nathaniel Ropes Professor of Political Economy

Martin C. Spechler, Lecturer on Economics and Social Studies

David A. Starrett, Associate Professor of Economics

Joseph J. Stern, Lecturer on Economics

Carl M. Stevens, Visiting Professor of Economics (Reed College)

Lance Taylor, Assistant Professor of Economics

T. Nicolaus Tideman, Assistant Professor of Economics (on leave fall term)

Thomas A. Wilson, Visiting Professor of Canadian Studies (University of Toronto)

Janet Yellen, Assistant Professor of Economics

Other Faculty Offering Instruction in the Department of Economics

Francis M. Bator, Professor of Political Economy (Kennedy School) (Public Policy, West European Studies)

Ralph E. Berry, Jr., Associate Professor of Economics (Public Health)

William M. Capron, Lecturer on Political Economy (Kennedy School) (Political Economy and Government)

Peter B. Doeringer, Associate Professor of Political Economy (Kennedy School)

Rashi Fein, Professor of Economics of Medicine at the Center for Community Health and Medical Care (Medicine)

Sherwood Frey, Assistant Professor of Business Administration (Business)

Herbert M. Gintis, Lecturer on Education (Education)

Joseph J. Harrington, Associate Professor of Environmental Health Engineering (Public Health)

Henry D. Jacoby, Associate Professor of Political Economy (Kennedy School)

James R. Kurth, Associate Professor of Government (Government)

Walter J. McCann, Jr., Associate Professor of Education (Education)

Harold A. Thomas, Jr., Gordon McKay Professor of Civil and Sanitary Engineering (Engineering and Applied Physics)

Richard J. Zeckhauser, Professor of Political Economy (Kennedy School)

Source: Harvard University, Faculty of Arts and Sciences. Courses of Instruction for Harvard & Radcliffe, 1972-73, pp. 183-185.

Image Source: Littauer Center (July 1970). Harvard University Archives.

Categories
Economics Programs Harvard

Harvard. Visiting Committee Reports. 1895, 1901, 1903, 1906, 1908, 1914

Around the turn of the 20th century visiting committees to the Harvard economic department were supposed to submit reports to the Board of Overseers of Harvard College at least once every three years. When I first saw that at archive.org there were six items listed dealing with Harvard’s visiting committees I was excited, hoping to find a rich mine of material as extensive as Arthur F. Brimmer’s 1974 Report of Economics Department Visiting Committee. As you can see from below, quite the opposite is the case. The early Visiting Committees submitted the nano-reports transcribed and posted below.

Perhaps one indication of the purely formal nature of the reporting system is that the “department of political economy” was renamed the “department of economics” in 1892 but the visiting committee was still referred to the visiting committee “on political economy” as late as 1912-1913.

__________________________________

LII.

REPORT OF THE COMMITTEE TO VISIT THE DEPARTMENT OF POLITICAL ECONOMY.
May 22, 1895

To the Board of Overseers of Harvard College:

ln the department of Political Economy the Professors feel the pressure of increased numbers of students, the lack of satisfactory space for lecture rooms, and the lack of time for independent work and research.

The difficulty is most serious in Economics 1. The lecture room available is ill-fitted for speaking or hearing, and the great number of students make it impossible, with the present corps of instructors, to divide them into sections small enough for adequate teaching and discussion. The difficulty has been met, as far as practicable, by strenuous and continuous work on the part of the Professors and instructors. This department, like others, feels the want of more books for reference, and of more library space for books and reading.

ARTHUR T. LYMAN, Chairman

Source: Harvard University. Reports of the Visiting Committees of the Board of Overseers of Harvard College from February 6, 1890 to January 8, 1902. (Cambridge, Massachusetts, 1902), p. 299.

__________________________________

XCIX.

REPORT OF THE COMMITTEE ON POLITICAL ECONOMY.
June 5, 1901.

To the Board of Overseers of Harvard College:––

The Committee on Political Economy has had this year and last year long conferences with the Professors in the Department, and members of the Committee have also corresponded with the head of the Department.

A very large proportion of the students take Political Economy, and many take the more advanced courses. The courses seem to be satisfactorily conducted and the lecture system is supplemented by conferences and teaching in sections.

A course in Accounting has been taken by a considerable number of students, and next year an excellent programme is proposed for a course in business law. It is intended that such special courses should be confined to graduates or to seniors, and it seems best that they should not interfere with the general and broader courses of instruction in this or any other department.

The great loss to the University and to the Department of Political Economy from the death of Professor Charles F. Dunbar has been referred to by the President in his report.

For the Committee,

ARTHUR T. LYMAN,
Chairman

Source: Harvard University. Reports of the Visiting Committees of the Board of Overseers of Harvard College from February 6, 1890 to January 8, 1902. (Cambridge, Massachusetts, 1902), p. 621.

__________________________________

CXXXIV.

REPORT OF THE COMMITTEE ON POLITICAL ECONOMY.
September 30, 1903.

To the Board of Overseers of Harvard College:—

The Committee on Political Economy met the professors and instructors.

Professor Taussig, who has been absent for a long time on account of illness, will resume a part of his work in October.

There continues to be a very large attendance in the courses of Economics — 519 in Economics 1, which deals with the general outlines of the subject, and one hundred or more in four other courses on banking, economic history of the United States, problems of labor and industrial organization, and currency legislation with recent experience and theory. These large classes involve the usual difficulties in giving interesting and adequate instruction, but they are gratifying evidences of the interest in this important subject. They make necessary many instructors for proper division of large classes, and consequently large expense.

The list of courses in the Catalogue shows the wide range of the instruction.

The Quarterly Journal of Economics, published by the Department, is well worthy of your attention. In recent numbers Professor Taussig has given an interesting and instructive history of the iron industry in this country, and the sugar problem has been fully treated.

Many other articles on important practical and theoretical questions have appeared in this Journal, which, like all publications of this class, needs the increased subscriptions and financial support to which its merits fully entitle it.

For the Committee,

ARTHUR T. LYMAN,
Chairman

Source: Harvard University. Reports of the Visiting Committees of the Board of Overseers of Harvard College from January 8, 1902 to July 30, 1909. (Cambridge, Massachusetts, 1909), p. 741.

Visiting Committee Members (1903):

Arthur T. Lyman, Charles S. Fairchild, Horace E. Deming, John E. Thayer, John F. Moors.

Source: Harvard University. Reports of the Visiting Committees of the Board of Overseers of Harvard College from January 8, 1902 to July 30, 1909. (Cambridge, Massachusetts, 1909),  following p. 708.

__________________________________

CLXXVIII.

REPORT OF THE COMMITTEE ON POLITICAL ECONOMY.
May 9, 1906.

To the Board of Overseers of Harvard College:—

The Committee on Political Economy met the Professors in the Department at a long conference. They are able, accomplished, and interesting, and there is, as usual, a very large attendance on their various courses of instruction. These courses deal not only with economic theory, but with transportation, finance, taxation, currency legislation, and banking, economic history, the distribution of wealth, labor, socialism, commercial law, and accounting.

The course on the principles of accounting is taken chiefly by Graduates and Seniors, and it is the purpose of the Department not to sacrifice to it the general and fundamental courses, but to confine it mainly to Seniors or Graduates rather than to follow the plan of some of the Western State Universities in which courses are given throughout the college terms treating in an elaborate way the practical details of business in various branches.

The large attendance in several of the courses in Economics requires subdivisions and conferences, and instructors for such divisions. There is naturally considerable difficulty in securing the desired or desirable instructors, owing to the small salaries that can be afforded and the consequent lack of permanency of employment, and it is much desired that the needed competency and permanency should, if possible, be secured by the assignment of some fellowships to the best of the instructors in the subdivisions and conferences. Much of the time of the Professors might be saved by the addition of various apparatus needed. Some help in this matter has been given by members of the Committee and others.

For the Committee,

ARTHUR T. LYMAN,
Chairman

Source: Harvard University. Reports of the Visiting Committees of the Board of Overseers of Harvard College from January 8, 1902 to July 30, 1909. (Cambridge, Massachusetts, 1909), p. 925.

Visiting Committee Members (1906):

Arthur T. Lyman, Charles S. Fairchild, Horace E. Deming, John F. Moors, J. Wells Farley.

Source: Harvard University. Reports of the Visiting Committees of the Board of Overseers of Harvard College from January 8, 1902 to July 30, 1909. (Cambridge, Massachusetts, 1909), after p. 843.

__________________________________

CCVIII.

REPORT OF THE COMMITTEE ON POLITICAL ECONOMY.
April 8, 1908.

To the Board of Overseers of Harvard College:—

The Committee on Political Economy met the Professors of the Department, and considers that the conditions are satisfactory.

There seems to be nothing new to report, except that arrangements have become possible through the generosity of various persons for carrying on for five years advanced business courses, such as have been under consideration for several years past.

More space is needed for conferences and books, and some plans are under consideration for relief in these matters.

For the Committee,

ARTHUR T. LYMAN,
Chairman

Source: Harvard University. Reports of the Visiting Committees of the Board of Overseers of Harvard College from January 8, 1902 to July 30, 1909. (Cambridge, Massachusetts, 1909), p. 1128.

Visiting Committee Members (1908):

Arthur T. Lyman, Frederic A. Delano, John F. Moors, Charles S. Fairchild, William Endicott, Jr., J. Wells Farley, Charles G. Washburn.

Source: Harvard University. Reports of the Visiting Committees of the Board of Overseers of Harvard College from January 8, 1902 to July 30, 1909. (Cambridge, Massachusetts, 1909), after p. 1118.

__________________________________

EXECUTIVE COMMITTEE.
REPORT OF THE SECRETARY FOR THE ACADEMIC YEAR 1913-14.
April 13, 1914.

5. Economics. — The enrolment of students in Economics for the first, half of the academic year shows a falling off from 1634 to 1520 students, the most significant loss taking place in the course on Banking. The amount of instruction offered at Harvard in Economics exceeds that offered at any other University except Michigan, while in the amount of money expended Harvard is far in the lead. The Committee notes the continuing eminence and prosperity of the Quarterly Journal of Economics, published by the Department since 1886, which maintains its place as one of the foremost periodicals in the subject in the English Language, and perhaps the foremost. It has the largest circulation of any periodical published by the University, except the Law Review, and there is no more certain way of making a reputation as a writer on Economic subjects than to publish an article in this journal. The most important need of the Department is a fund for the endowment of Economic Research, which will make possible the scientific investigation of pressing economic questions. A very interesting supplemental report is made by Mr. John Wells Morss, one of the members of the Committee, representing the views and experience of a man of business, rather than of a student of education. It should be read as a whole, as it is difficult to give a satisfactory impression of its interest and value by quotations or a summary.

Source: Appended to: Harvard University. Reports of the Visiting Committees of the Board of Overseers of Harvard College for the Academic Year, 1914-15. (Cambridge, Massachusetts, 1915), three pages after page 260.

Image Source: John Harvard statue, ca. 1904. U.S. Library of Congress Prints and Photographs Division.

Categories
Economics Programs Exam Questions Harvard

Harvard. Mid-year and Year-End Final Exams in Economics and Social Ethics, 1896-1897

 

The collection of transcribed Harvard semester examinations here at Economics in the Rear-view Mirror is growing slowly. This post adds the exam questions from 1896-1897 for all the economics courses and for Francis Peabody’s philosophy course “The Ethics of the Social Questions”. 

_____________________

From the Preface to the Announcements
for 1896-97

The courses primarily for Undergraduates are open (and in some cases recommended) to Graduate Students, but are not ordinarily counted towards any of the higher degrees. The courses for Graduates and Undergraduates are, under certain limitations, open to any properly qualified student. To the courses primarily for Graduates Undergraduates are admitted only on the recommendation of the Instructor.

No starred (*) course and no course of research can be taken without the previous consent of the Instructor….

By recent action of the Governing Boards, the requirement of two years of residence at this University of a candidate for the degree of Doctor of Philosophy or Doctor of Science has been rescinded. The minimum requirement of residence in now one year, this period being fixed for all degrees by the Statutes. It is not the purpose of the change thus made to lower the standards for these two degrees; but only to reduce the amount of compulsory residence at this University. (April, 1896).

Source: Harvard University. Announcement of the Courses of Instruction Provided by the Faculty of Arts and Sciences for the Academic Year 1896-1897, pp. iii-iv.

_____________________

Philosophy 5.
The Ethics of the Social Questions.
1896-1897

Course Announcement

[Philosophy] 5. The Ethics of the Social Questions. — The problems of Poor-Relief, the Family, Temperance, and various phases of the Labor Question in the light of ethical theory. — Lectures, special researches, and required reading. Tu., Th., Sat., at 10. Professor Peabody.

Source: Harvard University. Announcement of the Courses of Instruction Provided by the Faculty of Arts and Sciences for the Academic Year 1896-1897, p. 26.

Course Enrollment

[Philosophy] 5. Professor Peabody. — The Ethics of the Social Questions. — The problems of Poor-Relief, the Family, Temperance, and various phases of the Labor Question in the light of ethical theory. — Lectures, special researches, and required reading. 3 hours.

Total 56: 1 Graduate, 31 Seniors, 10 Juniors, 3 Sophomores, 11 Others.

Source: Harvard University. Report of the President of Harvard College 1896-1897, p. 66.

*  *  *  *  *
[Mid-year examination,
still to be added]

*  *  *  *  *

Philosophy 5.
THE ETHICS OF THE SOCIAL QUESTIONS.
Year-End Examination,
1896-97

[This paper should be considered as a whole. The time should not be exhausted in answering a few questions, but such limit should be given to each answer as will permit the answering of all the questions in the time assigned.]

  1. Indicate, briefly, the place in the history of the modern Labor Question of:
    Chalmers;
    Von Ketteler;
    Lassalle;
    The Rochdale Pioneers;
    Carlyle;
  2. Ruskin as an Economist.
  3. The anarchist’s criticism of the socialist; the socialist’s criticism of the anarchist, and the communist as he is criticised by both.
  4. What do you understand to be the “quintessence” of socialism, as expounded by Schäffle; and what criticisms on this whole social programme appear to you most serious?
  5. Arbitration and conciliation — their differences, varieties, advantages, and limitations.
  6. The history of co-operation in Great Britain, its fortunes in the United States, and the conditions of its success.
  7. Various types of industrial partnership, — their special advantages and limitations.
  8. The Scandinavian Licensing System compared with the present Massachusetts Liquor Law. (Fanshawe, 187-229.)
  9. Sum up, briefly, the general doctrine of social duty which our study of various social questions is intended to illustrate.

Source: Harvard University Archives. Harvard University Examination Papers, 1873-1915. Box 4. Bound volume: Examination Papers 1896-97. Papers Set for Final Examinations in Philosophy, History, Government, Economics, Fine Arts, Architecture, and Music in Harvard College, June 1897, p. 7.

_____________________

ECONOMICS.
Primarily for Undergraduates.

_____________________

Economics 1.
Outlines of Economics,
1896-97

Course Announcement

[Economics] 1. Outlines of Economics. — Principles of Political Economy. — Lectures on Economic Development, Social Questions, and Financial Legislation. Mon., Wed., Fri., at 9. Professors Taussig and Ashley, Asst. Professor Edward Cummings, and Dr. John Cummings.

Source: Harvard University. Announcement of the Courses of Instruction Provided by the Faculty of Arts and Sciences for the Academic Year 1896-1897, p. 33.

Course Enrollment

[Economics] 1. Professors Taussig and Ashley, Asst. Professor Edward Cummings, and Dr. John Cummings. Outlines of Economics. — Principles of Political Economy. — Lectures on Economic Development, Social Questions, and Financial Legislation. 3 hours.

Total 464: 1 Graduate, 40 Seniors, 131 Juniors, 235 Sophomores, 12 Freshmen, 45 Others.

Source: Harvard University. Report of the President of Harvard College 1896-1897, p. 65.

Economics 1.
Mid-year Examination,
1896-97

  1. “Productive labor is that which produces utilities fixed and
  2. embodied in material objects. All other labor, however useful, is classed as unproductive.” Why? or why not?
  3. Capital is the result of saving: capital is produced by labor; all capital is consumed. Can you reconcile these propositions?
  4. “Those circumstances of a country, in which population can with impunity increase at its utmost rate, are rare and transitory.” What are they? and why rare? What is the utmost rate?
  5. Give examples of (a) differences in wages arising from different degrees of attractiveness in different employments; (b) differences arising from natural monopolies.
  6. Suppose a tax were imposed on land precisely equal to the economic rent paid for its use; could the owner of the land shift the tax to his tenant by charging a higher rent than before?
  7. What is meant when it is said that rent does not enter into the cost of production?
  8. It is said to be immaterial whether a community has a large or a small stock of money; and it is said to be harmful for a community to resort to inconvertible paper. Can both of these propositions be sound?
  9. “With enormous shortsightedness, the people of the United States send abroad every year over one hundred millions of dollars, with which to pay for sugar which might have been produced at home.” Why, or why not, is there shortsightedness in this operation?
  10. Wherein does a country gain, if other countries demand more of its exports?
  11. In the stationary state. as described by Mill, what determines the rate of interest? the rate of wages?

Source: Harvard University Archives. [Examinations] Scrapbook of F. W. Taussig, p. 58.

Economics 1.
Year-end Examination,
1896-97

[Answer nine questions, selecting at least one from each of the four groups. Arrange your answers strictly in the order of the questions selected. Give your reasons in all cases.]

I.

  1. Mention a case in which the income received for the use of a piece of real estate is to be regarded as rent; one in which it is to be regarded as interest; and one in which the classification would be doubtful.
  2. Mention a commodity whose value is permanently governed by cost of production; one whose value is permanently governed by the equation of demand and supply; and one whose value is permanently affected by both causes.
  3. Does the gain from foreign trade arise from the sale of exports? from the purchase of imports?

II.

  1. Is the law of Rent stated by Mill applicable either to the German peasant of the early part of the century or to the American farmer of to-day? Give your reasons.
  2. What are the functions of the entrepreneur? Give some account of the origin of the entrepreneur
  3. Illustrate the influence of the feeling of nationality as an economic factor.

III.

  1. How far have English trade unions tried to control (a) the wages of labor? (b) the supply of labor? In what respects have they been most useful?
  2. What do you understand by the rise of the modern factory system? In what sense is this system responsible for what is known as the labor problem?
  3. If all the productive and distributive business of a community were in the hands of cooperative societies would the labor question cease to exist?

IV.

  1. Point out wherein the deposits of a bank resemble its notes, and wherein they differ from its notes; and consider why one or the other should be regarded as part of the circulating medium of the community.
  2. What was the amount outstanding (in round numbers) of legal-tender notes in 1867? in 1877? in 1897? What were the laws under which the notes outstanding at those several dates had been issued, and what was the state of the legislation then in force for their redemption?
  3. Wherein does the Reichsbank of Germany, as to its management of notes and deposits, resemble the Bank of France? Wherein the Bank of England?

Source: Harvard University Archives. Harvard University Examination Papers, 1873-1915. Box 4. Bound volume: Examination Papers 1896-97. Papers Set for Final Examinations in Philosophy, History, Government, Economics, Fine Arts, Architecture, and Music in Harvard College, June 1897, pp. 37-38.

_____________________

ECONOMICS
For Graduates and Undergraduates.

The Courses for Graduates and Undergraduates are open to students who have passed satisfactorily in Course 1. Courses 5, 6, 7, 8, and 9 are also open to Juniors and Seniors of good rank who take Course 1 at the same time; and Course 11 is open to students who have taken either Economics 1 or History 1.

_____________________

Economics 10.
The Mediaeval Economic History of Europe

[[Economics] *10. The Mediaeval Economic History of Europe. Tu., Th., (and at the pleasure of the instructor) Sat., at 12. Professor Ashley.]

Omitted in 1896-97. Courses 10 and 11 are usually given in alternate years.

Source: Harvard University. Announcement of the Courses of Instruction Provided by the Faculty of Arts and Sciences for the Academic Year 1896-1897, p. 33.

_____________________

Economics 11.
The Modern Economic History of Europe and America (from 1600),
1896-97

Course Announcement

[Economics] 11. The Modern Economic History of Europe and America (from 1600). Tu., Th., (and at the pleasure of the instructor) Sat., at 12. Professor Ashley.

Courses 10 and 11 are usually given in alternate years.

Source: Harvard University. Announcement of the Courses of Instruction Provided by the Faculty of Arts and Sciences for the Academic Year 1896-1897, p. 33.

Course Enrollment

[Economics] 11. Professor Ashley. The Modern Economic History of Europe and America (from 1600). 2 hours.

Total 29: 16 Graduates, 6 Seniors, 1 Junior, 1 Sophomore, 5 Others.

Source: Harvard University. Report of the President of Harvard College 1896-1897, p. 65.

Economics 11.
Mid-Year Examination,
1896-97

N. B. — Not more than eight questions must be attempted.

  1. Give some account of Asiatic commerce in the middle ages.
  2. Enumerate very briefly some of the reasons for which Genoa deserves attention in economic history.
  3. “The mooste part of the lordes have enclosed their demeyn lands and meadows and kept them in severalties.” (Fitzherbert, c. 1530). Explain the nature and effects of the action here described.
  4. What features, if any, were common to the Peasant Risings in the different countries of Western Europe in the fourteenth, fifteenth, and sixteenth centuries?
  5. Compare the action of the English government in relation to industry in the sixteenth century with that of the French government.
  6. “Our merchants may do well to provide for the Russians such wares as the Dutch nation doth serve them of.” Explain and comment.
  7. Describe the industrial condition of Norfolk in the time of Defoe.
  8. Explain, with illustrations, what is meant by the “territorial” period in German economic development.
  9. What impressions do you derive from Defoe’s Essay upon Projects as to the constitution, temper and interests of the business circles of London in his time?
  10. Explain the following terms Droit de vaine pâture, Société en commandite, Niederlegung von Hufen, Hausindustrie, Fondaco.
  11. Give a critical account of any really important work (not on the printed list) of which you have read any considerable portion in connection with this course.

Source: Harvard University Archives. Harvard University Mid-Year Examination, 1852-1943. Box 4. Bound volume: Examination Papers, Mid-Years 1896-97.

Economics 11.
Year-end Examination,
1896-97

N.B. – Not more than eight questions must be attempted.

  1. “The fifteenth century was the golden age of the English labourer.” What is the evidence for that contention? How is that evidence to be interpreted?
  2. Explain the part played by Hamburg in the economic history of the sixteenth and seventeenth centuries.
  3. Give some account of the history, since the massacre of Amboyna, of the Dutch East Indian Empire.
  4. Indicate very briefly the chief points in Schmoller’s account of mercantilist policy. In what directions does it seem open to criticism?
  5. State the causes and criticize the alleged consequences of the drain of specie from the English colonies in the eighteenth century.
    [Not to be taken by those who have written theses on the Navigation Act.]
  6. “English industries could not have advanced so rapidly without protection.” Examine this statement.
  7. Describe the main features of English industrial life in the early part of the eighteenth century.
  8. What changes did the French Revolution make in the position of the rural population of France.
  9. Discuss the application of the Infant Industries argument to the United States during the early decades of the present century.
  10. Give some account of the competition between railroads and canals as means of transportation.
  11. What exactly were the English “Corn Laws,” repealed in 1846? Have the anticipations of Cobden been realized?
  12. “You made me look rather a fool, Arminius,” I began, “by what you primed me with in Germany last year about Stein settling your land question.” “I dare say you looked a fool,” says my Prussian boor, “but what did I tell you?” “Why,” says I, “you told me Stein had settled a land question like the Irish land question, and I said so in the Cornhill Magazine, and now the matter has come up again by Mr. Bright talking at Dublin of what Stein did, and it turns out he settled nothing like the Irish land question at all, but only a sort of title-commutation affair.” “Who says that?” asked Arminius. “A very able writer in the Times,” I replied. — May we have your opinion?

Source: Harvard University Archives. Harvard University Examination Papers, 1873-1915. Box 4. Bound volume: Examination Papers 1896-97. Papers Set for Final Examinations in Philosophy, History, Government, Economics, Fine Arts, Architecture, and Music in Harvard College, June 1897, p. 46.

_____________________

Economics 15.
The History and Literature of Economics to the Middle of the Nineteenth Century,
1896-97

Course Announcement

[Economics] *15. The History and Literature of Economics to the Middle of the Nineteenth Century. Mon., Wed., (and at the pleasure of the instructor) Fri., at 12. Professor Ashley.

Source: Harvard University. Announcement of the Courses of Instruction Provided by the Faculty of Arts and Sciences for the Academic Year 1896-1897, p. 34.

Course Enrollment

[Economics] 15. Professor Ashley. — The History and Literature of Economics to the Middle of the Nineteenth Century. 2 hours.

Total 14: 10 Graduates, 2 Seniors, 2 Juniors.

Source: Harvard University. Report of the President of Harvard College 1896-1897, p. 65.

Economics 15.
Mid-Year Examination,
1896-97

N.B. — Not more than eight questions must be attempted.

  1. Explain the influence of the Greek conception of the State upon the economic speculation of Greek philosophers.
  2. “The sacredness of property is a notion far more fixed in modern than in ancient times.” Comment upon this remark
  3. Explain and illustrate the influence of the example of Sparta on Greek social thought.
  4. In what sense is it true that Plato anticipated Adam Smith’s teaching concerning division of labour?
  5. In what sense is it true that Aristotle anticipated the modern distinction between Value in Use and Value in Exchange.
  6. Set forth briefly Aristotle’s doctrine as to Chrematistic.
  7. Discuss the question as to whether Christianity destroyed slavery.
  8. What sanction, if any, has Socialism or Communism in the teaching of the Christian Fathers?
  9. Trace the early history of the doctrine of “interest” in the original sense of that word.
  10. Distinguish between the various senses attached to the term Mercantilism. Which do you think most convenient?
  11. What ideas prominent in modern Protectionist argument are absent from Mercantilism as represented by Mun?

Source: Harvard University Archives. Harvard University Mid-Year Examination, 1852-1943. Box 4. Bound volume: Examination Papers, Mid-Years 1896-97.

Economics 15.
Year-End Examination,
1896-97

N.B. — Not more than eight questions must be attempted.

  1. Mention some of the practical questions which called forth economic pamphlets in the period 1650-1700, and give some illustrations.
  2. Illustrate and criticize the attitude of the mercantilist writers towards the regulation of internal industry.
  3. Explain the relation of Locke’s doctrine of price to the immediate purpose of his Considerations.
  4. Describe, as definitely as possible, the relation of the Physiocrats to the other reforming or revolutionary movements of their time.
  5. Criticize Adam Smith’s criticism of Physiocratic doctrine.
  6. What elements in his teaching do you conceive Adam Smith to have derived from Hume?
  7. What does Smith mean by “the component parts of the price of commodities”?
  8. Compare Smith’s definition of capital with that of John Stuart Mill.
  9. What does Smith mean by “the natural rates of wages”?
  10. What bearing has the teaching of Malthus on “Socialism.” Explain in your answer what sort of “Socialism” you have in mind, and mention the sources whence you have derived your impression of it.

Source: Harvard University Archives. Harvard University Examination Papers, 1873-1915. Box 4. Bound volume: Examination Papers 1896-97. Papers Set for Final Examinations in Philosophy, History, Government, Economics, Fine Arts, Architecture, and Music in Harvard College, June 1897, pp. 50-51.

_____________________

Economics 2.
Economic Theory from the Middle of the Nineteenth Century to the Present Time,
1896-97

Course Announcement

[Economics] *2. Economic Theory from the Middle of the Nineteenth Century to the Present Time. — English Writers. — The Austrian School. Mon., Wed., Fri., at 2.30. Professor Taussig.

Source: Harvard University. Announcement of the Courses of Instruction Provided by the Faculty of Arts and Sciences for the Academic Year 1896-1897, p. 34.

Course Enrollment

[Economics] 2. Professor Taussig. — Economic Theory from the Middle of the Nineteenth Century to the Present Time. — English Writers. — The Austrian School. 3 hours.

Total 42: 12 Graduates, 12 Seniors, 13 Juniors, 2 Sophomores, 3 Other.

Source:   Harvard University, Annual Reports of the President and Treasurer of Harvard College, 1896-97, p. 65.

Economics 2.
Mid-Year Examination,
1896-97

  1. “According to Ricardo, the exchange value of commodities contains neither return to capital nor rent, but simply labor.” Why? or why not?
  2. Sketch concisely the development of the general theory of value at the hands of Ricardo, Mill, Cairnes.
  3. “Skill, as skill, produces no effect on value; in other words, commodities do not under any circumstances exchange for each other in proportion to the degree of skill bestowed on them. Skill, though in itself inoperative on value, nevertheless affects it indirectly in two distinct ways; first, where competition is effective among producers, through the cost which must be undergone in acquiring the skill; . . . and secondly, in the absence of competition, through the principle of monopoly.” — Cairnes.
    Explain and illustrate.
  4. “If there really was a national fund the whole of which must necessarily be applied to the payment of wages, that fund could be no other than an aggregate of smaller similar funds possessed by the several individuals who compose the employing part of the nation. Does, then, any individual employer, possess any such fund? Is there any specific portion of any individual’s capital which the owner must necessarily expend upon labour? . . . May he not spend more or less on his family and himself, according to his fancy, — in the one case having more, in the other less, left for the conduct of his business? And of what is left, does he or can he determine beforehand how much shall be laid out on buildings, how much on materials, how much on labour? . . . Be it observed, fixity of definiteness is the very essence of the supposed wages-fund. No one denies that some amount or other must within any given period be disbursed in the form of wages. The only question is, whether that amount be determinate or indeterminate.” — Thornton, On Labour.
    State carefully, and consider critically, the answers Cairnes made to these questions.
  5. Would you accede to the statement that “President Walker’s theory is, in reality, not a theory of manager’s earnings at all, but a theory of differences in manager’s earnings”?
  6. “For an understanding of the machinery by which distribution is accomplished, the classification of sources of income should thus be different from that to be adopted for an explanation of the fundamental causes.” — Taussig.
    Wherein different?
  7. Explain what is meant by Consumer’s Rent; and consider how its significance is affected by inequalities in wealth.
  8. “As a rule, the poorer soils rise in value relatively to the richer, as the pressure of population increases.” — Marshall. Why?

Source: Harvard University Archives. Harvard University Mid-Year Examination, 1852-1943. Box 4. Bound volume: Examination Papers, Mid-Years 1896-97.

Economics 2.
Year-end Examination,
1896-97

  1. Do you believe that a permanent gain for the theory of wages has been made by Walker’s discussion of that subject? If so, wherein? if not, why not?
  2. Does Marshall’s analysis of the different grades of labor, and of the barriers between them, differ in essentials from Cairnes’s? from Mill’s?
  3. Explain what “quasi-rent” is, wherein it differs from true rent, wherein resembles true rent; and state whether the conception seems to you a helpful one, deserving to be permanently embodied in economic theory.
  4. What do you conceive the difference to be between what Walker calls “current product,” Marshall “the national dividend,” and the instructor in the course “real income”?
  5. On what grounds does Marshall maintain that “the extra income earned by natural abilities may be regarded as a rent, when we are considering the sources of the income of individuals, but not with reference to the normal earnings of a trade”? What is your own opinion?
  6. “The attribute of normal value implies systematic and continuous production.” Cairnes. Would Böhm-Bawerk accede to this proposition? Why, or why not? Give your own opinion.
  7. Explain what Böhm-Bawerk means by (subjective) “value”; and consider his analysis of the relation between value and cost.
  8. Enumerate the grounds on which Böhm-Bawerk maintains that “present goods have greater value than future goods of like kind and quantity”; consider to which of these grounds he gives most attention; and give your opinion as to the justice of this emphasis.

Source: Harvard University Archives. Harvard University Examination Papers, 1873-1915. Box 4. Bound volume: Examination Papers 1896-97. Papers Set for Final Examinations in Philosophy, History, Government, Economics, Fine Arts, Architecture, and Music in Harvard College, June 1897, p. 38. Previously transcribed: https://www.irwincollier.com/harvard-history-of-economic-theory-final-exam-questions-taussig-1897-1900/

_____________________

Economics 13.
Scope and Method in Economic Theory and Investigation

[*13 hf. Scope and Method in Economic Theory and Investigation. Half-course. Professor Taussig.]

Omitted in 1896-97; to be given in 1897-98.

Source: Harvard University. Announcement of the Courses of Instruction Provided by the Faculty of Arts and Sciences for the Academic Year 1896-1897, p. 34.

_____________________

Economics 3.
The Principles of Sociology.
1896-97

Course Announcement

[Economics] *3. The Principles of Sociology. —Development of the Modern State, and of its Social Functions. Mon., Wed., (and at the pleasure of the instructor) Fri., at 1.30. Asst. Professor Edward Cummings.

Source: Harvard University. Announcement of the Courses of Instruction Provided by the Faculty of Arts and Sciences for the Academic Year 1896-1897, p. 34.

Course Enrollment

[Economics] 3. Asst. Professor Edward Cummings. — The Principles of Sociology. — Development of the Modern State, and of its Social Functions. 2 hours.

Total 47: 6 Graduates, 23 Seniors, 13 Juniors, 2 Sophomores, 3 Other.

Source:   Harvard University, Annual Reports of the President and Treasurer of Harvard College, 1896-97, p. 65.

Economics 3.
Mid-Year Examination,
1896-97

(It is the purpose of the following questions to elicit first, an intelligent statement of the gist of what has been read or discussed; second, a free statement of an intelligent opinions you may be forming for yourselves.
Arrange your answers in the order in which the questions stand. Limit the discussion of each question to about an hour.)

I.

A critical comparison of Mr. Giddings’ treatise on the principles of Sociology with that of Mr. Spencer:—
(a) In regard to method, arrangement, and terminology :
(b) In regard to fundamental resemblances in the theories presented:
(c) In regard to supposed differences

II.

“Next in order come the problems of the social consciousness, or social mind, including its content of common memories and ideas, its aspirations and its volition.” What explanation of the phenomena in question is offered by (a) Giddings; (b) Spencer; (c) Durkheim; (d) Tarde? What is your opinion of the relative merits and the practical bearing of the several explanations?

IlI.

The significance and the function of the family (a) in the earlier and (b) in the later phases of social evolution.
(c) If you still have time, give some account of the successive theories and of the present state and significance of the controversy in regard to early forms of marriage.

Source: Harvard University Archives. Harvard University Mid-Year Examination, 1852-1943. Box 4. Bound volume: Examination Papers, Mid-Years 1896-97.

Economics 3.
Year-end Examination,
1896-97]

[Answer the questions in the order in which they stand. Give an hour to each group.]

I.

Discuss the conceptions of progress found in the following authors: Spencer, Comte, Giddings, Kidd, Kelly. Bluntschli. Indicate in each case (a) the nature of progress, (b) the criteria, (c) the chief causes.

What do you mean by progress?

What evidence of progress do you find in the historical sequence of the various forms of political organization which have lead to the modern state? Illustrate carefully.

Name and classify the principal types of political organization. Indicate briefly the social and industrial characteristics of each type.

II.

Give a critical summary of the views of Haycraft in Darwinism and Race Progress.

By what other writers, ancient or modern, have similar views been urged?

What importance do you attach to this school of thought?

What is the practical bearing of such views upon (a) the problems of scientific philanthropy and the treatment of defective and criminal classes? (b Upon socialism?

III.

State and criticise Bluntschli’s theory as to the nature and functions of the State, — the relation of the State to society and to the individual.

Compare Bluntschli’s theory with that of other writers, — Pollock, Spencer, Ritchie, Giddings.

What in your opinion are the merits and the defects of Bluntschli’s treatise?

Source: Harvard University Archives. Harvard University Examination Papers, 1873-1915. Box 4. Bound volume: Examination Papers 1896-97. Papers Set for Final Examinations in Philosophy, History, Government, Economics, Fine Arts, Architecture, and Music in Harvard College, June 1897, p. 39.

_____________________

Economics 14.
Socialism and Communism.

Course Announcement

[Economics] *14. Socialism and Communism. — History and Literature. Tu., Th., (and at the pleasure of the instructor) Sat., at 9. Asst. Professor Edward Cummings.

Source: Harvard University. Announcement of the Courses of Instruction Provided by the Faculty of Arts and Sciences for the Academic Year 1896-1897, p. 34.

Course Enrollment

[Economics] 14. Asst. Professor Edward Cummings.—Communism and Socialism.—History and Literature. 2 hours.

Total 13: 10 Seniors, 2 Juniors, 1 Sophomore.

Source: Harvard University. Report of the President of Harvard College, 1896-97, p. 65.

Economics 14.
Mid-Year Examination,
1896-97

(Arrange your answers in the order of the questions. Omit one.)

  1. The different senses in which the word Socialism is used. Where do you intend to draw the line between Socialism proper, and familiar forms of government interference and control – such as factory legislation, municipal water works, and government postal, telegraph or railroad services? Why?
  2. “National communism has been confused with the common ownership of the family; tenure in common has been confused with ownership in common; agrarian communism with village commons.” Discuss the evidence.
  3. “Just as Plato had his Republic, Campanella his City of the Sun, and Sir Thomas More his Utopia, St. Simon his Industrial System, and Fourier his ideal Phalanstery…. But the common criticism of Socialism has not yet noted the change, and continues to deal with the obsolete Utopias of the pre—evolutionary age.” What do you conceive to be the character of the change referred to? How far did earlier Utopias anticipate the ideals of the modern social democracy?
  4. What indication of Socialistic tendencies are to be found in the discipline of the Christian church? Explain the triple contract and its bearing on the doctrine of the usury.
  5. The contributions of Greek writers to the development of economic thought.
  6. To what extent are the theories of Karl Marx indebted to earlier writers in the 19th-century?
  7. How far are the economic series of (a) Lasalle, (b) Marx related to the theories of the so-called orthodox Economists? Explain critically.
  8. How far do you trace the influence of historical conditions in the social philosophies of Plato, More, Bacon, Rousseau, St. Simon, Karl Marx?

Source: Harvard University Archives. Harvard University Mid-Year Examination, 1852-1943. Box 4. Bound volume: Examination Papers, Mid-Years 1896-97.
Previously transcribed: https://www.irwincollier.com/harvard-exams-and-enrollment-for-economics-of-socialism-and-communism-edward-cummings-1893-1900/

Economics 14.
Year-end Examination,
1896-97

I.

  1. “The figures of Cardinal Manning and Monsignor Bagshawe in England, of De Mun in France, of Decurtins in Switzerland, of Abbé Hitz in Germany, and of Von Vogelsang in Austria, will ever stand apart as State Socialists who, while looking to the Church for moral reform, expected and wished all economic reforms to come from the State alone.”
    Give some account of the leaders mentioned, and discuss the peculiarities of so-called Catholic Socialism in each country.
  2. How far does the programme of “Catholic Socialism” in different countries harmonize with the programme of the German Social Democratic party.
  3. State carefully what has been the attitude of the Vatican towards Socialism? What are the personal views of Leo XIll?

II.

  1. Describe the origin, development, fortunes and present strength of the Social Democratic party in Germany, — with special reference to the Eisenach, Gotha and Erfurt programmes.
  2. State and criticise Marx’s Theory of Surplus Value. Explain carefully the formula

\left( S=P_{n}\times \frac{s\ l\  t}{n\  l\  t} \right)

  1. What are Schäffle’s chief criticisms of the Socialistic State?

III.

[Take one question.]

  1. “Though social conflicts are as old as civilization itself, Socialism as we now understand it is of scientific origin, and essentially modern.” State carefully your reasons for agreeing or disagreeing.
  2. What are the characteristics of modern Utopian ideals, as contrasted with the ideals embodied in earlier literature of the kind? Contrast Bellamy, Hertzka, Morris.
  3. Trace in a general way the influence of socialistic doctrines in the establishment of socialistic and communistic societies in the United States. What light has experiment thrown upon socialistic and communistic ideals?
  4. State accurately the reading you have done in this course during the second half-year. Give a careful summary of the views of the author you recently selected for your special reading.

Source: Harvard University Archives. Harvard University Examination Papers, 1873-1915. Box 4. Bound volume: Examination Papers 1896-97. Papers Set for Final Examinations in Philosophy, History, Government, Economics, Fine Arts, Architecture, and Music in Harvard College, June 1897, p. 49-50.

_____________________

Economics 9.
The Labor Question in Europe and the United States

Course Announcement

[Economics] 9. The Labor Question in Europe and the United States. — The Social and Economic Condition of Workingmen. Tu., Th., Sat., at 10. Asst. Professor Edward Cummings and Dr. John Cummings.

Source: Harvard University. Announcement of the Courses of Instruction Provided by the Faculty of Arts and Sciences for the Academic Year 1896-1897, p. 34.

Course Enrollment

[Economics] 9. Asst. Professor Edward Cummings and Dr. John Cummings. — The Labor Question in Europe and the United States. — The Social and Economic Condition of Workingmen. 3 hours.

Total 50: 5 Graduates, 9 Seniors, 27 Juniors, 7 Sophomores, 2 Others.

Source:   Harvard University, Annual Reports of the President and Treasurer of Harvard College, 1896-97, p. 65.

Economics 9.
Mid-Year Examination,
1896-97

Divide your time equally between A and B.

A.

  1. The labor problem and the unity of the labor movement as manifested in trade unionism, co-operation, and socialism.
  2. How, if at all, has the introduction of machinery directly modified methods of industrial remuneration?
    What are the several bases for determining remuneration?
    Explain the unpopularity, (a) with employers and (b) with employees, of certain methods of industrial remuneration, and, if you can, offer some general principle justifying the adoption of one method rather than another.
    Tell in what way, if at all, the amount of remuneration is affected by the method of remuneration.

B.

Take six.

  1. The “Old” and the “New” Trade-Unionism. Give an account of growth of the English Trade Unions.
  2. What social and economic motives have contributed to the growth of modern cities? Explain so far as you are able the migratory movements which have led to the aggregation of population within certain industrial centres.
  3. Define: “nibbling”; “lump system “; “pay-as-you-please” piece work; the “lump of labor” theory; “chasing”; “collective gain-sharing”; the “plus” system; “butty-gangs”; “tut-work”; “working in pocket”; “garret master”: “product-sharing”; “bribe participation.”
  4. How far are the various forms of profit-sharing “sops to Cerberus”? What is the essential difference between a profit-sharing firm and a co-operative association? How far does industrial co-operation enable workmen to become their own employers?
  5. To what extent is the development of modern machine industry dependent upon the location of the world’s coal fields?
  6. What are the economic and social conditions of industrial depressions?
  7. Are the evils of sweating due to underpay, to overwork, or to the method of remuneration? Are they peculiar to some particular method of remuneration?
  8. The methods of estimating the annual revenue of Great Britain and its distribution.
  9. Has the introduction of machinery lessened the demand for labor?

Source: Harvard University Archives. Harvard University Mid-Year Examination, 1852-1943. Box 4. Bound volume: Examination Papers, Mid-Years 1896-97.

Economics 9.
Year-end Examination,
1896-97

I.

  1. State the subject of your special research during the second half-year. How far were you successful in getting material from (a) newspapers, (b) magazines, (c) books, (d) other sources? What general results did you reach?
  2. Explain the essential difference between a socialistic policy and what Schäffle calls a “positive social policy.” What, according to Schäffle, are legitimate objects of protective legislation? Give some recount of German protective legislation, and the “factory labor” to which this legislation applies.
  3. What according to Mallock, determines the minimum wage in any occupation? How far is this minimum rate subject to legislative or other control? Has the introduction of machinery tended to raise or lower this rate? To increase or to lessen the proportion of wages to product?
  4. What do you understand by the statement that “today the labor contract is perfectly free”? Discuss the legality of (a) strikes; (b) boycots; (c) intimidation; (d) “molesting.”
  5. Describe accurately the German compulsory system of old-age insurance. State the precise arguments which are urged for and against the adoption of some such system in England. How far may the German system be said to be the product of peculiar local conditions?

II.

[Omit two questions.]

  1. Describe some of the chief agencies and methods devised for dealing with the unemployed. How far have they been successful?
  2. Describe the strength, composition and programme of the labor party in Belgium, and its relation to trade-unions, cooperation and socialism. Contrast the situation in Belgium with that in Germany.
  3. Show in what respects the general policy of France towards Associations of workingmen during the present century has differed from that of England. Indicate briefly the effects of this policy upon trade-unions, coöperation, etc.
  4. Discuss, the growth, character, programme and strength of existing labor organizations in the United States, — contrasting the situation in this country with the situation in England and explaining differences.
  5. By what peculiar local conditions in each case do you account for the success or the failure of the coöperative movements in (a) England, (b) France, (c) the United States? What do you predict for the future of coöperation?

Source: Harvard University Archives. Harvard University Examination Papers, 1873-1915. Box 4. Bound volume: Examination Papers 1896-97. Papers Set for Final Examinations in Philosophy, History, Government, Economics, Fine Arts, Architecture, and Music in Harvard College, June 1897, pp. 44-45.

_____________________

Economics 4.
Theory and Methods of Statistics,
1896-97

Course Announcement

[Economics] *4, Theory and Methods of Statistics. — Applications to Economic and Social Questions. —  Studies in the Movement of Population. Mon., Wed., Fri., at 11. Dr. John Cummings.

Source: Harvard University. Announcement of the Courses of Instruction Provided by the Faculty of Arts and Sciences for the Academic Year 1896-1897, p. 34.

_____________________

Course Enrollment

[Economics] 4. Dr. John Cummings. — Theory and Methods of Statistics. — Applications to Economic and Social Questions. — Studies in the Movement of Population. 3 hours.

Total 15: 8 Seniors, 7 Juniors.

Source: Harvard University. Report of the President of Harvard College, 1896-1897, p. 65.

Economics 4.
Mid-Year Examination,
1896-97

[Divide your time equally between A. and B.]

A.

  1. The development of scientific statistics and the statistical method as employed in the social sciences.
  2. Social and economic causes of the migratory movements which have taken place in the populations of Europe and America during this century, and the laws in accordance with which those migrations have taken place where you can formulate any.

B.

(Take five.)

  1. Rural depopulation and the growth of cities in the United States.
  2. Define: “mean after life,” “expectation of life,” “mean duration of life,” “mean age at death.” What relation does the mean age of those living bear to the mean age at death? To the mean duration of life?
  3. Anthropological tests of race vitality as applied to the American negro?
  4. Explain how the economic value of a population is effected by its age and sex distribution.
  5. The United States census: either (1) an historical account of it, or (2) an account of the work now undertaken by the Census Bureau.
  6. Explain the various methods of calculating the birth rate of a population.
  7. How far are social conditions in a community revealed in the birth rate, the death rate, the marriage rate? Of what are fluctuations in these rates evidence in each case?
  8. What do you understand by the “index of mortality”?

Source: Harvard University Archives. Harvard University Mid-year Examinations, 1852-1943. Box 4. Bound volume: Examination Papers, Mid-Years. 1896-97.

Economics 4.
Year-End Examination,
1896-97

I.

  1. Give an historical account of the United States census, and a general statement of the ground covered in the census of 1890; also show how the census taking is supplemented by work done in the Department of Labor and in the statistical bureaus established in connection with the several administrative departments.
  2. Define Körösi’s “rate of natality,” and state any statistical evidence you know that the rate is affected by the standard of living.
  3. “It must, at all times, be a matter of great interest and utility to ascertain the means by which any community has attained to eminence among nations. To inquire into the progress of circumstances which have given pre-eminence to one’s own country would almost seem to be a duty….The task here pointed out has usually been left to be executed by the historian.” Porter: “The Progress of the Nation.”
    What contribution has statistics to make in the execution of this task? What do you understand to be the nature of the statistical method, and what are the legitimate objects of statistical inquiry?

II.

[Take two.]

  1. What light does statistics throw upon the “natural history of the criminal man”?
    Give Ferri’s classification of the “natural causes” of crime, and comment upon that classification. Of criminals.
    What do you understand by “rate of criminality”? By “criminal saturation”?
  2. To what extent in your opinion is suicide an evidence of degeneration in the family stock?
    Discuss the influence upon the rate of suicide of education, religious creed, race, climate and other facts of physical, political and social environment.
  3. Comment critically upon the tables relating to crime in the last five federal censuses taken in the United States.
  4. What difficulties beset a comparative study of criminality in different countries?
  5. How far is it possible to give a quantitative statement to moral and social facts?

III.

[Take one.]

  1. What are some of the more salient facts concerning the movement of population and wealth in the United States, England, and France during the present century, so far as those facts are evidenced in the production, consumption and distribution of wealth?
  2. Discuss the movement of wages and prices in the United States since 1890.
  3. What do you understand by “index figures,” “average wages,” “average prices,” and “weighted averages”?

IV.

[Take one.]

  1. How do you account for the increase in the proportion of urban to rural population during this century? What statistical evidence is there that the increased density of a population affects the mean duration of life? What importance to you attach to this evidence?
    Explain the effect of migratory movements upon the distribution of a population according to age, sex and conjugal condition, and upon the birth rate, death rate and marriage rate.
  2. Define and distinguish: “mean age at death”; “mean duration of life”; “mean age of those living”; “expectation of life.”
  3. The “law of population” as formulated by Malthus and by subsequent writers.

Source: Harvard University Archives. Harvard University, Examination papers, 1873-1915. Box 4, Bound volume: Examination Papers 1896-97. Papers set for Final Examinations in History, Government, Economics, Philosophy, Fine Arts, Architecture, and Music in Harvard College. June 1897, pp. 39-41.
Previously transcribed: https://www.irwincollier.com/harvard-semester-exams-for-statistics-john-cummings-1896-1900/

_____________________

Economics 51.
Railway Transportation
1896-97

Course Announcement

[Economics] 51 hf. Railway Transportation. — Lectures and written work. Half-course. Tu., Th., Sat., at 1.30 (first half-year). Professor Taussig.

Source: Harvard University. Announcement of the Courses of Instruction Provided by the Faculty of Arts and Sciences for the Academic Year 1896-1897, p. 34.

Course Enrollment

[Economics] 51. Mr. Virtue. — Railway Transportation. 3 hours.

Total 62: 2 Graduates, 33 Seniors, 20 Juniors, 5 Sophomores, 2 Others.

Source:   Harvard University, Annual Reports of the President and Treasurer of Harvard College, 1896-97, p. 65.

Economics 51.
Final Examination,
1896-97

[Omit one.]

  1. Sketch the railway history of Italy.
  2. What light does the internal improvement movement in the United States throw upon the question of the public management of the railway industry?
  3. What is the present position of the courts with regard to the power of State legislatures to fix railway rates? On what ground does the public claim the right to interfere in the fixing of such rates.
  4. Discuss the attempt made by any one of the State governments to control railways by means of a commission.
  5. Describe the Hungarian system of passenger tariffs and its effects.
  6. As a basis of rate making, what is the relative importance of the principle of “charging what the traffic will bear,” and that of charging according to the “cost of service”?
  7. It is said of the railway business that “where combination is possible, competition is impossible.” Why is this true? Is it peculiarly true in the railway business? Give your reasons fully.
  8. Discuss the rate policy set forth in the following passages:
    “In the proposed reform of our transportation taxes it will probably be found advisable, at the beginning, to follow the example of Sir Rowland Hill in his reform of the old English postal system, and adopt, as the uniform rate for each class of service for all distances, the lowest rate now charged for the shortest distance for that class of service.”… “If distances of hundreds of miles, can be safely disregarded in the local transportation of milk and potatoes and grain, then surely there is every reason to believe that a general grouping of all the railway stations in the country with a uniform rate will prove to be the best possible system that can be devised for the common good of all.”
  9. A railroad company subject to the “Interstate Commerce Law” charges a much lower rate from New Orleans to San Francisco, for goods which have been imported than for like goods of domestic manufacture. What is the decision of the courts as to the legality of such a charge? What is the economic justification for the decision?
  10. What do you regard as the greatest defects of the “Interstate Commerce Law,” as at present interpreted, and what legislation should you suggest to remedy such defects?
  11. What is the “railway problem”?

Sources: Harvard University Archives. Harvard University Mid-Year Examination, 1852-1943. Box 4. Bound volume: Examination Papers, Mid-Years 1896-97. Copy also in Harvard University Archives. Harvard University, Examination papers, 1873-1915. Box 4, Bound volume: Examination Papers 1896-97. Papers set for Final Examinations in History, Government, Economics, Philosophy, Fine Arts, Architecture, and Music in Harvard College. June 1897, pp. 41-42.

_____________________

Economics 61
History of Tariff Legislation in the United States,
1896-97

Course Announcement

[Economics] 62 [sic] hf. History of Tariff Legislation in the United States. Half-course. Tu., Th., at 2.30, and a third hour at the pleasure of the instructor (second [sic] half-year). Professor Taussig.

Source: Harvard University. Announcement of the Courses of Instruction Provided by the Faculty of Arts and Sciences for the Academic Year 1896-1897, p. 34.

Course Enrollment

[Economics] 61. Professor Taussig. — History of Tariff Legislation in the United States. Hf. 2 hours. 1st half-year.

Total 74: 7 Graduates, 39 Seniors, 20 Juniors, 3 Sophomores, 5 Others.

Source:   Harvard University, Annual Reports of the President and Treasurer of Harvard College, 1896-97, p. 66.

Economics 61.
Final Examination,
1896-97

[Arrange your answers strictly in the order of the questions. Answer all the questions.]

  1. How was the course of commercial history between 1789 and 1810 connected with customs legislation and with the state of opinion on the tariff during the same period?
  2. What were the “abominations” of the tariff act of 1828? What became of them?
  3. Was there similarity between the stages in tariff policy reached by England and by the United States in 1846?
  4. “The ultimate reduction of the price of American to that of the British rolled iron can only, and ultimately will, be accomplished in that western region, which abounds with ore, and in which is found the most extensive formation of bituminous coal that has yet been discovered in any part of the globe.” — Gallatin, in 1832.
    What were the conditions as to the production of iron in the United States when Gallatin wrote? When and how was his prediction fulfilled?
  5. When did the argument appear that protective duties serve to maintain a high rate of wages in the United States? Why at that time? How far do you think it sound?
  6. Why is no carpet wool raised in the United States? Why is no flax cultivated for fibre? Are high wages an obstacle to the production of such commodities?
  7. Was Webster a consistent advocate of free trade in 1824? Gallatin in 1832? Secretary Walker in 1846?
  8. Sketch the main features in the history of duties on wool and woollens from 1846 to 1896.
  9. Wool, pig iron, sugar, — on which would you now remit duties first, and on which last? Why? What are the present duties?

Source: Harvard University Archives. Harvard University Mid-Year Examination, 1852-1943. Box 4. Bound volume: Examination Papers, Mid-Years 1896-97.

Copy also in Harvard University, Examination papers, 1873-1915. Box 4, Bound volume: Examination Papers 1896-97. Papers set for Final Examinations in History, Government, Economics, Philosophy, Fine Arts, Architecture, and Music in Harvard College. June 1897, pp. 42-43.

_____________________

ECONOMICS 81.
History of Financial Legislation in the United States

[81 hf. History of Financial Legislation in the United States. Half-course. Tu., Th., at 2.30, and a third hour at the pleasure of the instructor (first half-year). Professor Dunbar.]

Omitted in 1896-97.

Source: Harvard University. Announcement of the Courses of Instruction Provided by the Faculty of Arts and Sciences for the Academic Year 1896-1897, p. 34.

_____________________

ECONOMICS 162.
Selected Topics in the Financial History of the United States,
1896-97

Course Announcement

[Economics] *161 [sic] hf. Selected Topics in the Financial History of the United States. Half-course. Tu., Th., at 2.30(first half-year [sic]). Professor Dunbar.
Course 16 may be taken as a full course by Graduate Students and by candidates for Honors in Political Science.

Source: Harvard University. Announcement of the Courses of Instruction Provided by the Faculty of Arts and Sciences for the Academic Year 1896-1897, p. 34.

Course Enrollment

[Economics] 62. Professor Dunbar. — Selected Topics in the Financial History of the United States. Hf. 2 hours. 2dhalf-year.

Total 21: 11 Graduates, 6 Seniors, 4 Juniors.

Source:   Harvard University, Annual Reports of the President and Treasurer of Harvard College, 1896-97, p. 66.

Economics 162.
Final Examination,
1896-97

  1. Under the present arrangement of our financial system how far is the original theory of the Independent Treasury Act of 1846 preserved?
  2. What are the considerations for and against the use of banks as government depositaries?
  3. Trace the steps by which the idea that redeemed notes might be reissued became familiar and was finally embodied in the existing law.
  4. How far was the successful resumption in 1879 the result of fortunate circumstances, not to be foreseen in 1875?
  5. What is the legal authority for receiving United States notes in payment of duties on imports?
  6. What would be the probable effect on the Treasury, if Congress were now to adopt a tariff producing ample revenue, without making any change in the currency legislation?
  7. In Secretary Sherman’s Report for 1879 “it is respectfully recommended that by law the resumption-fund be specially defined and set apart for the redemption of United States notes, and that the notes redeemed shall only be issued in exchange for or purchase of coin or bullion.”
    Trace the effect of such a provision in case of large exports of gold. How far would it have served to prevent the difficulties which have actually occurred since 1893?
  8. In his Report for 1889, Secretary Windom recommended the following measure: —
    “Issue Treasury notes against deposits of silver bullion at the market price of silver when deposited, payable on demand in such quantities of silver bullion as will equal in value, at the date of presentation, the number of dollars expressed on the face of the notes at the market price of silver, or in gold, at the option of the Government; or in silver dollars at the option of the holder. Repeal the compulsory feature of the present coinage act.”
    What was this measure expected to accomplish? How would its operation probably have differed from that of the Silver Purchase Act of 1890?

Source: Harvard University Archives. Harvard University Examination Papers, 1873-1915. Box 4. Bound volume: Examination Papers 1896-97. Papers Set for Final Examinations in Philosophy, History, Government, Economics, Fine Arts, Architecture, and Music in Harvard College, June 1897, pp. 51-52.

____________________

Economics 72.
The Theory and Methods of Taxation,
1896-97

Course Announcement

[Economics] *72 hf. The Theory and Methods of Taxation, with special reference to Local Taxation in the United States. Half-course. Mon., Wed., Fri., at 9 (second half-year). Mr. ——.

Source: Harvard University. Announcement of the Courses of Instruction Provided by the Faculty of Arts and Sciences for the Academic Year 1896-1897, p. 35.

Course Enrollment

[Economics] 72. Professor Taussig. — The Theory and Methods of Taxation, with special reference to Local Taxation in the United States. Hf. 3 hours. 2d half-year.

Total 51: 6 Graduates, 17 Seniors, 18 Juniors, 5 Sophomores, 5 Others.

Source:   Harvard University, Annual Reports of the President and Treasurer of Harvard College, 1896-97, p. 66.

Economics 72.
Final Examination,
1896-97

[Arrange your answers strictly in the order of the questions.]

  1. Which among the following would you call a “tax,” and why,—
    (a) an assessment for betterment (e.g. for a sidewalk) on real estate;
    (b) a liquor license;
    (c) the charge for a postage stamp;
    (d) the charge for tobacco in France;
    (e) the charge for a ticket on a Prussian railway.
  1. How far is there separation of local taxes on real estate from state taxes on real estate in France? in Prussia? in England? In American states? Which adjustment seems to you the best, and why?
  2. State the points of resemblance and the points of difference between the system of local taxation in England and the usual method of local taxation in the United States.
  3. Is a tax like the French personelle-mobilière adapted for use in American states? one like the Prussian Business Tax?
  4. Do you think progressive taxation to be sound in principle? Why? or why not?
  5. Point out similarities and differences between the methods of taxing the holders of securities in England and in Pennsylvania.
  6. What do you conceive to be the methods and effects of the taxation of mortgage debts in Massachusetts? in California?
  7. On what grounds would you approve or disapprove of taxes on inheritances and successions, levied by the several American states? of income taxes, similarly levied?
  8. “The statute in Massachusetts, which taxes corporations on their capital stock less the value of real estate and machinery, is indefensible. According to the Massachusetts law, corporations are taxable locally only on their real estate and machinery, while they are taxable for commonwealth purposes only on the value of their capital stock deducting the value of the machinery and the real estate, they are therefore taxed only once on their total property. Individuals, on the other hand, pay not only a general tax for state purposes, but also another general property tax for local purposes. Corporations thus are treated more leniently than individuals.” — Seligman.
    Is this an accurate statement of the legislation in Massachusetts? and are corporations more leniently dealt with than individuals?

One of the following questions may be substituted for any one of the preceding.

  1. What distinctions are made, in the process of assessment under the Prussian income tax, between incomes under 3000 marks and those above?
  2. How are state and local control combined in the assessment of income taxes in Prussia? in England?

Source: Harvard University Archives. Harvard University, Examination papers, 1873-1915. Box 4, Bound volume: Examination Papers 1896-97. Papers set for Final Examinations in History, Government, Economics, Philosophy, Fine Arts, Architecture, and Music in Harvard College. June 1897, pp. 43-44.

_____________________

Economics 71.
Financial Administration and Public Debts

[[Economics] *71 hf. Financial Administration and Public Debts. Half-course. Tu., Th., Sat., at 11 (first half-year). Professor Dunbar.]

Omitted in 1896-97. Courses 71 and 72 are usually given in alternation with Courses 121 and 122.

Source: Harvard University. Announcement of the Courses of Instruction Provided by the Faculty of Arts and Sciences for the Academic Year 1896-1897, p. 35.

_____________________

Economics 121.
Banking and the History of the Leading Banking Systems

Course Announcement

[Economics] *121 hf. Banking and the History of the leading Banking Systems. Half-course. Tu., Th., Sat., at 11 (first half-year). Professor Dunbar.

Source: Harvard University. Announcement of the Courses of Instruction Provided by the Faculty of Arts and Sciences for the Academic Year 1896-1897, p. 35.

Course Enrollment

[Economics] 121. Professor Dunbar. — Banking and the History of the leading Banking Systems. Hf. 3 hours. 1st half-year.

Total 47: 1 Graduate, 18 Seniors, 24 Juniors, 4 Others.

Source:   Harvard University, Annual Reports of the President and Treasurer of Harvard College, 1896-97, p. 66.

Economics 121
Final Examination,
1896-97

  1. If, as McLeod says, all modern banks are banks of issue, how is it that discussions and legislation about banking are chiefly taken up with questions as to bank notes?
  2. When can a note currency be said to be elastic, and what is necessary to give it that quality? Illustrate by actual cases.
  3. Name as many banking systems as you can, which
    (a) protect creditors by a stockholders’ liability of any sort; or
    (b) protect notes by a prior lien on assets.
  4. What advantages are derived from the system of branch banks, and where is it used?
  5. Which of the great banks, — the Bank of England, the Bank of France, the Reichsbank, —  appears to have the best title to be called a government bank?
  6. Suppose that, in a country having well developed banks, a large issue of inconvertible legal tender notes should be made; in what way should you expect its inflating effect upon prices and credits to be produced?
  7. How is the unequal distribution of national banks in the United States, during the years when banking under the national act was most profitable, to be explained?
  8. The following items being given, viz. :
Public Deposits £7.7 Other Securities £30.0
Other Deposits £48.0 Notes in circulation £28.1
Government Securities £20.7 Coin and Bullion £40.4
Government Debts £11.0

construct a Bank of England account, with its separate Departments of Issue and Banking.

  1. Why is it that a comparison of the English country bank circulation with the Scotch shows that one is gaining while the other is dying out?
  2. Under the German bank act what are the two requirements as to holding cash? What is counted as cash under these requirements respectively? Under what conditions does a bank find one or the other of these requirements practically inoperative?
  3. The account of the Bank of France may be simplified as follows:—
Cash, Surplus, and Profits fr. 225. Loans and Investments fr. 1.141.
Sundries fr. 72. Government Securities fr. 353.
Deposits fr. 822. Sundries fr. 75.
Notes fr. 3.612 Cash fr. 3.162.
fr. 4.731. fr. 4.731.

How much of its circulation could the bank pay off and yet earn its present profit? State the account as it would appear if such a change were made?

Source: Harvard University Archives. Harvard University, Examination papers, 1873-1915. Box 4, Bound volume: Examination Papers 1896-97. Papers set for Final Examinations in History, Government, Economics, Philosophy, Fine Arts, Architecture, and Music in Harvard College. June 1897, pp. 47-48.

_____________________

Economics 122.
International Payments and the Flow of the Precious Metals,
1896-97

Course Announcement

[Economics] *122 hf. International Payments and the Flow of the Precious Metals. Half-course. Tu., Th., Sat., at 11(second half-year). Professor Dunbar and Mr. Meyer.
Courses 121 and 122 are usually given in alternation with Courses 71 and 72.

Source: Harvard University. Announcement of the Courses of Instruction Provided by the Faculty of Arts and Sciences for the Academic Year 1896-1897, p. 35.

Course Enrollment

[Economics] 122. Professor Dunbar and Mr. Meyer. — International Payments and the Flow of the Precious Metals. Hf. 3 hours. 2d half-year.

Total 20: 9  Graduates, 2 Seniors, 6 Juniors, 3 Others.

Source:   Harvard University, Annual Reports of the President and Treasurer of Harvard College, 1896-97, p. 66.

Economics 122.
Final Examination,
1896-97

It is expected that one-half of the time of this examination will be required for the questions in division A of this paper.

A.

  1. It is often said that the resumption of specie payments by France and by the United States and the adoption of the gold standard by Germany made nearly simultaneous demands upon the world’s stock of gold. Discuss this statement at length.
  2. The transfer of international securities,
    (a) during the funding operations of the United States, 1872-79;
    (b) as a consequence of the French Indemnity payments.
    Take one of these two cases.
  3. The conditions which led to the flow of gold to the United States,
    (a) in 1896;
    (b) in the fiscal years 1880 and 1881.
    Take one of these two cases.

B.

  1. What were the contributions of Ricardo, Mill and Cairnes, respectively, to the discussion of the laws determining the exchange of commodities between nations?
  2. To what extent are the principles involved in exchanges of goods and services between nations also applicable in domestic exchanges?
  3. Describe the” triangular” operation in exchange between three countries, whereby an export of specie may take place from one of them before the price of exchange has fallen to the shipping point. Illustrate by an actual case.
  4. Explain the difference in rates for long exchange as compared with short, and show the conditions under which an unusual divergence of rates may exist. Illustrate by an actual case.
  5. Why is it that in the dealings between England and other countries bills of exchange are chiefly drawn upon England and few are drawn by her upon others? How are the transactions between England and the United States adjusted, when the bills are for the most part drawn by us upon England?

Source: Harvard University Archives. Harvard University, Examination papers, 1873-1915. Box 4, Bound volume: Examination Papers 1896-97. Papers set for Final Examinations in History, Government, Economics, Philosophy, Fine Arts, Architecture, and Music in Harvard College. June 1897, pp. 48-49.

_____________________

 

Primarily for Graduates.

Economics Seminary Announcement

[Economics] 20. Seminary in Economics. Mon., at 4.30.
Professors Dunbar, Taussig, and Ashley and Asst. Professor Edward Cummings will guide competent students in research on topics assigned after consultation. The Seminary will hold weekly meetings; and in addition each student will confer once a week, with the instructor under whose guidance he carries on his investigations.

Source: Harvard University. Announcement of the Courses of Instruction Provided by the Faculty of Arts and Sciences for the Academic Year 1896-1897, p. 35.

Economics Seminary Enrollment

[Economics] 20. Professors Dunbar, Taussig, and Ashley and Asst. Professor Edward Cummings. — Seminary in Economics.

Total 20: 17 Graduates, 2 Seniors, 1 Other.

Source:   Harvard University, Annual Reports of the President and Treasurer of Harvard College, 1896-97, p. 66.

Image Source: Memorial Hall, Harvard University. From Library of Congress Prints and Photographs Division Washington, D.C.

 

 

 

 

Categories
Economics Programs Harvard Undergraduate

Harvard. President of Harvard responding to Economics Dept Visiting Committee Report, 1952

In can hardly be surprising that the relationship between a visiting committe dominated by business people and an academic department of economics might suffer from incompatible visions of what constitutes “good” economic research, teaching and policy.

The Harvard’s visiting committee in 1950 thought the secret sauce missing from a proper economics department was a professorial advocate of business enterprise to counterbalance an alleged dominance of Keynesian and socialist positions. This was the principal criticism of the committee. Other shortcomings claimed were inadequate planning/coordination between graduate and undergraduate programs, too few professorial heavyweights teaching in the undergraduate program, and a tendency for professors’ policy consulting activities to crowd out their expected instructional and research duties.

The chairman of the economics department’s visiting committee at mid-century was the Chicago businessman, Clarence B. Randall (Harvard A.B., 1912).

Harvard President’s James B. Conant’s conclusion in his 1952 response:

Over the last fifteen years the Department of Economics has been at fault in not attempting to meet the Visiting Committee in a spirit of wholehearted cooperation. The Board of Overseers has been at fault, I venture to suggest, by not widening the membership of the Visiting Committee to include more professional economists and more businessmen who have been working closely with university economists.

___________________________

For private circulation NOT for publication

CONFIDENTIAL REPORT OF THE PRESIDENT OF THE UNIVERSITY TO THE TWO GOVERNING BOARDS ON THE DEPARTMENT OF ECONOMICS OF THE FACULTY OF ARTS AND SCIENCES

(Accepted by President and Fellows of Harvard College on January 7, 1952, and by the Board of Overseers on January 14, 1952)

TO THE PRESIDENT AND FELLOWS OF HARVARD COLLEGE:
TO THE BOARD OF OVERSEERS OF HARVARD COLLEGE:

On November 27, 1950, the Chairman of the Committee to Visit the Department of Economics reported in writing to the Board of Overseers. The report, which is attached, raises serious questions about future appointments to the permanent staff. As the President of Harvard is responsible for presenting to the two Governing Boards the names of those who he is persuaded should be the future professors, such questions concern him directly. I have, therefore, felt obliged to examine personally the validity of the “most pressing criticism” in the report of November 27, 1950. My findings and recommendations are contained in this confidential report which I hope may be accepted by formal vote of each of the Governing Boards in January.

For a number of months now I have been studying the teaching of economics at the university level. In so doing, I have talked with academic economists on three continents, with those employed by business and by government, and with members of the business community. I am convinced that the Harvard Department of Economics is a distinguished department. As far as the types of economic theory and analysis presented to the students are concerned, it is typical of departments of economics in the leading universities of the English-speaking world. The educational problems discussed in the first seven paragraphs of the Visiting Committee’s report are likewise typical. Indeed, they are not confined to teachers of economics. Difficulties in reconciling the needs of the undergraduate and the graduate student with the scholarly pursuits of a professor and calls for expert services are to be found in the majority of the departments of the Faculty of Arts and Sciences. It is the constant aim of the administration to hold the balance even between the various types of teaching and research. To this end, the informed criticisms of visiting committees are helpful. But important as these questions are, they do not warrant a special written report from the President of the University. Therefore, I shall state here only that I am satisfied that the department is taking steps to improve the teaching of undergraduates and will take further steps in this direction, particularly as regards the introductory course.

My examination of the status of economics in American universities today has revealed the fact that in at least fourteen major universities questions are being raised by persons who are not economists about the teaching of economics. It is a curious fact that at the same period of history in which there is a certain degree of national unrest about academic economists, one group of businessmen (the Committee for Economic Development) is closely associated with professors of economics in a series of investigations of vital problems. It seems a pity that the confidence that part of the business community has in at least some university economists does not receive as much publicity as do the attacks by others who claim our schools and colleges are teaching “collectivism.” Not that any such charge is made by the Overseers Committee here at Harvard. What is criticized is only “that the Department as presently constituted lacks balance with respect to the viewpoint of its members.” This is a reasonable criticism and warrants a careful investigation. The Visiting Committees of the Board of Overseers are both special pleaders for and critics of the departments or faculties which they visit. That doubts and questions should be raised by them from time to time on any or all matters is obviously of great advantage to the University.

At the outset of my inquiry the difficulties of formulating criteria for cataloguing the viewpoints of economists became evident. I tried the test of Keynesian and anti-Keynesian but soon discovered I was using a totally inadequate analytic tool; I became convinced that Keynes himself was an anti-Keynesian before he died. The Overseers report states categorically that there are “one or more socialists” in the Department. With this statement I must respectfully but firmly disagree and in so doing point out both the difficulties and the necessity of defining terms in the social sciences. The term “socialist” as used in countries where socialism is a live political issue means one who advocates by democratic political action “the nationalization of the means of exchange, production and distribution.” It might be a good thing to have a socialist on the staff of a department of economics, but as a matter of fact there are no socialist professors of economics at Harvard today.

One could classify economists, at least theoretically, in terms of their political beliefs, but except for communists and socialists this is a very difficult matter in the present flux of political opinion. Furthermore, people’s political convictions, like their religious beliefs, are often subject to violent change. Everyone speaks of the dangers of introducing political criteria into the consideration of academic appointments. If analyzed, I believe these dangers stem largely from the fact that political views do not represent a bias relevant to an academic intellectual discipline as does a philosopher’s adherence to a philosophic doctrine such as idealism or logical empiricism. Political opinions are temporary, emotional, and subject to change under social duress; it is to avoid such duress that politics and religion are considered “out of bounds” in judging persons for academic posts in the United States in the mid-twentieth century.

The Chairman of the Visiting Committee in his report speaks of a “social spectrum.” I have attempted to use this concept to classify present-day economists as radical or conservative without getting into the political quagmire to which I have just referred. I have had little success except that in a vague sort of way a number of informed observers have expressed the view that the leading universities of the nation were about equally radical or conservative as regards their departments of economics. But if the President is to direct a department or an ad hoc committee as to future appointments, he must have some more definite criteria as to a man’s position in the social spectrum, and these I have failed to find. For example, I find it difficult to decide whether advocacy of strengthening the Sherman Anti-Trust Act is radical or conservative. I ask myself was the Harvard Department a generation ago radical or conservative? In retrospect it seems conservative to many; forty years ago it was considered radical, as the free-trade point of view predominated. When I first took office, some discussion in the Board of Overseers indicated that there were those who used a man’s attitude towards organized labor as the touchstone of his radical or conservative outlook. This is no longer so. As a consultant to the Government, an economist may take a strong position as to need for immediate drastic action to offset a depression or control an inflation. In recent years such rather technical economic opinions have bulked large in some people’s minds in classifying economists as being to the left or right. For example, if you confine your attention to fiscal policy in the immediate past, you could find two professors in the Harvard Department today to place in opposition to one another. But I have become convinced that no criteria of lasting value in terms of a social spectrum can be devised for the guidance of any body charged with responsibility for nominating candidates for appointment in a department of economies.

Balance in a department of economics today, I have concluded. should be first, balance between special fields, and second, balance between types of methods employed by the professors. As to fields such as labor, agriculture, money and banking, the Committee has raised no issue; there is no problem special to economics here. The question arises in chemistry, in history, in biology, to name but three instances. As far as I can, I insist that for the permanent appointments a balance of fields be a secondary consideration since an adequate coverage of all subjects can be taken care of through appointing assistant professors. Rigid insistence on having each field represented by a permanent appointment limits the number of candidates and tends to encourage the appointment of “good” rather than “excellent” men. The same is true as regards methods. Yet, as in the case of special fields, I must admit that there should be some effort made to achieve a balance among the permanent members of the staff, provided that in so doing there is no sacrifice of the quality of the appointments.

From my studies I have concluded that a layman may well classify economists in three groups according to the methods they employ: (1) theorists using models and the logical deductive approach; (2) investigators concerned with statistical aggregate analysis; (3) an empirical approach to specific problems as illustrated by the ad hoc case study of business problems. I have the impression that, in general, college departments of economies are relatively weak as regards the third of these methodological classes. In contrast, the Harvard School of Business Administration is strong here and until recently has been less concerned with the other two methodological approaches. The Harvard Department of Economies, if I understand the Chairman correctly, has felt for some time that this relative methodological weakness needed correction. Two professors of the Business School faculty are now giving a course in the Department. Further, in a letter replying to the criticism of the Visiting Committee, the Chairman, speaking for the Department, writes:

“As a result of your letter and our discussions with you, we have carefully considered the question of balance of fields of interest in the Department. While we are not prepared to concede that we are more unbalanced than other departments of economics, we agree that our balance could be improved. In particular, our Department, like most others, could be improved if we had at least one member whose major interest was what we might call the economics of enterprise. We believe that this is a field of growing importance, but it is one that has not been widely cultivated in economics departments. An additional member of the Department who could give an undergraduate course in the Economics of Enterprise and a graduate seminar on the same lines would contribute to a better balance of the Department. We suggest that the Corporation consider allocating an additional permanent position to the Department at the full professor level.”

To follow this suggestion would lead to no end of difficulties in the Faculty of Arts and Sciences; other departments would be quick to press for an increase in their quota of permanent places. But I am glad to report that much the same end can be accomplished because the Dean of the Business School has expressed his interest in a joint appointment. With his consent and with the concurrence of the Provost I therefore recommend that the Corporation agree to appoint one full professor of economics over and above the quota allowed by the schedule of appointments for the Faculty of Arts and Sciences established a decade or so ago. I further recommend that this professor hold an appointment in three faculties, namely, the Faculty of Arts and Sciences, the School of Business Administration, and the School of Public Administration, and that his salary be charged to the three faculties in such amounts as the President shall determine. Further, that the nomination for the new chair be made by the permanent members of the Department of Economics of the Faculty of Arts and Sciences and six members of the Faculty of the School of Business Administration appointed by the President after consultation with the Dean, the two groups to sit together as a nominating committee, and the name or names thus nominated to be passed on by an ad hoccommittee as is usual in the Faculty of Arts and Sciences.

The directive to the nominating committee would be as follows: to submit one or more names of men of character, high scholarly distinction and first-rate teaching ability who have an understanding of business as it is actually operated. To that end, the man in question should have had contact as a scholarly investigator or consultant with the operations of industry and commerce; he should have an awareness of the positive role of business enterprise in a changing and developing economy. His teaching would be directed towards presenting to Harvard College students a realistic view of business management and its relation to the total economy. If this report is accepted by the two Governing Boards, I shall proceed with this appointment.

The last paragraph of the report of the Visiting Committee requires special comment. It is stated that “This problem of balance within the Department will not be solved by the ad hoc committees. There only the qualifications of the particular man are under consideration. It is not the function of such a committee to determine whether the man’s appointment will restore balance or add to lack of balance.” I must beg leave to take exception to this exposition of the role of the ad hoc committees, and in so doing call the attention of the new members of the Board of Overseers to the Report of the Special Committee to Review the Operation of the “Ad Hoc” Committees in the Faculty of Arts and Sciences. The ad hoc committees determine nothing definitely, that is true. But they advise the President and through him the two Governing Boards as to whether or not the appointment suggested by the department is the best possible appointment that can be made all things considered; and among the considerations are the needs of the department for teachers and scholars in this or that subdivision of the field and with this or that scholarly technique at their disposal.

An ad hoc committee does far more than pass on “the qualifications of the particular man under consideration”; an ad hoc committee often recommends that someone other than the candidate nominated by the department should be considered. And such recommendations have more than once resulted in the appointment of a person who had not even been on the list considered by the department. As presiding officer of these ad hoc committees, I can certify from experience as to their effectiveness; I can assure the members of the two Governing Boards that in the field of economies, as elsewhere, I shall endeavor to see to it that the names I present are in my opinion the names of the best people to appoint. For the temporary appointments at the assistant professor and instructor level, the Dean of the Faculty performs the same function as the ad hoc committee.

The acceptance of this report by the two Governing Boards will mean that they agree with me that the issue of an individual’s radicalism or conservatism or a man’s political attitude is inadmissible in connection with his appointment. (I have made it clear elsewhere that I would not be a party to the appointment of a member of the Communist Party, for reasons I need not here repeat.) Balance between special fields and different methodological approaches in economics we shall strive for, and I recognize that it is a proper function of the Governing Boards from time to time to see that this is done, though not with respect to a particular appointment. There will be no directives to the nominating group or the ad hoc committee in terms of a man’s political views or his position on what has been referred to as a social spectrum. Since that will be the case in economics as in other fields, only the validity of the evidence I present as to a man’s character and competence as a scholar and teacher will be relevant to the decision about an individual in either the Corporation or the Board of Overseers. Once the ground rules are determined by the two Boards, the responsible officials must be trusted to operate within them. On no other basis, in my opinion, can this University function satisfactorily.

In conclusion I wish to express my deep appreciation for the spirit in which the report of the Visiting Committee is written. The Chairman states that it is not his intention “to initiate controversy or to suggest that we view with extreme alarm any phase of the Department’s work.” And later in the report he states, “No friend of academic freedom need fear the purpose which underlies our comment on this matter. . . We would be the first to insist that a professor must teach that which he honestly believes and we know that the fact that this differs from viewpoints which we may hold as individuals is altogether immaterial.”

I feel sure that the Chairman speaks not only for his Committee but for the whole Board of Overseers when he makes these statements, which are by no means universally accepted today in the United States. I need hardly state that even questions concerning the criteria to be employed in judging candidates for appointment in controversial areas — questions that touch sensitive nerve centers in a university today — are quite within the province of the visiting committees. Indeed, no one can have the slightest objection to the critical discussion of these matters in a university, provided, as in this case, the discussion is initiated by duly constituted committees within a constitutional framework.

Unfortunately, the public criticisms of professors in these days do not all conform to the restrained pattern set by this report. Rather the demands for “firing” or “muzzling” professors or censoring textbooks have increased in number and intensity in the last few years. I suppose all members of the two Boards are familiar with such irresponsible attacks as those of Zoll in his “Reducators” and the rather violent statements about the teaching of economics emanating from more reputable sources. I mention these matters for they have a certain relation to the problem that a president of a university faces today when he must recommend action in a controversial area such as economics. The analogy with his distant predecessors’ problems in theology comes to mind.

The existence of hostile critics and extremists makes it imperative for fair-minded men concerned with the future of education to thrash out their differences of opinion around a table. Over the last fifteen years the Department of Economics has been at fault in not attempting to meet the Visiting Committee in a spirit of wholehearted cooperation. The Board of Overseers has been at fault, I venture to suggest, by not widening the membership of the Visiting Committee to include more professional economists and more businessmen who have been working closely with university economists. But the situation is better in both respects than it was a few years past; in my opinion it can be still further improved.

In these critical days when economic decisions play so vital a part in determining national and international policies, it is unfortunate that an atmosphere of hostility exists to some degree throughout the country between the management of industry and academic economists. Whatever can be done here at Harvard to increase the understanding between men of good will within and without the University cannot fail to be of service to the nation.

Respectfully submitted,
JAMES B. CONANT

January 3, 1952

___________________________

No. 2 REPORT OF THE COMMITTEE TO VISIT THE DEPARTMENT OF ECONOMICS

TO THE BOARD OF OVERSEERS OF HARVARD COLLEGE:

Some three years have elapsed since a written report has been submitted to the Board of Overseers on behalf of the Committee to Visit the Department of Economics.

Once each year since that last report the Committee has met socially at dinner with the members of the Department, has met privately with the Provost to discuss the problems of the Department, and then has met in executive session. In addition to these annual meetings, the individual members of the Committee have endeavored conscientiously to inform themselves privately about the organization of the Department and the program of instruction.

It is not our purpose in making a report at this time to initiate controversy or to suggest that we view with extreme alarm any phase of the Department’s work. We do have apprehensions, but our viewpoints have been fully expressed to the President and the Provost, both of whom have encouraged us at all times to be frank in such criticisms as we have had to present.

The particular points which we have to make may be rather briefly stated.

It will be recalled that in the last previous report attention was drawn to the fact that the Department seems to lack over-all planning. We still think that a sound criticism. The Department contains brilliant individuals who are passionately devoted to their particular approach to the subject matter. But it could hardly be fairly said that their efforts are coordinated into a comprehensive plan, whether it be furnishing undergraduates a well-rounded training in economies or carrying on research at the graduate level.

Another criticism which has been made by others is that the members of the Department seem to emphasize the importance of their work with graduate students to the prejudice of undergraduate instruction. Few, if any, of the distinguished members of the Department are ever seen or heard by undergraduates, and we think this a great loss. We believe it to be important that Harvard give her best to those undergraduates who for the first time in their lives are approaching this highly significant subject, in order that their minds may be stimulated and broadened by the inspiration of great teaching.

Another criticism which has been made is that too many members of the Department absent themselves for extensive periods from their University duties. We recognize the demands that Government properly makes upon the University for the loan of Harvard economists. We also believe that a Harvard professor can benefit through working on projects for business managements or labor organizations. There must, however, be reasonable balance between such occasional outside employment and the first duty of the professor to the University. We believe there is ground for this criticism of the Economics Department and that the matter warrants careful study by the Provost and the Department Chairman.

The most pressing criticism, however, which we have to offer is that the Department as presently constituted lacks balance with respect to the viewpoints of its members. It is particularly the trend toward lack of balance which disturbs us.

No friend of academic freedom need fear the purpose which underlies our comment on this matter. We would be the first to insist that a professor must teach that which he honestly believes and we know that the fact that this differs from viewpoints which we may hold as individuals is altogether immaterial. This is too obviously right to need discussion.

But that is a totally different question from that of believing that all viewpoints should be ably represented within the Department. The most determined champion of academic freedom would join us we believe in urging the importance of balance in a controversial field. This is an old and familiar problem at universities; for example, in the departments of philosophy. There we believe that sound university administration always seeks such balance. We suggest that comparable balance is not presently to be found within the Department.

We have in the Department, for example, one or more Socialists, some zealous followers of British economist, John Maynard Keynes, and some who advocate the extension of economic controls by Government. Some of these men are nationally known for their views and are both active and zealous in promoting Them. But on the other side of the social spectrum, the Department seems to lack men of equal ability and zeal who hold opposing views and are prepared to teach them.

This problem of balance within the Department will not be solved by the ad hoc committees. There only the qualifications of the particular man are under consideration. It is not the function of such a committee to determine whether the man’s appointment will restore balance or add to the lack of balance. That delicate question can be solved only through leadership by the President, the Provost, and the Chairman. We have confidence that they share our concern and we hope that this statement of our apprehensions will be helpful to them.

CLARENCE B. RANDALL

November 27, 1950

Source: Harvard University Archives. Confidential Report of the President of the University to the Two Governing Boards of the Department of Economics of The Faculty of Arts and Sciences, 1952 January 3 (Archives Stacks UAI.20.962.5)

___________________________

Members of Visiting Committee,
Department of Economics
1950-1952

Chairman:

Clarence B. Randall
President, Inland Steel Company

Vice-Chairman:

David Rockefeller
Foreign Department, Chase National Bank

Members:

Henry W. Clark
Maritime Associates

Jack I. Straus (1951-52)
President, H. H. Macy Company

Sinclair Weeks
United Carr Fasteners Corp., Reed and Barton Corp.

Frederick C. Crawford
President, Thompson Products, Inc.

David F. Edwards
President, Saco-Lowell Shops

Devereux C. Josephs
Carnegie Corporation

Walter Lichtenstein
First National Bank of Chicago

Thomas S. Lamont (1950-51)
New York

David E. Lilienthal
formerly Head of Atomic Energy Commission

Edward R. Mitton (1951-52)
Jordan Marsh Company

Gilbert H. Montague
New York Lawyer

Edwin G. Nourse
formerly with Council of Economic Advisers

Ralph Robey
National Association of Manufacturers

Charles F. Rowley
Peabody, Brown, Rowley and Storey

Hermon Dunlop Smith
Marsh and McLennon, Insurance, Chicago

George Terborgh
Allied Machinery

Leo Wolman
Professor, Columbia University

Source: Harvard University Archives. Department of Economics, Correspondence and papers (UAV 349.11), Box 25, Folder “Visiting Committee, 1950-52”.

Images Sources:

(Left)  James B. Conant PageAtomic Heritage Foundation website.
(Right) Portrait of Trustee of the University of Chicago, Clarence B. Randall, from the University of Chicago Photographic Archive, apf1-03000-082, Hanna Holborn Gray Special Collections Research Center, University of Chicago Library.

Categories
Economics Programs Economists Harvard

Harvard. The Data Resources Inc. connection. Galbraith asks Eckstein, Feldstein, Jorgensen. 1972

 

“As Ed Mason tactfully hints, I’ve had enough lost causes for one year.”–Galbraith

In the following exchange of letters initiated by John Kenneth Galbraith in December 1972 we find multiple instances of seething rage barely concealed under veneers of formal academic politeness. Critical hiring and firing decisions regarding the subtraction of radical voices from the economics department faculty went overwhelmingly for the consolidation of mainstream economics earlier that month and Galbraith appears to have sought a vulnerability of this counterrevolution in its potential for conflicts of interest as he imagined coming from Otto Eckstein’s start-up, Data Resources, Inc. Eckstein’s response provides us with some interesting backstory to DRI. Feldstein and Jorgensen offered their witness testimony regarding this early episode in what would ultimately result in the so-called empirical turn in economics

But even after suffering this tactical defeat, Galbraith’s strategic point was to be confirmed by history:

“I do have one final thought. In accordance with the well-known tendencies of free enterprise at this level, one day one of these corporations is going to go down with a ghastly smash. It will then be found, in its days of desperation or before, to have engaged in some very greasy legal operations. The Department and the University will be held by the papers to have a contingent liability. It will be hard to preserve reticence then. It would have been better to have taken preventative action now.”

The conflict of interest cases brought by the U.S. Department of Justice in 2000 against economics professor Andrei Shleifer and the Harvard Institute for International Development resulted in a settlement that required Harvard to pay $26.5 million to the U.S. government.

_____________________________

On behalf of the Department,
Galbraith wants to know more about DRI

JOHN KENNETH GALBRAITH
HARVARD UNIVERSITY
CAMBRIDGE. MASSACHUSETTS

December 20, 1972

Professor Otto Eckstein
Littauer Center

Professor Martin S. Feldstein
1737 Cambridge Street

Professor Dale W. Jorgenson
1737 Cambridge Street

Dear Otto, Marty and Dale:

It will hardly be news that I have been deeply concerned over the several recent actions of the Department of Economics on appointments as well as the academically less consequential problem of the less than gracious response to those of us who have expressed alarm.

There is an impression, of which you will undoubtedly be sensitive, that the positions of some of those favoring the recent action could reflect, however subjectively and innocently, their corporate involvement in conflict with their academic responsibilities. I do not wish in any way to prejudge this matter or even to be a source of embarrassment. The problem does seem to me sufficiently somber so that in the interest of everyone you no less than the rest of us the circumstances should be clearly known. In this spirit I raise the following questions:

  1. Could you indicate the nature of Data Resources, Inc? I have reference to assets, sales, employees, services rendered, identity of corporate clients and charges.
  2. I believe it can fairly be assumed from general knowledge that the Corporation owes part of its prestige and esteem to association with members of the Harvard Department of Economics. The foregoing being so and reputation being a common property of the Department and Harvard University, could I ask as to your ownership or other interest or other participation of whatever sort and return?
  3. Has the Corporation employed students and nontenured members of the Department of Economics and would you indicate the names?
  4. Could I ask if you have participated in the past in the consideration of Harvard promotion of any such employees, consultants or people otherwise associated with the Corporation and in what cases?
  5. Could past service or inferior service or present or potential utility to the Corporation or extraneous judgment based on business as distinct from academic performance create, again perhaps subjectively, the possibility of a conflict of interest in your passing on Harvard promotions? How have you handled this conflict in the cases in which people with an association, past or present, with the Corporation have been up for Harvard promotion, always assuming that there have been such cases?
  6. In the recruiting of clients for the Corporation, what of the danger that they will be affected by the close relation between the Corporation and the Department? Specifically could there be effort, however subjective, to quell their fears? The radical economists come obviously to mind. But, as you are perhaps aware, even I am not a totally reassuring figure to many businessmen department with too many people of my viewpoint might also evoke alarm. Does safety here suggest that one with major corporate interest disqualify himself on all appointments?
  7. Is there a possibility — I by no means press the point that the kind of economics that serves corporate interest will take on an exaggerated importance when some of our ablest faculty members, and students are working on such problems?

Let me repeat that I ask these questions only for a clarification in which we share a common interest. I do not of course raise the more general question of outside activity. This would come with very poor grace from me — it is indeed the reason why I have sought not to be a charge on university resources,

Yours faithfully,

John Kenneth Galbraith

CC: Professor James S. Duesenberry

Dean John T. Dunlop

JKG:mih

_____________________________

Eckstein provides his answers to Galbraith’s “interesting questions”

Otto Eckstein
24 Barberry Road
Lexington, Mass. 02173
January 8, 1973

Professor J. Kenneth Galbraith
Department of Economics
Harvard University
207 Littauer Center
Cambridge, Mass. 02138

Dear Ken:

Pursuing the habits of a lifetime, you raise interesting questions in your letter of December 20th. Let me answer them by giving you an account of the origins and development of Data Resources, Inc., and of its relations to Harvard. I believe this will respond to all of your questions.

(1) Origins of DRI

As you know, my professional career has largely been devoted to the application of the techniques of economics to actual problems of the U.S. economy. After my most recent period of full -time government service in 1966, my views on the economy were sought by business and financial organizations. I quickly discovered that they made little use of macro economics or econometrics. The gap between macro and micro was unbridged. They typically ignored the overall situation. Econometrics, which always looked to me to be a very practical way to establish quantitative relationships, received little use and remained an academic plaything. I had already discovered in the government that even macro-decisions were made on the basis of very crude quantitative work, without the benefit of the thirty years of methodological development of econometrics.

In mid-1967, I had the idea that the technology of the time-sharing computer provided the missing link that would make it possible to use the modern techniques to improve private and public planning on a day-to-day basis. The time-sharing technology had the potential of overcoming the mechanical hurdles of programming, data punching, batch runs, etc. which had made econometrics a slow process open only to economists of exceptional mechanical aptitude. The time-sharing technology had the potential of bringing high quality data bases to researchers of providing them with the programs that would allow them to develop individual equations and to combine these equations into simulation models, and to evaluate their “satellite” models for historical analysis, contingency analysis and micro-forecasting. Such satellite models might encompass revenues and costs of their own industries or products, the detailed composition of unemployment, regional incomes, and the tax collections of governments.

These satellite models are constructed by users, at their own remote locations, combining their own data with the national data banks on the central computers. The programs allow the construction of the models and their on-line linkage to the centrally managed national models. Once the models are built, the particular company or government can quantitatively assess its own demand, costs, production, etc., assuming a particular macro-situation. It can see its own revenue and cost outlook assuming the central forecast, or alternatively what would happen if the economy should do better or worse. The micro-implications of changes in fiscal or monetary policy are also made apparent.

Besides making the tools that are our main stock-in-trade widely useable in the actual economy, the existence of such a system could accomplish these goals:

(1) There would be a rationally decentralized structure of information flows. The national data banks would be large and accessible, but local private information would remain where it belonged — in the confidential hands of the local analysts best equipped to use it.

(2) Analysis itself would be rationally decentralized. National forecasting could be done centrally with the use of lots of resources and with the benefit of an enormous data base and model collection. Micro forecasting would be done by the user organization itself.

(3) Micro-analysis would consider macro-environments as quantitative inputs. If the macro-forecasts are better than the crude assumptions previously made, the errors in micro-decisions should be reduced.

(4) As a result, the stability of the economy should be enhanced. There should be fewer and smaller mistakes in private and public economic decisions. Some of the benefits of indicative planning are realized without the political risks.

Once the basic ideas were clear, how was it to be done? The obvious possibilities were (1) a foundation financed project at Harvard; (2) persuade the government to undertake this work; (3) go to a large company  such as a computer manufacturer or bank; or (4) organize a new, small private enterprise. After some reflection, I decided that the new, small private enterprise form was the only suitable one. A Harvard project was ruled out immediately because of the poor experience with the Harvard Economic Barometers of the late 1920’s, an episode with which I was familiar from reading the archives of The Review of Economics and Statistics. Also, the system would require considerable operating staff for the computers, data banking, service and marketing. A university is not a good employer for such a staff nor a good working environment for these functions. I knew from my government experience that such a project was beyond the capacities of public agencies, at least in the United States, and budget stringency would have made federal funding unlikely, The large company would have posed difficult personal and political questions. Further, I felt that if the scheme were successful — and I had a good deal of faith in it — it could grow and reach its full potential by generating its own revenues. Finally, the idea of ultimately supporting my family from my main activities rather than “moonlighting” was attractive.

In 1968, Mitchell, Hutchins and Company, an investment firm with whom I was consulting, found the venture capital, an amount in seven figures. Donald Marron, its President, and I then co-founded DRI. The largest fraction of the capital was provided by First Security Corporation, an asset management group under the leadership of Mr. Robert Denison, a summa graduate of Harvard College and the Business School. The Board of Directors of the company are Mr. Marron, Mr. Denison, myself, and Mr. Stanton Armour, the Chairman of the Operating Committee of Mitchell, Hutchins.

The project required managers, econometricians, programmers, and computer experts. Mitchell, Hutchins managed the organization of the company, provided the initial business background and management, recruited personnel, etc. Dr. Charles Warden, previously special assistant to several chairmen of the CEA joined the company and took on many of its managerial burdens. Later on the company was organized into three divisions, each headed by a Vice-President.

Given the complexity and ambition of the scheme, I recognized that I needed the collaboration of the very best econometricians in terms of ideas, review and quality control. Mr. Marron and I, therefore, put together a founding consulting group, consisting of Jorgenson, Nerlove, Fromm, Feldstein, Hall and Thurow. This group made major contributions in the design stage. Today, the academic consultants mainly direct policy studies that DRI has been asked to undertake by government agencies and foundations. At all stages, the largest part of the work of developing and operating the DRI system and forecast was done by full-time professional employees of the company.

To help assure the widest application of the new techniques and to be able to offer alternative model forecasts, DRI entered into an agreement with the Wharton model group directed by Lawrence Klein. We continue to collaborate with them, and the Wharton model and its forecasts are maintained on the DRI computers. Subsequently, we have entered into arrangements with the model building group at the University of Toronto and with Nikkei, the sponsors of the Japan Economic Research Center.

As for the distribution of ownership, about half of the equity is in the hands of the institutions who provided the capital. Professional employees have ownership or options on another substantial fraction of shares, and my children and I own about a fifth of the shares. The academic consulting group has about 5% of the shares, received at the time of the founding of the company. All of the stock is restricted; it is not registered with the SEC and hence not saleable. The academic consultants are paid on a per diem basis as they actually spend time. In order to give the company a better start, I did not take any pay in the first three years; last year I began to receive a modest compensation.

(2) The Status of DRI Today

On the whole, my hopes and aspirations for DRI have been realized The economic data bases are the most comprehensive in existence and their accuracy is unquestioned. The econometric models have advanced that art in certain respects. The forecasts have been good and are now followed and reported quite widely. The people — management, research economists, service consultants, data processing and programming experts, and marketing — are capable and the organization is strong. While it inevitably takes time for new concepts and techniques to gain acceptance and be widely adopted, more than half of the fifty largest industrial companies and a large fraction of the financial institutions utilize the DRI system. Every major government agency involved in macro economic policy as well as every major data producing government agency is a user of the DRI system. The research environment created by the DRI data banks, software, models and computers has proved so attractive that even organizations with considerable internal facilities find it useful to have access. DRI as an organization has no political views, though individuals associated with the company can take any position they wish.

Our system has also been used by ten universities and colleges and we have just begun to develop special services for the state governments. As DRI is becoming better known and our communications network to our computers spreads to cover a far greater number of communities, we expect that more colleges and universities will find it possible to take advantage of these research facilities.

The company reached the break-even point in the twentieth month of operation after expending the larger part of the venture capital to create the initial version of the DRI system. It is now moderately profitable and earnings are advancing rapidly. Thus far, the capitalists have earned no return of dividends or interest. They have been extraordinarily forbearing in not pressing for quick returns, preferring to let the company use all of the resources in these early years to bring the DRI concept to full fruition. The probabilities are good that the investors will be handsomely rewarded over the next few years. Having taken the risk and waited, they will have earned their return.

(3) The Relation of DRI to Harvard University

Recognizing the sensitivity of this issue from the beginning, I have made sure that Data Resources produced a flow of benefits to Harvard and that Harvard would not provide resources to DRI. The Board of Directors, heavy with Harvard alumni, formally instructed me early in our development to provide free use of the DRI system to Harvard students. Quite a few have done so, including students on my small NSF project on prices and wages. This Fall, for the first time, I have a graduate working seminar in econometric model building. Each of the seven students enrolled is building his own model, simulating it, and writing a paper. The projects include the first econometric model of Ghana, a small scale two-country model of Canada and the United States, an exercise in policy optimization using the DRI model, a study to use macro models to estimate the changing distribution of income, a study of tax incidence using translog production functions, and a model of Venezuela. If this experimental seminar is successful, a lot more can be done, of course.

In terms of relations with professors, Feldstein and Jorgenson were members of the original academic consulting group, along with professors at MIT, Chicago, Brookings and Wharton. I direct and take responsibility for the DRI forecasts, working with full -time employees. The others have focussed on policy studies, including three major studies for the Joint Economic Committee which received considerable attention. They have also done studies for the U.S. Treasury, the Ford Foundation, etc. These studies have not been a significant source of profit to the company, but they surely help to build Data Resources as an authoritative source of economic analysis and serve the public interest.

DRI has had very limited relations with the non-tenured faculty in the Harvard Economics Department. We cooperated with the Department in January 1969 to make it possible for Barry Bosworth to assume his appointment a semester early when he wished to leave the Council of Economic Advisers. He did some useful research that spring and summer, most of which reached fruition in his subsequent papers at The Brookings Institution. His half-time support was transferred to a project at Harvard after one semester. Mel Fuss collaborated in the early stages of our analysis of automobile demand sponsored by General Motors. Bill Raduchel has done some consulting in the programming area with us, but this was always was a very minor part of his activities. While it would be improper to recount the precise role of myself or Feldstein and Jorgenson in the promotion considerations of these three men, it is perfectly obvious and easily documented that there is no substantive historical issue of DRI considerations entering into Harvard appointments. Bosworth went to Brookings before his appointment came up; Fuss and Raduchel were not promoted.

Perhaps this is the point to digress on my philosophy on Harvard promotions. I believe that assistant professors should be selected on the basis of professional promise, their potential contribution to the undergraduate teaching program and whatever publication record they already possess. Promotion to associate professor should mainly be based on research accomplishments as well as teaching performance, with both prerequisites. I have always strongly felt that collaboration in the research projects of senior professors should be given no weight in non-tenured appointments because of the considerable risk that the Harvard appointment thereby becomes a recruiting device for the personnel of these projects. In my years at Harvard, I have never asked the Department to appoint anyone whose presence would be useful to me, and I never will make such a request. To the best of my knowledge, Feldstein and Jorgenson have pursued the same policy. I recommend adoption of procedures that would assure that all of us avoid such appointments.

There are more intangible relations between DRI and Harvard which are hard to assess and easy to exaggerate. If I did not possess a professional reputation which has been enhanced by my professorship here my career would have been different, and I might not have received my extraordinary opportunities of public service. As far as the development of DRI is concerned, my greatest institutional indebtedness is to the Council of Economic Advisers. It was this experience which made me appreciate the importance of accurate and quick information and of the tremendous potential of using econometrics to bridge the gap between macro- and micro-economics. As far as the relations with our private and public clients are concerned, a sophisticated group containing numerous Harvard graduates, they understand perfectly well the tremendous diversity of people and ideas present at Harvard. They know that Harvard has no institutional position on political questions or on the merits or demerits of the existing social, political or economic system. It is also clear to them that Data Resources is a totally distinct entity. I am not responsible for your views and you will not be tainted by mine.

Your final question, whether “the kind of economics that serves corporate interest will take on an exaggerated importance when some of our ablest faculty members and students are working on such problems” is a deep philosophical one which I can only attempt to answer in this way. The Harvard Economics Department has always contained individuals with widely varying concepts of their role in life and preferences in their professional activities. Compared to its historical position, the Department at this time is exceptionally heavy in abstract theory and methodology, and in social philosophy and criticism of the existing order. I represent a different point of view that has always been common in our department. It is my aim to apply economics to the country’s problems in the belief that the existing system can be made to meet the needs of the good society. The development of Data Resources is my current personal expression of this philosophy.

Sincerely yours
[signed] Otto
Otto Eckstein

OE/gc

_____________________________

Feldstein reports being a satisfied user of DRI services

HARVARD UNIVERSITY

MARTIN S. FELDSTEIN
Professor of Economics

1737 CAMBRIDGE STREET, 617
CAMBRIDGE, MASSACHUSETTS 02128

January 9, 1973

Professor J. K. Galbraith
Department of Economics
Harvard University
Littauer 207

Dear Ken:

Although I was surprised by your letter, I am happy to describe my relations with Data Resources. I have been an “economic consultant” to DRI since it was organized. I would describe both the amount of work that I have done and my financial interest as very limited. Last year, my only DRI work was a study of the problem of unemployment that I did for the Congressional Joint Economic Committee. The Committee contracted with DRI for the study. DRI provided the use of the DRI model and data bank and the special computing facilities. Professor Robert Hall of MIT, another DRI consultant, worked on the study for a few days. The study, Lowering the Permanent Rate of Unemployment, was used as the background for hearings in October and will be published by the Committee this year. I am enclosing a copy for your interest. I might also note that although the work on this for DRI is now complete, I am planning to continue on my own to do research on some of the problems that I examined in this study. A graduate student who helped me during the summer became so interested in some of the questions of labor force participation that he is considering doing his thesis on that subject.

Before last year I worked on developing the financial sector of the Data Resources model. The basic work here was building a bridge between the usual Keynesian analysis and the Fisherian theory with its emphasis on the expected rate of inflation. My work here started as direct collaboration with Otto Eckstein; we published a joint paper, “The Fundamental Determinants of the Interest Rate,” in the 1970 Review of Economics and Statistics. This research led me to consider the importance of expected inflation in all studies of the impact of interest rates; I described my work on this in “Inflation, Specification Bias, and the Impact of Interest Rates” (Journal of Political Economy, 1970). Although further work on the financial sector is now done primarily by members of the DRI full-time staff, I did some work in 1971 on extending the analysis of expectations and testing alternative econometric models of expectations. This work is described in a recent paper, “Multimarket Expectations and the Rate of Interest” with Gary Chamberlain, that has been submitted for publication.

I have described my DRI studies in such detail to give you a sense of both the substance and nature of the work. It has been scientific research on substantively and technically interesting questions of macroeconomics and macroeconomic policy. I have also found the access to the DRI facilities, particularly the macroeconomic model system and data bank, to be useful in my other research and teaching.

I cannot believe that my association with DRI could create any of the problems that you indicate in your questions 5, 6 and 7. I believe that Otto is writing to you about the specific points that you raised about DRI in your questions 1 through 4. I hope that all of this material reassures you about the relations between DRI and members of our department.

Please call me if you have any further questions,

Sincerely,
[signed] Marty
Martin S. Feldstein

MSF:JT

Enclosure

_____________________________

Galbraith to Feldstein: You did not address my concern about “problems of conflict of interest”

January 19, 1973

Professor Martin S. Feldstein
Room 617
1737 Cambridge Street

Dear Marty:

Many thanks for your detailed — and good-humored — response. I’m grateful also for the JEC Study of which Otto spoke and which I am taking to Europe for my own reading. I have taken the liberty of giving a copy of your letter to Ed Mason who, as you perhaps know, is making a study of this whole problem.

As you can guess, I am untroubled by work done directly or through DRI for the government. I am concerned about the problems of conflict of interest that seem to me to arise when a corporation which owes its esteem to members of our Department markets profit-making services to other corporations. But this is something on which I should like to reserve comment until Ed Mason has come up with his conclusions.

Yours faithfully,

John Kenneth Galbraith

JKG:mjh

_____________________________

Jorgenson: I think you are barking up the wrong tree

HARVARD UNIVERSITY
DEPARTMENT OF ECONOMICS

January 22, 1973

DALE W. JORGENSON
Professor of Economics

1737 CAMBRIDGE STREET, ROOM 510
CAMBRIDGE, MASSACHUSETTS 02138
(617) 495-4661

Temporary Address until 6/30/73:
Department of Economics
Stanford University
Stanford, California 94305

Professor John Kenneth Galbraith
Littauer 207
Harvard University
Cambridge, Massachusetts 02138

Dear Ken:

Many thanks for your letter of December 20 and your note of December 21. Let me take this occasion to thank you for the copy of your AEA Presidential Address you sent to members of the Department. It was a masterpiece of the genre and will be long remembered by its readers. I am very sorry that I was unable to attend your oral presentation at Toronto.

I share your deep concern over recent actions of the Department of Economics on non-tenure personnel, even though our views on these matters do not always coincide. In view of the strong feelings involved I found the discussion to be remarkably free of personal considerations. I hope that I have not been a party to what you describe as a less than gracious response to vour own views. If I have, I hope that you will accept my apologies.

Since your letter is addressed to Otto Eckstein, Martin Feldstein and myself, I will limit this response to my own role in DRI. I am a stockholder and consultant to DRI and have been for almost four years. In my work for DRI, I have acted as a consultant to several U.S. government agencies and to the Ford Foundation. I have had only one corporate client for my services. My main current activity for DRI is a study of energy policy for the Ford Foundation.

DRI provides a unique environment for certain types of research in applied econometrics. My current work on energy policy would be infeasible without the DRI system. The computer software, computerized data bank, and econometric forecasting system have been indispensable in modeling the energy sector and in studying the effects of economic policies related to energy. The facilities available at DRI have reduced the burden of data processing and computation for econometric model-building by several orders of magnitude.

To my mind the two most important features of the DRI system are its high quality from the scientific point of view and its ability to assimilate the results of research and to make them available for routine application. The data bank is unparalleled in scope and reliability and is constantly expanding as new sources of data are made available. The computer software package is highly sophisticated and is under continuous development as new econometric methods are designed. The forecasting system is the core of DRI’s operations and has undergone a process of improvement and extension that has continued up to the present.

The performance of the DRI system is the main source of attraction for DRI’s clients. This is certainly the case for my study of energy policy. You raise a general question about the concerns of DRI’s clients and the views of members of Harvard’s Department of Economics. In my experience there is no connection, either positive or negative. The clients of DRI are buying the services of DRI. As I have already indicated, this is a rather unusual product, unavailable at any university economics department, including Harvard’s.

On the issue of non-tenured members of the Department of Economics who are also employee-consultants of DRI, I have not employed any non-tenured members of the Department in my work for DRI, as I indicated in our telephone conversation. I find it difficult to envision circumstances in which any conflict of interest related to junior appointments could arise from my DRI association. There have been no such circumstances in the past.

I hope that these observations help to clarify the issues you raise

Yours sincerely,
[signed] Dale
Dale W. Jorgenson

DWJ: cg

cc: E. Mason, J. Dunlop, H. Rosovsky, R. Caves, J. Duesenberry, O. Eckstein, M. Feldstein

_____________________________

Galbraith back to Jorgenson: we need to avoid even the appearance of a  “conflict of interest”

Gstaad. Switzerland
February 13, 1973

Professor Dale W. Jorgenson
Department of Economies
Stanford University
Stanford, California 94305

Dear Dale:

Many thanks for your letter and for your nice comments. I hope life goes well for you at Stanford. I am writing this from Switzerland where I am on the final pages of what I intend shall be my last major effort on economics. When I get tired I propel myself across the snow and think how good the mountains in the winter would be in a world where one did not feel obliged to take exercise.

I must say that my attention after writing was shifted to yet another of our corporations of which, to my annoyance, I was unaware. It functions currently, I gather, as a subsidiary of the antitrust problems of IBM.

I do feel that there are serious problems here. Participation in the management of the Department, especially in the selection and recruitment of personnel, and in the management of a profit-making enterprise are bound to involve if not the reality of conflict of interest then the appearance of conflict. Appointments, it will be held, are influenced by what influences corporate customers or needs. This must be avoided. It is especially clear if the corporation sells such services as antitrust defense. But it is also the case if the corporation becomes large and successful —, as I would judge, DRI is certain and deservedly to be.

The proper course, as I have suggested to Ed Mason and informally to Otto, is not to deny any professor the right to participation in a profit-making enterprise. Rather it is to separate the two management roles. A man should be free to have an active ownership role in a corporation or an active position in Department management. He should not do both. This would obviate problems of conflict or seeming conflict and protect the positions of all concerned. Needless to say, I would have the same rule apply to all.

Yours faithfully,

John Kenneth Galbraith

JKG:mjh

cc: E. Mason, J. Duesenberry, O. Eckstein, M. Feldstein, R. Caves, H. Rosovsky, F. Ford

_____________________________

“Economics Dept. Reports On Faculty’s Outside Ties”
by Fran R. Schumer. Harvard Crimson, March 20, 1973

A committee in the Economics Department reported yesterday that business connections between Economics professors and outside corporations do not interfere with hiring decisions and teaching practices.

James S. Duesenberry, chairman of the three-man committee, said yesterday that business ties do not impose a conservative bias on the Department’s hiring practices and do not limit the faculty’s teaching time.

Complaints

The committee’s investigation was prompted by complaints raised last term by John Kenneth Galbraith, Warburg Professor of Economics.

Galbraith attributed the Department’s “conservative hiring practices” to faculty members’ ties with business firms. “The fact that the Department sells its services to American business firms biases its administrative decisions,” Galbraith said.

Despite the committee’s negative findings, Otto Eckstein, professor of Economics and president of Data Resources Inc., a consulting firm, has requested to go on half-time status at Harvard, effective September 1.

Eckstein said yesterday that his decision resulted from Galbraith’s complaints and a new rule prohibiting professors from spending more than one day a week consulting. The rule, previously implicit, was formally written into University law this year.

Galbraith voiced objections to faculty members’ business ties several weeks after the Department’s decision last December not to rehire two radical economists.

At that time, Galbraith told Duesenberry that “business ties necessarily impair the faculty’s ability to impartially judge economists, especially radical economists.”

Galbraith also complained that the Department’s decision last December not to promote William J. Raduchel, assistant professor of Economics, was based on the quality of Raduchel’s work for an outside Resources had little influence on the consulting firm and not on his research and teaching abilities in the Department.

Raduchel is a consultant for Data Resources Inc. and is also a sectionman for Galbraith’s course, Social Science 134, “The Modern Society.”

The committee, composed of Duesenberry, Arthur Smithies, Ropes Professor of Political Economy, and Richard E. Caves, Stone Professor of International Trade, reported last January that Raduchel’s work for Data Resources had no influence on the Department’s decision.

The committee also reported that outside ties do not prejudice the Department’s hiring decisions and do not interfere with normal administrative functioning.

The committee reported its findings only to Duesenberry, the chairman of the Economics Department. Committee members refused to comment on how they investigated the problem.

Duesenberry attributed Galbraith’s objections to the Department’s decision not to promote Raduchel. “Galbraith is annoyed because his boy didn’t get promoted,” he said.

Raduchel told The Crimson last month that he was satisfied with the Department’s decision not to promote him. He said that the decision had “nothing to do with my connection to Data Resources, and was based on my academic work.”

Eckstein agreed with Duesenberry’s conclusion that Raduchel’s work at Data Resources had little influence on the Department’s decision.

Explaining his own position at Data Resources Inc. Eckstein said that his case is no different than that of other faculty members who do consulting work.

Currently, at least three senior faculty members and one junior faculty members do consulting work at Data Resources.

Eckstein described consulting work an inevitable product of Harvard’s hiring policies. “Harvard naturally attracts people who get involved in the outside world,” he explained.

He said that he has a “clear conscience” about the work he is doing at Harvard.

_____________________________

Galbraith to Chairman Duesenberry:

Gstaad, Switzerland
March 27, 1973

Professor James S. Duesenberry
Littauer M-8

Dear Jim:

Herewith some good-humored thoughts on our final talk the other day about our corporate affiliates. As you request, I will now leave the problem to the President, Steiner and whomever.

  1. Although both you and Henry Rosovsky had earlier expressed discomfort about our corporation and some action now seems in prospect, you say I’m severely viewed for raising the issue. Isn’t this a little hard? The important thing, I suggest, is to get things right. However, although given my sensitive soul it has been difficult, I have steeled myself over the years to the idea of not being universally loved.
  2. You say that the bias from combining business entrepreneurship with professorial activities in the eye of some of our colleagues is not greater than that deriving from my (or Marc Roberts’) support of George McGovern. I somehow doubt that the faculty would agree. There is indication of difference, I think, in the way one reacts. I do not find myself shrinking especially from identification even with anything now so widely condemned as the McGovern campaign. I detect a certain desire to avoid public discussion of our corporations.
  3. In keeping with the desire for reticence, I told Ed Mason I wouldn’t talk with the press. The Crimson tells me that you have explained that I raised the issue only out of pique over the non-promotion of Raduchel. Isn’t this a bit one-sided? However, beyond denying any such deeply unworthy motive, I’ll stick to my agreement, always reserving the right of self-defense.
  4. As to my motives, so far as I can judge them, I did feel that Raduchel got judged on his corporate work, while — as Smithies and I both complained — there was no consultation with those who best knew about his teaching. His teaching has been very good. I suggest that we are always in favor of improving undergraduate teaching in principle but not in practice. Also I do not agree that he was unpromotable. He has a lively, resourceful mind and has worked hard for the University and the students. I think him far, far better than the dull technicians we do carry to the top of our nontenured ranks, possibly even beyond.
  5. But, as I probe my soul for the purest available motive, it was not Raduchel. I simply think that, when a professor speaks or acts on a promotion, we should know that he is doing it as a professor and not as a businessman.
  6. I had thought that the separation of our business arrangements from the Department management might be a solution, with the proposed withdrawal of voting rights from the aged as a precedent. This, I gather, will not wash, so I subside. As Ed Mason tactfully hints, I’ve had enough lost causes for one year.

I do have one final thought. In accordance with the well-known tendencies of free enterprise at this level, one day one of these corporations is going to go down with a ghastly smash. It will then be found, in its days of desperation or before, to have engaged in some very greasy legal operations. The Department and the University will be held by the papers to have a contingent liability. It will be hard to preserve reticence then. It would have been better to have taken preventative action now.

Conforming to your wish that I restrict communications on this subject, I’m not circulating this letter. But would it trouble you If I added it discreetly to the file in the President’s office? Do let me know.

Yours faithfully,

John Kenneth Galbraith

JKG:mjh

Source: John F. Kennedy Presidential Library. John Kenneth Galbraith Personal Papers. Series 5 Harvard University File, 1949-1990. Box 526. Folder “Harvard Dept. of Economics. Discussion of appointments, outside interests and reorganization, 1972-1973 (1 of 2)”.

Image Sources: John Kenneth Galbraith (1978), Harvard University Archives; Otto Eckstein (April 1969), Harvard University Archives; Martin Feldstein (ca. 1974), Newton Free Library, Digital Commonwealth, Massachusetts Collections Online; Dale Jorgenson. (1968). John Simon Guggenheim Memorial Foundation.

Categories
Columbia Curriculum Economics Programs Harvard Yale

Columbia. Plans for Graduate Classes and Creation of a School for Civil Service Preparation. Burgess, 1880

 

In the beginning was the pitch. On June 7, 1880 the Board of Trustees of Columbia College passed resolutions to establish the School of Political Science (within which the field of political economy was embedded). The School of Political Science would open for students beginning with the 1880-1881 academic year. The pitch and plan for a “School of Preparation for the Civil Service” of February 1880 by the School of Political Science founder John W. Burgess  is transcribed below.

From this document is clear that in Burgess’  initial vision the principal mission of the future Faculty of Political Science would be to “prepare young men for the duties of public life”. So one can think of the ancestor faculty from which today’s department of economics at Columbia University descended was actually a School of Public Policy in which History, Law, and Philosophy constituted the supporting disciplinary pillars. 

__________________________

OUTLINE OF A PLAN
FOR THE
INSTRUCTION of GRADUATE CLASSES
FOR THE
EXTENSION OF THE ELECTIVE SYSTEM OF STUDY
IN THE
UNDERGRADUATE DEPARTMENT.
AND FOR THE CREATION OF
A SCHOOL OF PREPARATION
FOR THE
CIVIL SERVICE

Macgowan & Slipper, Printers, 30 Beekman Street, New York.

__________________________

COLUMBIA COLLEGE

GRADUATE INSTRUCTION, AND THE EXTENSION OF
THE ELECTIVE SYSTEM

To the Committee of Trustees of Columbia College on the Course and Statutes:

GENTLEMEN: The circumstances of the College are at length such as to make it possible to provide for giving instruction of a more advanced character than is required in the undergraduate classes. This is an object which many members of the Board of Trustees have long esteemed to be desirable, and of which the accomplishment has been regarded as only a question of time. Educational institutions of a nominally superior order are numerous enough in our country, but those which possess any just claim to be considered fountains of really high learning are as yet comparatively few. The country has, nevertheless, need of such institutions, as is made evident by the yearly increasing numbers of American youth who resort for the completion of their education to the Universities of the Old World. Such institutions will accordingly be erected, but it is hardly to be expected that they will be built from the ground up, on new and independent foundations. They will rather be developed out of existing institutions already well established, well endowed, and possessed of instrumentalities in actual operation, adequate, with some modification of the mode of their application, to accomplish the new and higher work proposed, without interfering with their present usefulness, or materially increasing the burden of their maintenance. The number of educational institutions in our country capable of such development is not at present great, nor is it important that it should be. It is far from being desirable, in the interest of the higher education itself, that universities (employing the term in the commonly accepted sense) should be numerous relatively either to the population or to the schools of lower grade. It is, on the other hand, vastly preferable that they should be few, strong, and largely attended, rather than many and, as an almost necessary consequence, ill-supported and feeble.

Circumstances already very distinctly indicate certain of the educational institutions of our country as destined inevitably to occupy this higher grade. There are others, doubtless, which will yet be added to the number, whose destiny is at present less clearly manifest; but as to these few there can be no possibility of mistake. Among the conditions which will aid in determining the future of these institutions is, of course, the present possession of adequate financial strength; but another, which is of nearly equal importance, is geographical position. Many reasons conspire to make the city of New York the fittest place on this continent for the growth of an educational institution of the highest order. It is not merely because of its superior population or its superior wealth that it possesses this advantage. It is because here are gradually concentrating themselves, in the most marked degree, all the most important accessories to mental cultures, all that most strikingly illustrates the triumphs of the human intellect in science, in letters, and in the arts of civilized life. And it is because in the ebb and flow of the vast human ocean which overspreads the continent, this city is the point toward which the currents of the population are continually tending, or from which they are retiring; and this is therefore, the point in which knowledge may be most conveniently sought, and from which it may be most easily diffused.

Columbia College occupies this very favorable position. It has already an honorable and well-earned reputation. It has around it a large body of Alumni interested in its prosperity, zealous for the promotion of its usefulness, and occupying, many of them, positions of influence in the social and political world. It has gathered together the most important of the instrumentalities necessary to the efficiency of instruction, or to the aid of literary research or scientific investigation. It has built up professional schools which, in there several specialties, take easily precedence of all others of their class in the United States. Its several faculties embrace men accomplished as scholars, thoroughly versed in the various departments of science, and profoundly learned in history, philosophy, and the law. What it needs to give it the character of a true university is to open its doors to aspirants for knowledge, who are seeking, not, as at present in the undergraduate department, knowledge in its mere rudiments, nor, as in the professional schools, knowledge for its immediately useful applications, but knowledge in the largest sense—knowledge pursued for its own sake, or to qualify the learner to contribute on his own part to the intellectual progress of the race.

To aspirants of this class opportunities may be offered for instruction in a great variety of subjects, without any addition, not demanded in the interest of the undergraduate department alone, to the present educational staff. To this end it is only necessary to adopt the simple expedient of distribution the hours of instruction over a larger portion of the day than they now occupy. According to the existing arrangement, all instruction is given within the limited space of three hours daily, and these are filled up with the exercises of the undergraduate classes. It is true, that within this limit it frequently happens that officers have unemployed hours; but these are not so distributed as to permit the arrangement of a working system of graduate instruction. To make such a system practicable, it is necessary that the subjects which a student may desire to combine should be taught at different hours. And this condition cannot be fulfilled without scattering the exercises over the greater part of the day. This is what is done in the School of Mines, in which all the hours from nine or ten o’clock in the morning till four or five in the afternoon are made available in one form or another for purposes of instruction.

But this expedient, while it provides that instruction shall be going on at every hour in one department or another, involves to the individual student the consequence that, between the hours in which he is engaged in class, there will occur other hours when he is without any scholastic exercise to employ him. During these hours it is proper that he should be occupied in study; and so he might be presumed to be were his residence in every instance within easy distance from the College. The institutions which provide lodgings for their students upon their own grounds find no difficulty in this matter. At Harvard University, for instance, exercises occur at every hour from nine till five; but the students, when not under instruction, are expected to study in their rooms. Since students with us have no private rooms, and are, in general, too distant from their homes to study there, the distribution of time proposed would hardly be judicious or just unless provision should be at the same time made for usefully employing the hours intervening between class duties. In the School of Mines abundant occupation for these hours is found in drawing, laboratory work, or practice in the use of tools and instruments. For the undergraduates it would be necessary to provide study rooms, to be used by students in common, by setting apart for the purpose some portion of the buildings not as yet otherwise assigned. The upper floor of the recently-erected building is entirely suitable to this object, and is quite sufficient. It is not needed for any present college uses, nor for any likely to arise so long as the old building continues to stand; and it is very evident that that building cannot be demolished until another shall have been erected somewhere else. Its demolition would necessitate the provision of other accommodations for the great variety of operations now going on in it; and though such might be found for the physical department in the new building, yet, for the Herbarium, for the classes in French, German, Mechanical Engineering, and Descriptive Geometry, for the operations of air and water analysis, for the library overflow, the gymnasium, etc., etc., no adequate space could be found there disposable. The future wants of the institution, for which the remaining floor of the new building is presumably held in reserve, will, therefore, undoubtedly be provided for in some manner not likely to interfere with any present disposition which may be made of that floor; and hence the devotion of that now unoccupied space to the uses of a study room cannot on that account be regarded as objectionable.

Supposing this arrangement to be adopted, it is easy, without adding to the number of our instructors, to plan a system of instruction for graduates which shall embrace reading or lecture courses in the Classics, English Literature, Anglo-Saxon, Philosophy and the History of Philosophy, Physiological Psychology, Political Science, Public Economy, History, the Higher Mathematics, Mechanics, Astronomy, Physics, Chemistry, Geology, and certain branches of Natural History, to occupy, on an average, not less than two hours weekly each. There can be no doubt that, when this system shall have been fully inaugurated, the number of students attracted by it will so far add to the revenues of the College as to justify the appointment of additional instructors on subjects not included in the above enumeration, such as Foreign Literature, Hebrew, Oriental Literature, Comparative Philology, Ethnology, Archæology, Natural History in its various ramifications, Animal and Vegetable Physiology, Methods of Research in Physics and Chemistry, Physical Astronomy, Architecture and the Fine Arts, and any others which may be necessary to make the institution a school of universal knowledge.

For the purpose of illustrating the practicability of carrying the proposed plan into effect without interfering with the undergraduate course already in operation, an ideal scheme of attendance is annexed to this communication, showing a distribution of the subjects of instruction to the hours of each day throughout the week, by means of which the convenience of instructors, as well of students, both graduate and undergraduate, may be satisfactorily provided for.

ENLARGEMENT OF THE ELECTIVE SYSTEM FOR UNDERGRADUATES.

An advantage incidental to the distribution of the hours of instruction over the entire day will be the opportunity it affords for giving to the students of the undergraduate department a greater liberty of option than they at present enjoy in the selection of their studies. It appears to the undersigned that the collegiate course for the student of our day cannot properly he made so severely a course of mere mental discipline as it was judicious to make it in the last century, or even during the earlier years of the present. The average age of our graduates is that of fully grown men. The age of the applicant for admission, a century ago, ranged from ten to fourteen years. The four years of college life embraced at that time the period of gradual mental as well as of physical development and growth; and it was eminently proper that the studies enforced upon the learner should be adapted rather to train the faculties than to inform the mind. At present this period is spent mainly in the preparatory schools, and it is in them that this work of mental gymnastics should be chiefly carried on. The end of college instruction is not merely to discipline, but also to inform; and since men differ no less in their mental than in their personal characteristics, it follows that different individuals do not acquire the same kind of knowledge with equal facility, nor are they equally profited by it. This provision of nature is one of great and beneficial importance to the progress of the race; for inasmuch as it is impossible that any one shall be proficient in all departments of human knowledge, the fact that each seeks spontaneously some special field insures the certainty that every such field will be energetically explored.

It seems, therefore, to be proper that at that period of comparative maturity at which young men begin to be conscious of their fitness for some particular pursuit or vocation, and desirous to acquire the knowledge most likely to be advantageous in following it, they should be allowed, to a pretty large extent, to select their subjects of study from among those which are in harmony with their tastes and aspirations. That public opinion in the social and the educational world recognizes this propriety is manifest in the flourishing condition of the institutions in which the elective system of study has been largely developed, and in the rapidity with which those institutions have grown in popular favor. In our own College the introduction of this system on an extended scale has been hitherto impracticable, for the same reasons which have made it impossible to provide for the instruction of graduates; yet the results of the limited trial of it which we have made have been altogether favorable.

The introduction of the elective system, however, in its largest latitude, does not by any means imply the necessity of discarding the system which at present exists. If there should be any who prefer still to cling to traditions, they may enter themselves from the beginning for the course of study prescribed for the degree of Bachelor of Arts in our statutes as they stand, and follow the same undeviatingly to the end. Degrees in Arts may be reserved to be conferred on such only; while those who prefer the optional course may be graduated as Bachelors of Letters or Bachelors of Science, according to the character of the studies which they chiefly pursue.

The adoption of a scheme of study largely elective would not necessarily require an increase in the number of instructors employed. As it is true, however, that, in such a system, the modern languages ought pretty fully to enter, it would be advisable to add to our present corps, a tutor in Italian and a tutor in Spanish. As to the French, arrangements could probably be made by which the instructor in that language in the School of Mines would teach also in the College; and for the German, we have already a professor who has hitherto given instruction in his proper department only under circumstances of serious embarrassment, and who would find the proposed change advantageous to his usefulness.

An ideal scheme of attendance accompanying this communication shows in what manner the exercises may be arranged so that students pursuing elective courses may attend along with candidates for degrees in Arts, so far as the subjects of study of these two classes of students are identical. Another appended paper shows the variety of particular courses of study which may be offered in the several departments indicated only by general titles in the scheme of attendance. In this latter paper are embraced some titles which do not fall within the special province of any of our present instructors; such as Hebrew Sanskrit, the Evidences of Religion, Anatomy, Physiology, and the principles of the Common Law.

In regard to the Evidences, there is no doubt that, under the voluntary system, the Trustees would think it advisable to reestablish the course; since with the abandonment of the principle of compulsion would disappear the reasons which have led to its discontinuance heretofore.

As to Anatomy and Physiology, arrangements could be made not involving any considerable expense, with Professors of the Medical School, to give annual courses adapted to the wants of the general learner, of great interest and value, though not extending to more than ten or twelve lectures each. A brief course of similar extent, upon the principles of the Common Law, could be given by our principal Law professor, embracing information highly important to educated men, but not now easily obtain able except in professional schools. Such a course used to be given to the Seniors at Yale College during the lifetime of the late Judge Daggett, which was always fully attended, though attendance was merely voluntary.

As to Sanskrit, a language occupying every year more and more the attention of scholars everywhere, there would be very little difficulty. Some of our own recent Alumni have already honorably distinguished themselves in this interesting study. One of our Fellows in Letters of the Class of 1878, now pursuing his studies at the University at Leipzig, had made himself by his own private and independent efforts a proficient in Sanskrit literature before his graduation, and since entering upon the course at Leipzig has received signal marks of approval and consideration from the professor in that department there. Another, of the Class of 1875, after completing his course of study abroad, has returned to this country bringing flattering testimonials to his attainments in general scholarship, and especially as to his proficiency in this particular branch. This young man, or some other similarly qualified, could be appointed to fill one of the new tutorships which it will presently become necessary to create in Greek or Latin in our College, and could be engaged at the same time to take charge of the instruction of any class which might be formed in Sanskrit. Considering that the condition of things is such in College at present as to make it impossible any longer to defer increasing the number of subordinate officers, it may be regarded as a happy concurrence of circumstances which enables us by the same act to provide a competent instructor in a subject in which as yet in this country the proficients are few.

All the subjects, however, for which instructors cannot be found among the members of our present Faculties, unless the modern languages be excepted, may be omitted, if necessary, from our scheme, till the success of the measure shall permit them to be provided for without adding to the burdens of the treasury.

PREPARATION OF YOUNG MEN FOR THE CIVIL SERVICE.

Since the passage of the resolution in regard to graduate classes, now before the Committee, a plan has been proposed for establishing, in addition to the special schools connected with the College already in existence, a new one of an original character, designed to meet what is believed to be a want of the present time, by preparing young men to engage intelligently in the service of the government. Though as yet in our country the civil and consular service has not been securely placed beyond the control of influences purely political, the popular sentiment manifests itself more and more strongly every year in favor of such a separation, and enough has been already accomplished to make it evident that, at least in the lower ranks of this service, proper qualification and the personal merit of the aspirant are likely hereafter to carry with them more weight in the competition for office than political favor. Even, moreover, should the so-called Civil Service reform receive a check, the course of instruction which it is proposed to give in the new school would still prove attractive to young men who desire to fit themselves to deal with questions of public interest, whether in professional life, or as politicians, or as managers of public journals. A school of a character similar to this has for several years been in successful operation in Paris, and counts among its teachers some of the most eminent publicists of France.

The proposition referred to proceeds from our Professor of Political Science, and is set forth in its details in a letter addressed by him to the undersigned, which is given below. As the proposed school involves the question of both graduate and undergraduate instruction, its consideration falls within the scope of the resolution referred to the Committee. An ideal scheme of attendance hereto appended shows in what manner the exercises of this course may be combined with those of the graduate and undergraduate classes, so that each may be accessible, if desired, to the students of the others.

In order that this project may be carried into effect, it would be necessary to appoint, within the next two years, two additional instructors, at salaries of $2,500 each. Within the same period the number of students entering for the course might reasonably be computed at not less than fifty, and would probably be greater. It would conduce very much to the success of the undertaking that the Law School should be removed from its present situation and united with the other departments upon the same ground, though that need not be regarded as in dispensable.

Following the letter of Professor Burgess below, will be found, marked A, a succinct résumé of the essential features of the plan herein recommended, and also of that of the proposed School of Preparation for the Civil Service, marked B. Also, marked C, an estimate of the probable effect upon revenue and expenditure of the adoption of these projects; and finally, an enumeration, marked D, of the subjects of the several courses of instruction which the proposed plan will embrace. The hypothetical schemes of daily attendance referred to above are presented separately.

All which is respectfully submitted,

F. A. P. BARNARD, President.

COLUMBIA COLLEGE, February 25, 1880.

__________________________

LETTER OF PROFESSOR BURGESS TO THE PRESIDENT.

323 West 57th St., Feb. 20th, 1880.

My Dear Dr. Barnard:

It seems to me evident that the time has now fairly arrived, both in the history of this nation and of this University, when a decisive step forward in the development of the political sciences is positively and specially demanded.

In the history of the nation it is so, not only because the Republic has now reached those mighty proportions demanding the finest training, as well as the finest talent, for the successful management of its affairs, but because the Government itself has recognized this fact, and in its Civil Service reforms, which, I think, are now fairly planted and destined, under the proper influences, to a noble growth, has opened the way for an honorable career to the young men of the nation in the governmental service, which may be successfully pursued by the best intelligence, skill, and fidelity, offering itself without any reference to political influence or patronage.

In the history of the University it is so, not only because it is the bounden duty of a university, worthy the name, to teach all that has been gathered by the world’s experience in this as well as all other departments of superior knowledge, and to add continually thereto, but because, also, of its metropolitan situation, which fits it better than any other in the nation, both to place its students in immediate connection with the Civil Service examinations, so far as they now exist, and to exert its influence with greatest efficacy for the extension of the same throughout every branch of that service, as the indispensable condition of appointment to governmental office, and because I think I may assert that the foundation is now already fairly laid in our University for the development which I now propose.

The elements of the plan which I would suggest are, in outline, as follows:

I

The course of study shall extend over a period of three years, and comprise the following subjects:

FIRST YEAR.— The History of Philosophy; The History of the Literature of the Political Sciences; The General Constitutional History of Europe; The Special Constitutional History of England and of the United States.

SECOND YEAR.— The Roman Law—the general principles of the jurisprudence of the existing codes derived therefrom; The Comparative Constitutional Law of the Principal European States and of the United States; The Comparative Constitutional Law of the Different Commonwealths of the American Union.

THIRD YEAR.— History of Diplomacy; International Law; Systems of Administration, both of the Nation and the Commonwealths in the American Union; The Comparison of the Administrative System of the United States with those of the principal European States; Political Economy in all its Branches, and Statistics.

II.

The requirements for admission to this department shall be:

FIRST.— The completion, successfully, of the Junior Year in any regular college of the United States; or—

SECOND.—The passing of a successful examination upon all  the studies of the first three years of the Academic Department of this University; and—

THIRDLY. — Including therein, in either case, a fair reading knowledge of the French and German languages.

III.

The degree conferred, upon the completion of all the courses and after successful examination therein, shall be that of A.M., Ph.D.; and if, in addition to the courses in this department, the student shall have received the degree of LL.B., either from the Law School of this University or from any other School of Municipal Law in good standing, the degree shall be that of D.C.L. For the students of a single year’s standing the degree will be that of A.B.

IV.

The fee for instruction in this department shall be the same in amount as is paid by undergraduates. But members of the Senior Class in the Academic Department of the University may attend the courses of the first year, and members of the School of Law may attend any or all of the courses free.

Finally, my dear Doctor, I would add that, in my opinion, this plan can be now realized without much, if any, additional expense to the Treasury of the University. When in complete running order it will require the appointment of two assistants to the Professor now in charge of the department — one for the courses in Roman Law and its branches, the other for the courses upon the administrative systems of Europe and the United States. I am confident that two fit persons, both graduates of the Law Department of this University, and now about finishing their courses of special study upon these topics in European universities — the one at Berlin and the other at the Ecole Libre des Sciences Politiques in Paris–can be secured at salaries of $2,500 each per annum; and if one of these should be appointed, his office and salary to take effect from October 1st, 1881, and the other from October 1st, 1882, I fully believe that the receipts from the fees of students in the department would be sufficient to cover one, if not both, of these salaries, and in five years from this time would be a source of revenue to the Treasury. I am informed that in the city of New York  alone fifty or more vacancies occur every year in the Civil Service of the United States, from death, resignation, or inability to serve; and that, through the present method of Civil Service examinations, fifty or more appointments are therefore annually made to offices having salaries of from $1,200 to $5,000; and I feel assured that the institution of learning which shall first seize the opportunity for establishing among its departments a school of training for the Civil Service of the National Government, will not only increase its own prestige and be a national benefactor, but will also reap no mean financial reward therefrom.

Trusting that these suggestions may meet your favor and support,

I am, as ever, your most obedient servant,

JOHN W. BURGESS.

President F. A. P. BARNARD, LL.D.

*  *  *  *  *  *  *  *  *  *  *  *  *  *

(A.)

PLAN FOR THE INSTRUCTION OF GRADUATE CLASSES, AND FOR THE EXTENSION OR THE ELECTIVE SYSTEM IN THE UNDERGRADUATE DEPARTMENT.

1.—Scholastic exercises to be distributed over all the hours of the day from 9½ A.M. till 4 P.M. from Monday to Friday inclusive, and from 9½ till 11 on Saturday.

2.—Every student to be under instruction in class for three hours each day till Saturday, and for two hours on Saturday.

3.—Students, when not under class instruction, to be occupied in study in rooms set apart for the purpose.

4.—A recess of half an hour to be allowed at mid-day for lunch.

DEGREES.

1.—If no modification of the course of instruction or no enlargement of its scope be made, the only degree conferred at the close of the four years’ course to be the degree of Bachelor of Arts.

2.—Should the course be enlarged and liberty of option be increased, the degrees of Bachelor of Letters and Bachelor of Science to be also given.

3.—The course leading to the degree in Arts to remain unchanged, consisting as at present of a prescribed curriculum up to the close of the Junior year, and of a partially elective course during the Senior.

4.—For degrees in Letters and Science, the course to be identical with the course in Arts during the Freshman year, and to embrace as obligatory during the subsequent years all the subjects of that course in the departments of English Literature, History, and Political Science, occupying six hours in the Sophomore year per week; five hours in the Junior, and four hours in the Senior. The remaining hours up to seventeen per week to be occupied by elective studies. The degree in Letters to be conferred on those who choose as electives the Languages, Ancient or Modern, Literature, Psychology, and Philosophy; and the degree in Science on those who choose principally the Mathematics, Physics, Mechanics, Astronomy, and Chemistry. Specific regulations in regard to this subject to be made by the Faculty, with the approval of the Trustees.

INSTRUCTION TO GRADUATES

1.—Instruction to be given to graduates of this and of other Colleges who desire to fit themselves for literary or scientific work not professional; as for authorship, historical or philological research, scientific investigation, etc.

2.—Special courses of instruction adapted to the wants of such graduates to be arranged in the departments of Greek, Latin, Mathematics, Physics, Astronomy, Chemistry, Philosophy, Political Science, Natural History, and the Modern Languages. Only such courses to be attempted at present as can be conducted without materially increasing the staff of instructors.

3.—Notice to be publicly given before the close of the present academic year of the purpose to institute courses for graduates, with specification of such as may be commenced in October, 1880.

*  *  *  *  *  *  *  *  *  *  *  *  *  *

(B.)

SCHOOL OF PREPARATION FOR THE CIVIL SERVICE, DIPLOMACY, AND THE EDITORIAL PROFESSION.

1.—A School to be instituted with a definitely prescribed curriculum of instruction, designed to prepare young men for public life, whether in the Civil Service at home or abroad, or in

the legislatures of the States or of the nation; and also to fit young men for the duties and responsibilities of public journalists.

2.—Students of our own College, or of other Colleges in good standing, who have completed with credit the undergraduate course up to the close of the Junior year, to be allowed to enter themselves as students in this School.

3.—The course of instruction to extend over three years.

4.—At the close of the first year, the student, on passing an approved examination, to be entitled to the degree of Bachelor of Philosophy.

5.—At the end of the course, the student who passes all his examinations with approval, to be recommended to the Trustees for the degree of Doctor of Philosophy.

6.—Students of this School to be permitted to attend such exercises of the Graduate or Undergraduate department as their time will allow, the approval of the Professors in the School being first obtained.

*  *  *  *  *  *  *  *  *  *  *  *  *  *

(C.)

PROBABLE EFFECT ON REVENUE AND EXPENDITURE OF THE ADOPTION OF THE PROPOSED SCHEME.

I.—EXPENDITURE.

It must, in the first place, be considered that the exigencies of the College will require, whether this plan be adopted or not, the appointment, in anticipation of the opening of the ensuing academic year, of an additional tutor in Greek, of one in Latin, and of one also in Mathematics. This will involve an increase of annual expenditure not to be charged to the present project, of $3.600.

For the supervision of students in their study rooms it will be expedient to appoint two Proctors at salaries of one thousand dollars each per annum. It is probable that this amount might be reduced by arranging that Tutors may act as Proctors.

The department of Modern Languages, in regard to which our College is now deplorably deficient, would require the appointment of an instructor in Italian, and an instructor in Spanish, who might probably be engaged for about $1,300 per annum each. The instructor in French of the School of Mines would probably be willing, for a slight increase of compensation, to take charge of the French classes in College also.

These comprise all the additions to our annual outlay which it seems to be necessary at present to make. Summed up they are as follows:

Two Superintendents of study rooms $2,000
Two Instructors in Modern Languages $3,000
Increase of salary of Instructor in French $500
Total $5,500

II.—REVENUE.

The tuition fee now charged in the Undergraduate Department is less by fifty dollars per annum than that which is charged at Harvard College or at the University of Pennsylvania. It seems fitting that, in so largely increasing the advantages offered to students here, we should make our charges equal at least to what they are elsewhere. To increase the fee from $100 to $150 per annum would not diminish the number of our undergraduate students, and hence, even should the new attractions fail to add to our numbers, the effect of this measure would be to add $10,000 to our annual revenue.

It may, however, be safely estimated that, before two years shall have passed under the new system, we shall have at least twenty graduate students, and that the number of undergraduates will be more than equally increased. The following, then, are the items of the probable increase of revenue, viz.:

From increase of tuition fee $10,000
From graduate students (say 20) $3,000
From increased number of undergraduates (say 20 also) $3,000
Total $16,000

The probability, therefore, is that the adoption of the plan, if accompanied as it should be with a simultaneous increase of the tuition fee, will tend to enlarge rather than diminish the annual balance in the treasury.

If, now, we consider the further effect of the establishment of the proposed new school of preparation for the Civil Service, we shall have to add to the annual outlay the amount of the salaries of two Instructors or Adjunct Professors, one in Roman Law and one in Administrative Law, each of $2,500. On the other hand, we may safely count on attracting to this school, in the course of the next two or three years, at least fifty students—a number likely afterward largely and rapidly to increase. We may assume, then, at the end of the second year, something like the following, viz.:

I.—EXPENDITURE.

Total as above $5,500
Salaries of two instructors $5,000
Total $10,500

 

II. REVENUE.

Total as above $16,000
Fees of fifty students $7,500
Total $23,500

Leaving a probable balance in favor of the treasury of $13,000 per annum.

*  *  *  *  *  *  *  *  *  *  *  *  *  *

(D.)

COURSES OF STUDY.

The subjects or authors herein enumerated as for undergraduates, are in general those taught during the academic year 1878-9. In the classical departments these are liable to be changed from year to year. Should the proposed plan be adopted, every course not marked as obligatory will be open for election to all undergraduates above the Freshman grade, without regard to year.

DEPARTMENT OF GREEK.

1.— Iliad or Odyssey, one term 3 hours per week
Freshmen,
Obligatory.
2.— Herodotus, one term
3.—The Odyssey 2 hours per week,
Freshmen,
Voluntary.
4.—Aristophanes and minor poets 2 hours,
Sophomores,
Voluntary.
5.—Euripides, one term 3 hours per week,
Sophomores,
Elective.
6.—The Memorabilia, one term
7.—Sophocles, one term 3 hours per week,
Juniors,
Elective
8.—Plato, one term
9.—Æschylus, one term 3 hours per week,
Seniors,
Elective
10.—Demosthenes, one term
11.—(To be announced annually in advance) 2 hours per week,
Graduates.
12.—(To be announced annually in advance) 1 hour per week,
Graduates.

 

DEPARTMENT OF LATIN.

1.—Pliny 3 hours per week,
Freshmen,
Obligatory.
2.—Horace, one term 3 hours per week,
Sophomores,
Elective.
3.—Livy, one term
4.—Juvenal, one term 3 hours per week,
Juniors,
Elective
5.—Cicero De Officiis, one term
6.—Terence, one term 2 hours per week,
Seniors,
Elective
7.—Catullus, one term
8.—(To be announced annually in advance) 2 hours per week,
Graduates,
Elective.
9.—(To be announced annually in advance) 1 hour per week,
Graduates,
Elective.

 

DEPARTMENT OF MATHEMATICS.

1.—Geometry, one term 5 hours per week,
Freshmen,
Obligatory.
2.—Algebra, one term
3.—Curves of Second Order, one term 3 hours per week,
Sophomores,
Elective.
4.—Trigonometry, Mensuration and Surveying, one term
5.—Differential and Integral Calculus 2 hours per week,
Seniors,
Elective
6.—Calculus of Variations 2 hours per week,
Graduates.
7.—Quaternions 2 hours per week,
Graduates.

 

DEPARTMENT OF MECHANICS AND ASTRONOMY.

1—Analytical Geometry, one term 3 hours per week,
Juniors,
Elective.
2.—Mechanics, one term
3.—Astronomy 3 hours per week,
Seniors,
Elective.
4.—Practical Astronomy 2 hours per week,
Graduates.
5.—Spherical Astronomy 2 hours per week,
Graduates.
6.—Analytic Mechanics 2 hours per week,
Graduates.
7.—Physical Astronomy 2 hours per week,
Graduates.

 

DEPARTMENT OF PHYSICS.

1.—Elementary Physics 2 hours per week,
Freshmen,
Obligatory.
2.—Heat and Electricity 2 hours per week,
Juniors,
Elective.
3.—Optics and Acoustics 3 hours per week,
Seniors,
Elective.
4.—Mathematical Physics 2 hours per week,
Graduates.
5.—Methods of Physical Research 2 hours per week,
Graduates.

 

DEPARTMENT OF CHEMISTRY.

1.—Elementary Chemistry 1 hour per week,
Freshmen,
Obligatory.
2.—Theoretic Chemistry 2 hours per week,
Sophomores or Seniors,
Elective.
3.—Applied Chemistry 2 hours per week,
Juniors,
Elective.
4.—Organic Chemistry 2 hours per week,
Graduates.
5.—Methods of Chemical Research 2 hours per week,
Graduates.

 

DEPARTMENT OF PHILOSOPHY.

1.—Logic 2 hours per week,
Juniors,
Obligatory.
2.—Psychology 2 hours per week,
Seniors,
Obligatory.
3.—Philosophy 3 hours per week,
Seniors,
Elective.
4.—Physiological Psychology 1 hour per week,
Graduates.
5.—History of Philosophy 2 hours per week,
Graduates.

 

DEPARTMENT OF ENGLISH LITERATURE.

1.—Rhetoric 3 hours per week,
Freshmen,
Obligatory.
2.—History of Literature 2 hours per week,
Sophomores,
Obligatory.
3.—English Classics 1 hour per week,
Sophomores,
Obligatory.
4.—Logic 2 hours per week,
Juniors,
Obligatory.
5.—English Classics 1 hour per week,
Juniors,
Obligatory.
6.—The Early English Authors 2 hours per week,
Graduates.

 

DEPARTMENT OF THE MODERN LANGUAGES.

1a.— French, Elementary Course 3 hours per week each,
Undergraduates.
1b.—Course of French Literature
2a.— German, Elementary Course 3 hours per week each,
Undergraduates.
2b.— Course of German Literature
3a.— Italian, Elementary Course 3 hours per week each,
Undergraduates.
3b.— Italian Literature 1 hour per week each,
Graduates or Undergraduates.
4a.— Spanish, Elementary Course 3 hours per week each,
Undergraduates.
4b.— Spanish Literature 1 hour per week each,
Graduates or Undergraduates.

 

DEPARTMENT OF HISTORY AND POLITICAL SCIENCE.

1.—Outlines of General History 3 hours per week,
Sophomores,
Obligatory.
2.—History of England and the United States, one term 3 hours per week,
Juniors,
Obligatory.
—Political Economy, one term
3.—Political Economy and Statistics 3 hours per week,
Seniors,
Obligatory,and School of Civil Service.
4.—British and American Constitutional history 2 hours per week,
Seniors,
Obligatory.
5.—General Constitutional History 3 hours per week,

School of Civil Service.

6.—Literature of Political Science 1 hours per week,
3 hours per week,
School of Civil Service.
—Comparison of Constitutional law of Europe and the United States
7.—Systems of Administration 3 hours per week,
2 hours per week,
School of Civil Service.
—Comparison of Constitutional Differences of the American States
8.—Comparison of Systems of Administration 5 hours per week,
School of Civil Service.

 

ROMAN AND INTERNATIONAL LAW.

1.—Roman Law 3 hours per week,
School of Civil Service.
2.—International Law 3 hours per week,

School of Civil Service.

 

DEPARTMENT OF GEOLOGY.

1.— General Geology 1 hour per week,
Seniors,
Elective.
2.— Palæontology 1 hour per week,
Seniors,
Elective.
3.— Lithological Geology, first term 3 hours per week,
with School of Mines.
4.— Cosmical and Historical Geology, second term 3 hours per week,
with School of Mines.
5.— Economical Geology 2 hours per week, through the year
with School of Mines.

 

NATURAL HISTORY.

Botany, first term 2 hours per week,
with School of Mines.
Zoology 1 hour per week, throughout the year
with School of Mines.
Vegetable Physiology, Animal Physiology, Anatomy—Human and Comparative To be provided for.
Mineraology and Crystallography 2 hours per week,
with School of Mines.

 

LAW.

1.— Outlines of British Common Law Ten lectures.

 

SEMITIC AND ORIENTAL LANGUAGES.

Hebrew 2 hours per week,
To be provided for.
Sanskrit 2 hours per week,
To be provided for.

 

 

Source: Columbia University Archives. Historical Subject Files, Series I: Academics and Research/Series VIII: Events/I. Box 289. Folder 1 “Political Science, Faculty of, 1920s-1930s”.

Image Source: John W. Burgess, from the Columbia University, Department of History webpage: A Short History of the Department of History.

 

Categories
Economics Programs Harvard Teaching

Harvard. Haberler and Chamberlin fight over last-minute course changes, 1942-43

Exogenous shocks are really useful for finding out how the economy works. They also help dear colleagues reveal themselves when their private interests conflict with those of other colleagues in particular or with departmental needs in general. The U.S. entry into the Second World War forced several adjustments in the graduate and undergraduate instructional staffing at the Harvard economics department.

This post provides some light on the time Gottfried Haberler was asked to teach the first of the two term graduate economic theory sequence for the academic year 1942-43. The course was a direct descendent of Frank Taussig’s Economics 11 (the expansion of course offerings over the decades required moving into 3 digits for some course numbers and a zero was dropped into the middle of “Economics 11” to obtain “Economics 101”). At the last minute Chairman Edward Chamberlin decided that he wanted “his” course back for both semesters but Gottfried Haberler was clearly not one to go quietly. And so we witness the performance of an academic drama before colleagues, of Professor X and Professor Y claiming conflicting rights to a particular course.

The record presented here is incomplete. I have been unable to find Haberler’s written plea on his own behalf. Reading the material one might think that Chamberlin got his way and Haberler was left to find another course to satisfy his annual teaching obligation. However, a look into the annual report of the President of Harvard College for 1942-43 finds that as far as the staffing of Economics 101 in 1942-43 goes, ex ante equals ex post. The course was ultimately divided that year between Messrs. Haberler and Chamberlin.

___________________________

Economics 101: syllabi (with links to most readings) and examinations for fall and spring terms 1941-42 taught by Edward Chamberlin.

___________________________

Who ended up teaching what 1942-43

Edward Chamberlin

Economics 1a. First term, undergraduate course “Economic Theory”.

Economics 102b. Second term, graduate course “Monopolistic Competition and Allied Problems”.

Co-taught Economics 101 with Gottfried Haberler. Full-year graduate course “Economic Theory”. Presumably Haberler taught the first term and Chamberlin taught the second term.

Gottfried Haberler

Economics 18b. Second term, undergraduate course on the Economic Aspects of War,

Co-taught Economics 45a with Alvin Hansen. First term, undergraduate course  “Business Cycles”.

Economics 144. Graduate School of Public Administration Seminar “International Economic Relations”.

Co-taught Economics 101 with Edward Chamberlin. Full-year graduate course “Economic Theory”. Presumably Haberler taught the first term and Chamberlin taught the second term.

___________________________

Ex Ante Course Announcement

Economics 101. Economic Theory

Mon., Wed., and (at the pleasure of the instructors) Fri., at 12. Professors Chamberlin and Haberler.

Source: Final Announcement of the Courses of Instruction offered by the Faculty of Arts and Sciences during 1942-43. Official Register of Harvard University, Vol. XXXIX, No. 53 (September 23, 1942), p. 55.

___________________________

Ex Post Course Enrollment and Staffing

[Economics] 101. Professors Chamberlin and Haberler. — Economic Theory.

Total 24: 16 Graduates, 4 School of Public Administration, 1 Graduate Business School, 3 Radcliffe.

Source: Harvard University. Report of the President of Harvard College 1942-1943, p. 47.

___________________________

Presumably the statement prepared by Edward Chamberlin (referring to himself in the third person)

October 9, 1942

Course Economics 101 was announced in the catalogue for 1942-3 to be given jointly by Messrs. Chamberlin and Haberler. This arrangement was never considered as final but was subject to adjustment at the beginning of the college year in view of the general uncertainty as to the status of such of the graduate instruction until enrolments in various courses were known. (In particular, it seemed likely that either 102b or 163 or both might be bracketed, thus freeing either one half or one full course of Mr. Chamberlin’s time). It was, however, agreed between Mr. Haberler and Mr. Chamberlin that, in case the course were given jointly, Mr. Haberler would give the first half year and Mr. Chamberlin the second. Several times prior to the opening of college Mr. Haberler asked Mr. Chamberlin about the status of the course and was told that unfortunately nothing final could be decided until enrolments were known; it was agreed, however, that Mr. Haberler would take the first meeting, or meetings, of the course until a decision was reached. The matter was mentioned on Friday morning, October 2, at a casual meeting between classes at which time, since no final decision had been taken, Mr. Chamberlin said that it was still possible that the arrangement might stand. On Saturday, October 3, a final decision to take back the course was communicated to Mr. Haberler after considering a number of factors, among which were the following:

  1. The enrolment in Economics 102b was only two, plus five auditors. This course had always been given in the second semester, thereby opening it to the first year students who had had the first semester of 101. The bracketing of 163 made it possible to revert to this disposition of 102b, (or to bracketing it later on if this seemed necessary). The chief obstacle to Mr. Chamberlin’s giving the first half and therefore all of 101 was thereby removed.
  2. The class list of 101, received Friday afternoon, revealed that of 16 [or 18?] student then enrolled in the course all but two were foreigners. Many of these would have serious problems of adaptation to academic work in a new language and in a new country, and it seemed for the reason especially desirable to unify the introductory course in theory under one direction during the current year.
  3. During the past, two years the course had, for better or worse, been split both vertically and horizontally, not by action of the department but on the initiative of Mr. Chamberlin. This was done in part to open greater possibilities for discussion through smaller sections, and in part to share the course with others who wanted very much to teach theory. At no time during that period had Mr. Chamberlin given less than a full year of the course, and its outline and organization had always been his. It was his sincere belief that now that the course was again of manageable size the department would wish it to be given as it had directed earlier, and that he was fully competent to make the decision. At that time the work of the year had not really begun.

However, Mr. Haberler objected so strongly to the change that in order to settle the matter amicably, Mr. Chamberlin proposed on Sunday afternoon, and Mr. Haberler agreed, that the matter be left to a committee composed of Professor Crum as Chairman and other members to be chosen by Professor Crum. As this Committee could not possibly be assembled and give a decision before the Monday meeting of the course it was agreed that Mr. Haberler would take that meeting and that the Committee shouId render a decision before the Wednesday meeting. The decision was in fact rendered Tuesday afternoon and was unanimous that Mr. ChamberIin should give the course, When apprized of this decision, Mr. Haberler said he would like time to consider whether or not he was willing to accept it. From this point on Mr. Chamberlin became a passive duopolist, leaving all initiative to Mr. Haberler, who proposed that he take the Wednesday meeting of the course, finishing matters which he had begun on Monday, give a cut on Friday (there was a holiday the following Monday), and decide sone time before the Department meeting whether or not he would like to bring the matter before the Department. Meanwhile, the Committee had decided that certain questions which it had discussed apart from the immediate issue should be brought before the Department at its meeting October 13th. Mr. Haberler’s final decision on Thursday was that if the Department is going into the whole theory question anyway, they should also decide on the present status of Economics 101.

___________________________

Chairman Chamberlin announces his attention to return statement with statement

October 10, 1942

Dear Leonard [Crum]:

It has occurred to me that, since Haberler has given you in writing a statement of the facts as he sees them, I might, even at this late date do the same. My own statement will add some details and perhaps present a different emphasis at one or two points It may be used at your discretion in whatever way you think best, (including, of course, the possibility of no use at all beyond your own reading). I am sending a copy to Haberler.

Sincerely,

E. H. Chamberlin

___________________________

Chairman Chamberlin makes his written statement available to the department

October 17, 1942

To Members of the Department of Economics:

In view of the fact that Professor Haberler’s statement with respect to Economics 101 had some circulation prior to last Tuesday’s meeting and was also read in the meeting, whereas my own statement has to this moment been seen only by Professor Crum and one other member of the Department, I should like now to make both equally available to any who may wish to consult them. Accordingly, they will both be found in the blue folder in Mrs. Arnold’s office. Also in the blue folder are: (1) The minutes of the last three meetings, and 2) The report of the Chairman to the Dean of the Faculty covering the work or the Department for the past year.

Chairman [Chamberlin]

___________________________

Special Committee sides with Chamberlin

CONFIDENTIAL

(for use of Department
of Economics officers
only)

Report of a Special Committee
on the assignment for teaching Economics 101.

13 October, 1942

On Monday, October 5, the Chairman of the Department brought to my notice a personal disagreement between himself and Mr. Haberler concerning the assignment for teaching Economics 101, and asked that I serve as Chairman of a Special Committee to “arbitrate” in the case, and report before the meeting of Economics 101 on Wednesday the 7th. I was instructed to associate with myself such members of the Department as I saw fit in making up the Committee. I asked Mr. Burbank, formerly Chairmen of the Department, to be a member, and also four other members of the Department who have at present no active part in the teaching of economic theory and whose views on the matter at issue were unknown to me. One of these individuals was unable to serve because of his inability to meet with the Committee at any time available for meeting within the interval during which action had to be taken. The Committee, therefore, was made up as follows: Crum, Chairman, Black, Burbank, Dunlop, and Usher.

The Committee met and considered to the best of its ability all aspects of the case, and herewith reports certain recommendations to the Department for such action as it wishes to take. The Chairman of the Committee reported on Tuesday afternoon the 6th to Messrs. Chamberlin and Haberler the findings of the Committee in outline form because the Chairman thought that the two individuals concerned might have agreed to abide by the finding of the “arbitration” and might be willing to put the findings into effect immediately. The Chairman of the Committee did, however, report to both participants in the controversy that he did not regard the Committee as being clothed with any conclusive authority and that unless the participants in the controversy both accepted the findings of the Committee those findings would have to go to the Department as recommendations and would be subject to such action as the Department saw fit to take.

Statement of the issues.

Course Economics 101 is announced in the spring issue of the current Courses of Instruction pamphlet as to be given jointly Messrs. Chamberlin and Haberler, and I am informed that they had during the summer agreed among themselves that, in case the course was given jointly, Mr. Haberler would give the course during the first half year and Mr. Chamberlin during the second. Late in the week in which instruction of the present half year began Mr. Chamberlin indicated to Mr. Haberler that he thought the entire course should be given by Mr. Chamberlin. Mr. Haberler objected to any such change and insisted that he continue to give the course during the first half year. The issue, accordingly, was whether the conduct of the course should go forward on the basis of joint responsibility of Mr. Haberler in the first half year and Mr. Chamberlin in the second half year, or should be restored to the basis prevailing for several years in which Chamberlin gave the full course.

History of the case.

After the retirement of Professor Taussig, Course 101 (formerly called Course 11) was given for several years by Mr. Schumpeter by an arrangement which was understood to be provisional and subject to later change. At the end of this interim, after extended consideration of the needs and purposes of the Department with respect to the teaching of the several courses in economic theory, the Department took specific action directing Mr. Chamberlin to teach Course 101. At the same time arrangements were agreed upon by which several other specialists in economic theory in the staff of the Department participated in the instruction in economic theory. Presently the enrolmont in Course 101 became so large that its conduct as a single course by the discussion method became difficult; and, without specific vote of the Department, the course was divided into two sections, with one conducted by Mr. Chamberlin and Mr. Taylor in 1940-41 and by Mr. Chamberlin and Mr. Leontief in 1941-42, the other by Mr. Haberler and Mr. Chamberlin in both years.

With the decline in enrolment which has occurred, no occasion for such splitting of the course persists, and it has long been forseen that Economics 101 would be conducted as a single section during the present year. In recognition of this, an arrangement was made, without specific action by Department vote, to announce Economics 101 for the present year as to be given jointly by Messrs. Chamberlin and Haberler. At the time this arrangement was made the expectation was that Mr. Chamberlin would be giving during the first half year Economics 1a and Economics 102b and that he should not be called upon to carry the additional instruction involved in teaching Economics 101 during the first term.

The initial enrolment in Economics 102b was so small that the course has been withdrawn from the first term offering, and although it is announced for the second term doubt remains whether the enrolment will be sufficient even then to warrant giving it. In recent months, various changes in personnel of the Department and the necessity of distributing the teaching and other load in all branches of the Department work as fairly and efficiently as possible have resulted in various changes in the assignments of particular officers to particular duties. In these circumstances it became possible for Mr. Chamberlin to resume during the first term instruction in Economics 101 without making his course load excessive.

In connection with the controversy, the Chairman of the Committee had a conversation with Mr. Chamberlin in which the latter presented his own views concerning the history of the case and the points at issue. Mr. Haberler submitted a written statement to the Chairman of the Committee setting forth his ideas on the matter. Those items were brought to the attention of the Committee by its Chairman. Following the meeting of the Committee, Mr. Chamberlin also submitted a written statement to the Chairman of the Committee. Either or both of the written statements will be laid before the Department on request.

Meeting of the Committee.

The Committee met on Tuesday, October 6. The Chairman gave the Committee a history of the case and a summary of the information available bearing upon the point at issue. The Chairman also informed the Committee that he did not understand that the Committee had any conclusive powers and would be obliged to report its findings in the form of recommendations to the Department.

The Chairmen specifically urged the Committee, therefore, in proceeding toward its findings to consider the wisdom of bringing in findings which, in its opinion, would probably be supported by the Department. The Chairman reminded the Committee that adequate treatment of the particular matter at issue might well involve (a) recommendations by the Committee concerning certain related matters affecting other courses; and (b) recommendations by the committee concerning certain longer run matters relating to the general question of our offering in economic theory. The Chairman discussed with the Committee certain basic principles bearing upon the case, and received the concurrence of all the members of the Committee in these principles. They are outlined below.

The Committee then proceeded to discuss the matter at issue and various related matters. Discussion by the members of the Committee was free and active and the Chairman made a special effort to call forth the views of each member of the Committee. After this discussion the Committee agreed upon a set of recommendations to be made to the Department, and to be reported to Messrs. Chamberlin and Haberler in the hope they would accept the findings. The agreement of the Committee was unanimous. Those recommendations are presented below.

Basic principles.

In approaching a set of findings with respect to the issues raised the Committee had in mind a series of basic principles in which members of the Committee concurred. Those are as follows:

(a) Because of its compressed personnel in wartime and because of the extraordinary wartime adjustments needed in its work: the Department has a peculiarly difficult task of assigning functions to its various officers with a view to getting the essential work of the Department done with such distribution of the burden as will be primarily efficient from the point of view of the Department and secondarily fair from the point of view of the individuals.

(b) Even in peacetime the needs of the Department and the objective of securing maximum efficiency in the performance of Department work transcend the interests and preferences of individual officers. Although in peacetime many concessions can be made with a view to accommodating the preferences and interests of individual officers and with a view to protecting the rights or supposed rights of individual officers, the Department would in general not recognize that such individual interests can overrule the general interest of the Department. In wartime this condition is even more emphatically true, and in such time the individual preferences and interests may be obliged to give way to the general interest of the Department more frequently and more extensively than in normal times. Throughout the duration of the war many if not all of the officers of this Department will be doing work which they prefer not to do and will be denied the opportunity to do work which they would like to do. Without such sacrifices the essential work of the University cannot be effectively handled in wartime.

(c) The Department and the University cannot afford to allow the general interest to be sacrificed because of informal commitments or quasi agreements made among individual officers when such agreements fail to take adequate account of the general interest of the Department, even though those who made the agreements acted in good faith. That agreements thus made may from time to time have to be set aside in the interest of the Department, and that such setting aside may involve some sacrifice by one or more individuals involved must be accepted as one of the costs of giving primary importance to the general interest of the Department. Ordinarily it is to be expected that individuals will refrain from making arrangements for which they have no power under the law of the Department; but ever if such arrangements are entered into under a grant of power, the individuals concerned must recognize that the Department itself has a clear right to final determination at one of its meetings.

(d) To the best knowledge of the Committee, the purpose of the Department with respect to the assignment of instruction in Course 101 remains as it was last officially determined by Department vote several years ago, namely, that Course 101 should be given by Mr. Chamberlin.

(e) Under the stress of war the Department may be obliged to sacrifice in part some branches of its work, and the Committee believes that graduate instruction will probably need to be sacrificed before instruction in undergraduate courses, tutorial instruction, and other Department work directed toward the teaching of undergraduates. A policy which exposes graduate instruction to the principal sacrifices is also likely to result in the most frequent disregard of personal preferences and even of supposed rights of individual officers; but presumably the Department would nevertheless feel that such a policy must be adopted and maintained.

Recommendations to the Department.

After considering the facts laid before it in connection with the matters at issue and in the light of its own agreement on basic principles, the Committee agreed unanimously to present the following recommendations to the Department at its meeting on Tuesday, October 13, and to report these recommendations at once to the parties in the controversy:

(a) That during the present year Mr. Chamberlin be assigned to conduct the entire Course 101.

(b) That, in view of chancing conditions which may mean that the Department’s present total offering in economic theory covering the entire range of courses in that field does not most satisfactorily meet the needs of instruction in that field, the Department promptly and earnestly reconsider the total offering with a view to making such changes as may be necessary in the next announcement of courses. The Committee makes no recommendation as to how the reconsideration should be conducted, whether by the appointment of a committee or by general Department discussion or by a combination. It also makes no specific recommendation as to any changes in the present offerings of courses, but merely notices that such a general reconsideration may well cover the possibility that Mr. Haberler might be asked to give work in economic theory.

(c) The Committee recommends that the Department consider asking Mr. Haberler to take charge of an additional half course during the present academic year, with a view to replacing the first half of Course 101 in rounding out his teaching assignment. The Committee specifically recommends that Course 18b be considered as one of the possibilities for additional instruction by Mr. Haberler; and makes this recommendation because on the one hand the Committee feels that the hurried arrangement by which that course was assigned jointly to four officers won Mr. Harris withdrew may have been ill-advised in that use of too numerous instructors in such a course may damage the continuity from the point of view of the student; and on the other hand the Committee believes that Mr. Haberler’s areas of specialization would enable him to handle this particular course very effectively.

(d) The Committee recommends that the Department consider carefully the question whether in determining that the enrolment in a course is so small that the course should be withdraw only those enrolled for credit should be counted, or whether in addition the auditors should be counted (this question was raised before the Committee in connection with Course 102b in which the first term enrolment was two members for credit plus five others. Course 102b has been withdrawn from the first term offering, but will be announced again for the second term, and the question posed above may at that time again be raised).

W. L. Crumm

___________________________

Thus spake the Dean

HARVARD UNIVERSITY
CAMBRIDGE, MASSACHUSETTS

FACULTY OF ARTS AND SCIENCES

Paul Herman Buck, Dean
Henry Chauncey, Assistant to the Dean
Jeffries Wyman, Jr. Assistant Dean

5 University Hall

January 12, 1943

To the Senior Members of the Department of Economics:

After considerable contemplation of the issue which has arisen between Professors Chamberlin and Haberler and which I have undertaken to arbitrate, I find I am in complete accord with the Report of a Special Committee on the assignment for teaching Economics 101, dated October 13, 1942. I commend especially as sound, the basic principles outlined on page 3 of that report and I accept as my official decision the recommendations to the Department given on page 4 of that report.

Frankly, it seems to me most unfortunate that the issue should have descended into personalities. The department should be prepared to face the large problems of policy which I have outlined in a letter to your Chairman which, I trust, will be read at your meeting tonight. Obviously those problems will not be solved intelligently and equitably if they are not approached with a vision directed to the loyalties of one’s subject and university rather than to self. Is it asking too much to relegate the personal aspects of this issue to oblivion?

It seems to me very important so to do. I have taken a great deal of pride in the distinction of the Department of Economics at Harvard and I have spoken in many circles boastfully of having what seems to me one of the very few remaining great departments of economics in the world. Certainly the responsibility of keeping that department great and of enabling it to develop continuing leadership should be the major loyalty to which every other consideration is subordinate. The awareness of this responsibility and the opportunities it presents will preoccupy your time and energies. Let me conclude by saying that I have always had and retain confidence in the intelligence, initiative, devotion. and cooperative spirit of your membership. I write this with all the more assurance because I know so many of you intimately and appreciate from personal friendship the qualities I have mentioned.

Very truly yours,
[signed]
Paul H. Buck

 

Source: Harvard University Archives. Department of Economics, Correspondence & Papers 1902-1950. Box 25. Folder “Graduate Instruction in Theory. Economics 101. 1942-43.

Categories
Columbia Cornell Economics Programs Harvard Michigan Popular Economics Yale

“Political Economy and the Civil War” by Laughlin that provoked an Economist-Bashing editorial, 1885

Before becoming the founding father of the department of political economy at the University of Chicago, the 35 year old Harvard assistant professor J. Laurence Laughlin (Harvard Ph.D. 1876) published an essay, transcribed below, arguing that liberal college education needed to be expanded beyond Greek, Latin, mathematics, and philosophy to include courses dealing with economic theory and its policy applications. He provides us a table of the limited course offerings in political economy at five major colleges/universities at the time. I stumbled upon an unsigned editorial written in response to Laughlin that I have also transcribed and which is placed at the end of this post. The editorial provides us with historical evidence that ill-tempered economics-bashing is hardly a creation of the Twitternet Age. No siree Bob! The editor was not amused by Laughlin’s presumption, calling him and his college professor colleagues who taught political economy to boys…”vealy milksops”. I dare any or all visitors to sneak that expression into a footnote.

________________________________

POLITICAL ECONOMY AND THE CIVIL WAR.
By J. Laurence Laughlin.

Atlantic Monthly, v. 55 (April, 1885) no. 330, pp. 444-450.

In some parts of our country there is a current maxim among the old-fashioned gardeners to the effect that “a wind-shaken tree will bear much fruit.” There is some subtle force in it. In fact, it is an expression which may be regarded as finding its parallel in individual and social life. As individuals, we know that there is no real growth of character except by a conquest over opposing difficulties; the doing right when it is against our inclinations and prejudices. And in a social organism we seem to see a moral law of conservation of energy by which a sacrifice is the parent of some gain,— a thing which evidently underlies the movements attending many great convulsions in political life. We saw armies go out of our sight during the civil war, only to come back thinned, injured by disease, with half their number left dead on the field. Death meant bitter, indescribable sorrow in all our homes. The experiences of the war were felt to be pitiless, inexplicable, and hard. And yet, perhaps, a subtle suggestion may have come into our minds that it was not simply by dying, or in living, that the best law of our being was enforced; that there was, in truth, some Power behind it all; that some purpose was being worked out through each one of us, just as each leaf on the tree, for example, is necessary to the completed organism of the whole tree, and ceases to be when it is separated from the stem. Now, perhaps, even at this short distance from the struggle, we can begin to see some of the effects of that social and political upheaval, the greatest since the foundation of our government. It is worth while to examine whether the wind shaken tree has borne much fruit.

The process by which citizens from the secluded districts and remote towns were sent through new cities to opposite parts of the Union, exchanging ideas with men of different habits of thought, was a marked feature of the war period, and leavened the mental life of the American people in a way hither to little suspected. It was something like sending a country boy to college, only the effect was multiplied a million times. The rural population came into a knowledge of our cities, while the urban classes were carried out into new climates and into unvisited parts of our vast domain. New sights, new methods of cultivation, different habits of living, stimulated the dull and fired the active and enterprising men in the ranks. The life of the farm and the village was widened to an interest in the nation. About the same time, moreover, came a vast increase in easy means of communication by railways and a greater extension of the use of the newspaper and telegraph, by which provincial towns were brought into direct connection with the outside world. Even oddities of customs and dress began to disappear, in the process of comparison with the more attractive ways of the dwellers in the great cities and towns. In this fashion, the thinking horizon was extended. Dull intellects learned the presence of complicated problems, and brighter minds found new spurs to ambition in the questions of larger interest. On all sides men felt themselves coming daily into contact with new difficulties, under a dim consciousness of their bigness, but with a strong belief that the knowledge how to deal with them was inadequate. In short, the tremendous crisis through which we passed, apart from its effect on the preservation of the Union, has been subtly at work in moral and intellectual directions. The working of these new forces on a quick and susceptible race can easily be imagined. They have, in fact, under somewhat similar conditions, had a distinct influence on a more phlegmatic people than ours. Old students at Göttingen, who have returned to the university since the late wars in which Germany has been engaged, have been amazed to find the old-fashioned spot — where the customs, habits, and naive simplicity of one hundred years ago had prevailed until quite recently — now wholly changed. The commercial spirit has seized the formerly simple-minded peasants, and the quiet town now hears the heavy march of cosmopolitanism in its streets.

Like Germany, the United States had new problems to solve. While the conflict closed the long slavery struggle, it brought with it intricate questions, but of a character very different from those which had gone before. Without warning, and consequently without the ability to get due preparation or acquire proper training, our public men were confronted, as the war progressed, with matters of vital importance in international and constitutional law, in taxation, and in every form of administration and finance. The demand for men who had given themselves more particularly to the province of governmental science was an imperative one; but it was, generally speaking, met in a way which showed that there existed in the community a class from whom these necessary men could be recruited. That class was the legal profession of the country. The questions of reconstruction, the relation of the general government to the States, the civil rights of the negro, our relations with foreign powers during the blockade of Southern ports, were not abandoned to men who had never habituated themselves to discussions such as were involved in their settlement. There were differences of opinion, of course; but inasmuch as these differences of opinion were produced by different political theories, this proved that attention had been given to such subjects to the extent that a crystallized system of thought, formulated in dogmas, had been created by the various parties.

But, as has been suggested, new considerations arose. The magnitude of the military operations involved an enormous expenditure of money by the state, and made a demand upon our statesmen for financial skill of an almost unparalleled kind. To meet these extended questions of taxation, finance, and currency, what body of men could be called upon? To this, answer must be made that the war overtook us without a supply — or even a few — of trained economists and financiers. The economic part in the equipment of a public official had been wholly neglected. In fact, political economy and finance had never been seriously studied in the schools; but, if studied, they were classed in the old-fashioned required curriculum with Butler’s Analogy and the Evidences of Christianity. Although Adam Smith wrote his Wealth of Nations in 1776, political economy was an unknown science to the American people before 1860. It is an interesting study to examine the manner in which our people went under the burdens and tasks of our great civil conflict. There was the quick adaptability of Americans to start with; there was plenty of patriotism and good will, and no lack of those high qualities of self-sacrifice and heroism which are still fragrant to us; but lawyers, such as Chase and Fessenden, were practically our only financiers. Early in the war they were required to consider a scheme — for the right settlement of which a vast experience is necessary — of raising loans, and adjusting a plan of taxation corresponding to the extraordinary war expenses. Without considering alternatives, in a few years they created a debt as great as that incurred by old despotisms of Europe in centuries; without foresight, they drifted into a ruinous issue of irredeemable paper money; without intending it as the object of a definite policy, but through a desire simply to gain a war revenue, they established an extended system of “protection to home industries” by levying duties on imports, which has brought into existence business interests largely dependent on the continuance of these temporary war measures. When it is realized that principles of taxation are to-day probably less understood than any other branch of economics, it is not surprising to find that in 1864 Congress was occupied only five days in passing the most gigantic taxation measure of the war. The National Bank Act, which has given us the best system of banking ever enjoyed by the country, was, however, in reality passed as an act to facilitate the sale of our bonds and aid our tottering credit. We blundered egregiously, but we were capable of learning by experience. Yet it was from these very blunders, from this revelation of inexperience made evident by the demands of a great emergency period, that the community received an impetus toward the study of economic questions which was certain to result in good fruit.

In fact, it is now clear that a new interest in economics and finance has already arisen. The civil war was, so to speak, the creation of economic study in the United States. The war did for this country — in a different way, of course — even more than the corn-law agitation did for England. It actually gave birth to new motives for study. There never was a time in our history when there was so evident a desire to get light on the economic problems of the day as now. There is a new stir among the ranks of the young men at college; and the printing-press sends forth an increasing stream of new books upon subjects which are constantly discussed in the daily newspapers. There is unquestionably a new-born, slowly growing attention by the younger men of our land to the necessity (as well as the duty) of fitting themselves properly for the responsibilities of citizenship. If the war has given us this, — the absence of which used to be so often lamented a few years ago, — then may some of our sacrifices not have been in vain. The wind-shaking has resulted in abundant fruit.

In the present awakening in educational discussion, one phase of which has been called the “Greek Question,” it is worth while to notice the influence of the war period on the college curriculum. In most of our schools and universities, on the breaking out of the war (and even to the present day), the pecuniary resources and endowments had been tied down, under the force of old traditions, to supply instruction in the customary Greek, Latin, mathematics, and philosophy, which were then considered the only essentials of a liberal education. But when the rude shock of the war awakened us to our ignorance, and we looked around for the schools where the new studies could best be followed, it was discovered that the college curriculum made practically no provision for such instruction. In the old days when sailing vessels alone entered Boston harbor, only one channel was practicable, and all the fortifications were placed in a way to command it: but when steam took the place of sails, another channel was adopted, but it is now wholly undefended. The old ship channel must be defended, but so must the new one. So, in the collegiate studies, the old subjects are necessary, of course, but they are not the only necessary ones. The new demands, due to the progress of the age, must also be met. In fact, the response of the schools to these new demands is at once the evidence and result of the quickening and stimulating forces so briefly sketched in these pages. A comparison of the amount of instruction in political economy given by the principal institutions of the land in the years 1860, 1870, and 1884 will furnish us new proof that the wind-shaken tree is yielding full fruit.

Nothing could show more distinctly than the accompanying table how young any real systematic study of political economy is in this country, and it accounts for the lack of any number of trained economists among us. But the younger generation are happily recruiting their ranks, now that these better opportunities are open to them.

At no time, however, have public affairs demanded unpartisan study in economics more than to-day. In past centuries governments were supposed to labor, in an unsettled state of society, for the protection of life and property. Now that the general progress of civilization and Christianity has made life and liberty more secure, legislation in later years has concerned itself rather with property than life. In the Middle Ages trade was considered plebeian; to fight or to oppress was regarded as more noble. Now the chief solicitude of the modern state is the increase of wealth: the castles have become mills; retainers, productive laborers; and arms, the hammers and tools of the artisan.

1860.

1870.

1884.

Yale College. One third of Senior year One third of Senior Year 1.  Elementary Course. — Fawcett. — Discussions on currency, banking, and taxation. 3 hours a week for 13 weeks.
2.  Elementary Course. — Mill. — Currency, banking, and taxation. 2 hours a week for a year.
3.  Advanced Course. — Discussion of economic problems and fallacies, with selections from leading treatises. 2 hours a week for 20 weeks.
4.  Graduate Course. — Finance and the Art of Politics, as illustrated in the History of the United States. 2 hours a week for 2 years.
5.  Graduate Course (in alternate years.) — In 1883-4, Sociology. In 1884-5, Industrial History, History of Political Economy, Finance and Theory of Rights. 1 hour a week for each year.
6.  History, business methods, and social problems, of Railroads. 2 hours a week for a year.
[A course about equal to Courses 1 and 2 is given in the Sheffield Scientific School.]

Cornell University.

[Institution not founded]

One third of Junior Year

1. Elementary Course. — Lectures and Recitations. 2 hours a week 2/3 of a year.
2. Lectures on Political Economy.5 hours a week for 1/3 of a year.
3. Lectures on Finance.

University of Michigan.

Not in the Course of Study.

One Term of Senior Year.

1. Elementary Course. — Lectures. 3 hours a week ½ of a year.
2.  Advanced Course. — Competition, Free Trade and Protection, Commercial Depressions, Transportation, etc. 3 hours a week ½ of a year.
3.  Principles and Methods of Finance. — Banking, National Debts, etc. 2 hours a week ½ of a year.
4.  History of Industrial Society [not given in 1883-4]. 2 hours a week ½ of a year.
5.  Financial Seminary.— History of American Finance. 2 hours a week ½ of a year. [Not given 1883-4.]

Columbia College.

Elective in one part of Senior Year.

One Term of Senior Year.

1.  Principles of Political Economy.— Elementary Course. Rogers’ Manual. 2 hours a week ½ of a year.
2. History of Politico-Economic Institutions. 2 hours a week ½ of a year.
3.  Finance and Taxation. 2 hours a week ½ of a year.
4.  Statistical Science, Methods and Results. 2 hours a week ½ a year.
5.  Communistic and Socialistic Theories. 2 hours a week ½ a year.
6.   [Topics like railways, banks etc., are placed under Administrative Law.]

Harvard University.

One half of Senior Year.

1. Rogers’ Manual One half of Junior Year 1. Elementary Course.— Mill’s Political Economy. Lectures on Banking and the Financial Legislation of the United States. 3 hours a week for a year.
2. Elective Course for Seniors: Adam Smith, Mill, Bowen. 3 hours a week for a year. 2. Advanced Course.— History of Political Economy. Cairnes, Carey, George, and recent literature. 3 hours a week for a year.
3. Investigation of Practical Questions of the Day.— Banking, Money, Bimetallism, American Shipping, Note Issues, etc. 3 hours a week for a year.
4. Economic History since the Seven Years’ War.— 3 hours a week for a year.
5. Land Tenures in England, Ireland, France, and Germany.— 1 hour a week for a year.
6. History of Tariff Legislation in the United States.— 1 hour a week for a year.
7. Comparison of the Financial Systems of France, England, Germany, and the United States.— 1 hour a week for a year. [Omitted 1884-5.]
8. History of Financial Legislation in the United States. 1 hour a week for a year.

Consider the character of the questions at this time pressing upon Congress for immediate attention. If we omit the administrative and political legislation on the civil service, the succession to the presidency, and a national bankruptcy law, the remaining questions before Congress to-day are almost entirely economic. (1.) There is, in the first place, the false silver dollar, masquerading in sheep’s clothing, and waiting to catch the unwary business world napping, when it will suddenly assume its true depreciated character, and devour fifteen or eighteen per cent of all creditor’s dues estimated at present prices. What is Congress doing here? Just what it did in the last months of 1861, when it let the country drift on to the shoals of depreciated paper. Monometallists and bimetallists, business men and bankers, are assaulting the dangerous silver legislation, and yet Congress is a very Gibraltar in which the silver owners are intrenched. (2.) Next, there is the banking question. Nothing can be more delicate and sensitive than the machinery of credit and banking in a great commercial country such as this; and yet men, to satisfy the prejudices of constituents, handle this mechanism with about the same air of cheerful indifference as that of a child who drags a rag doll round by the heels. The present national bank notes give a stability to trade in separate parts of the Union, by means of a currency equally good in Maine and Texas, never reached in the days of the vicious and changing state banks; and yet the present system is gradually vanishing before our very eyes, as calls are made for government bonds. (3.) Again, Congress is struggling with the most difficult of all problems, – national taxation. It means a reëxamination of our whole scheme of taxation, the retention of internal taxes on distilled spirits and tobacco, the management of our surplus revenue, the whole sub-treasury system; while the situation inevitably requires a readjustment of our customs duties. Duties needed in order to procure a large revenue in time of war are no longer necessary when the war is ended, and the national debt is reduced one half. (4.) There are the barbarous and mediaeval navigation laws, to which we cling with a curious indifference to the influence of all progress and liberal ideas. The problem of our shipping and merchant marine needs the touchstone of some wider training than is furnished by selfish individual interests. (5.) Our public lands and the settlement of our vast Western domain are important matters of land tenures, and yet they are abandoned to accident, while the possibilities of good disappear under the cloud of accomplished facts, where nothing can be done. It will not be long before all the public lands will be gone, and yet no notice is taken of existing evils. (6.) Then, again, one has but to mention the word “railway,” and there arises to the mind a congeries of difficult questions dealing with Western “grangers,” the ability of the state to regulate freight and passenger charges, and in fact the whole vexed discussion of state interference. Here is a field by itself, to which a man may well give his whole life-work. (7.) It would be wearisome to more than mention the topics of Postal Telegraph, Chinese Labor, Strikes, Trades Unions, and Communism, which attract our instant attention. (8.) Then again the unfortunate legal-tender decision of Judge Gray has brought back to us all the troublesome and intricate discussions on the currency which we once thought had been forever settled. As matters now stand, power is given to Congress, if it chooses, to repeat all the errors of Continental currency policy, and we are put back a century in our paper money teaching. (9.) To pass from merely internal matters, so long as we were the only civilized people on the western continent, our relations with our neighbors gave us little thought. The growth of commerce, the expansion of populous areas north and south of us, the discovery of mineral wealth outside our own limits, which invites our capital, has forced on us the consideration of reciprocity with Canada and Mexico. We have refused reciprocity to Canada; but to-day we are considering the desirability of granting closer commercial relations with Mexico, while Cuba and Porto Rico have asked the same advantages by a new treaty.

Such, in brief, are some of the subjects which must be made matter of instruction in our schools and universities. It will be observed how overwhelming a proportion of public measures at present are economic, and what a heavy responsibility lies upon our institutions of learning, if they are to meet the new demands in a fitting manner. But there is a still stronger reason for strengthening our educational forces on the economic side. This is to be found in what may be called the “economic portents.” To the present time we have been properly called a “young country,” which to the economist means an abundance of unoccupied land, a scanty population, large returns to capital, and high wages. A full knowledge of our resources has not practically been reached as yet, and will not be, probably, for a considerable time to come. These resources and the lusty health of our young country have made it possible heretofore for legislators to blunder with impunity. Constantly receiving large returns, labor and capital would not naturally be over-critical and hostile to each other. The young-country theory has also led to the encouragement of unlimited immigration, with which to settle our prairies and build up our towns. These new-comers do not, in fact, all go upon the land; but, arriving on our seaboard, instead of being drawn off entirely, they remain in the cities, like dirty pools of water in the streets. Indeed, the importation of uneducated, un-American, un-republican workmen from foreign lands is a problem in itself, and makes a strong demand upon all who can possibly do so to educate these masses, both economically and politically. Lawless communism, it is said advisedly, feeds on bad workmen. A saving mechanic is never a communist. To-day these men mean little to us; but when, by an increasing population and a denser settlement of the country, land becomes more scarce and valuable, profits on capital lower, and wages less, then even honest men, finding themselves pinched by a barrier of their own creation, brought into operation by natural laws, unless economically trained, will not know what is happening, and may in entire ignorance fly in the face of the law, and do in the United States somewhat of the things they are now doing in Europe. The day is more or less distant when this may happen, but it is coming nearer in proportion as the methods of men accustomed to conditions in old and crowded countries are brought here by a never ending stream of immigration.

The war has plunged us into the consideration of gigantic questions of an economic character, and the growth of our country in numbers and wealth is making a true understanding of them more necessary than ever to the prosperity of the nation, and a rising tide of interest in such studies is unmistakably evident. But these new and increasing demands are met by meagre and inadequate means in the great schools. It is a surprising fact that in some of the most important institutions there is no separate provision for such studies, and not even one settled instructor. Above all, we must educate in an intelligent manner, by stimulating investigation into home problems, and by encouraging the preparations of monographs on some out of the multitude of our economic questions. The best of the men in the university cannot now find a career in economic teaching, because few positions exist in this country as an object for honorable and ambitious students. Men find a profession in teaching Greek and Latin, but not Political Economy. When the community wakes up to a realization of this gap in the instruction of the land, and the importance of filling it, we may hope to see a more correct relation between means and needs than now exists.

________________________________

COLLEGE PROFESSORS AS ECONOMISTS.

Mr. J. Laurence Laughlin, in the Atlantic Monthly for April, appends his name to one of those egotistical screeds which serve to make those who teach political economy to boys contemptible in the sight of those who have occasion to practice legislative economies as practical statesman. Its fundamental assumption is that for want of the wisdom with such boys as Laughlin and Sumner possess nearly all that Hamilton, Gallatin, Chase, and Fessenden have done in America and quite all that Colbert, Napoleon, Pitt, Turgot, and Bismarck have done in Europe in an economic and financial way has been sad botchwork. Why do magazines like the Atlantic Monthly publish such ridiculous rant?

Instead of Chase and Fessenden having been in need of going to school to such vealy milksops as J. Laurence Laughlin, this college tutor shows on every page that he writes how greatly he needs the practical information which he could have got by attending for two or three years on the sessions of the Ways and Means Committee at Washington. Indeed, it is not legislators that need to be educated in economics by college professors, but college professors who need some means of picking up a few grains of sense by being brought into contact with actual legislation.

It is a singular fact that no man who has ever accepted a chair in a college as a teacher of political economy to boys has ever yet rendered any demonstrable service either to the cause of economic science or of legislation. Laughlin has the impertinence to say that, though Adam Smith wrote his “Wealth of Nations” in 1776, political economy was an unknown science to the American people before 1860. Does Mr. Laughlin mean to assert that Franklin, the intimate personal friend of Adam Smith and suggestor of some of his views, or that Hamilton, Madison, Jefferson, Clay, or Webster, Chase, Fessenden, Garfield, or “Pig Iron” Kelly are any less familiar with Adam Smith’s crudities, blunders, wisdom, and garrulous mud than Laughlin himself is? Adam Smith fell so far below Alexander Hamilton, and in many respects below Madison and Chase, in economic insight that while every commentator on Smith points out errors of fact and of theory, stupidities of ignorance and obliquities of vision on every page of the old scotch dullard and mugwump, we challenge Laughlin to point out with equal ease the ignorances and blunders in Hamilton’s economic papers or financial reports.

Adam Smith had the merit, however, of only styling his work as an “Inquiry.” It is the men who come after him who arrogate for his utterly unscientific, undefined meandering, inconsequential and self-contradictory fog-banks the quality of a science. Still Smith is helpful matter to a sensible legislator, because the latter can generally see on the face of Smith’s statements wherein the good Scotch plodder was wool-gathering, and could rectify Smith’s errors out of his own more modern and ample reading. The notion however, that Cairnes, Mill, Jevons, McLeod, Say, Lavelaye, or any other boy teachers have ever been helpful in matters of practical legislation is not warranted by facts. Ricardo was listened to with great respect by practical legislators, but he was a practical businessman like Franklin, the Careys, and Greeley, who had never undertaken the egotism of a pedagogue. The only economists America has yet produced are those who have either never or hardly ever sat in a professor’s chair. There seems to be something in the air of a school room which, if the professor remains in it until it conquers him, unfits him absolutely to mingle as a man among men in the affairs of men. It causes a cranky adoption of the most impracticable and erratic notions on the most inadequate basis of observation and fact, and at the same time inflates with a lofty and unapproachable egotism which precludes its possessor from meeting the views of an opponent with anything but epithets, however superior his opponent may be to himself in learning, experience, or sagacity. A precipitancy that has no nerves left for investigation and patience at criticism marks his every act and word. Laughlin shows this demoralizing precipitancy, so fatal to level-headed usefulness, by speaking of the silver coin, whose equal dignity with gold coin in all legal respects is irrevocably fixed in the letter of the Constitution of the United States, “as the false silver dollar,” thereby implying, of course that from 1853 to 1870, when silver happened to be worth more than gold, we must have been under a “false gold dollar.”

Laughlin also calls those navigation laws which have never existed either among barbarous or medieval nations, but which began in England under Cromwell, “barbarous and medieval.” He might as well call steam or the art of printing “barbarous and medieval.” Sensible man weary of these impudent epithets flung at them by young and graceless upstarts who have still their spurs to win in everything that distinguishes useful men from snobs.

SourceThe Inter Ocean (Chicago, Illinois) April 15, 1885, p. 4.

Image Source: Portrait (1885-88) of James Lawrence Laughlin. Harvard Art Museums/Fogg Museum, Transfer from the Carpenter Center for the Visual Arts.